You are on page 1of 170

https://t.me/UPSC_PDF www.UPSCPDF.com https://t.

me/UPSC_PDF
https://t.me/UPSC_PDF www.UPSCPDF.com https://t.me/UPSC_PDF
https://t.me/UPSC_PDF www.UPSCPDF.com https://t.me/UPSC_PDF

editorial
note

V.P. Gupta, CMD


Rau’s IAS Study Circle

APPROACH & STRATEGY FOR


GENERAL STUDIES - CIVIL SERVICES EXAM
The syllabus of Civil Services Exam for General Studies Also, the syllabus is a combination of traditional and
consists of syllabus from social sciences, behavioural dynamic aspects*. Traditional aspects are those which
sciences, philosophy, psychology and applied science & remain more or less the same like History, Culture, and
technology. Upon careful reading and analysis of the GS Physical Geography etc. The dynamic aspects are those
syllabus, it can be aptly described as inter-disciplinary as which are continuously changing like environmental,
well as multi-disciplinary. The strategy, thus, for the socio-economical and international issues etc. The
preparation of General Studies is a bit different from that of dynamic aspects require more attention as those are to
preparation of any optional subject, since none of the be covered on a day to day basis. Amongst dynamic
segments of the syllabus can be studied as isolated aspects, emphasis on issues which have a bearing on our
disciplines. So, the question is, “what is the right approach society, economy and administration test a candidate's
and strategy”? awareness about new challenges of our country and

Well, for that, one needs to keep in mind the basic fact that if society on various fronts.

the goal is to reach the highest level of achievement, then After understanding the true orientation of the syllabus,
one must have not only ample patience for details, but the second step is the selection of right study material(s)
should be able to keep the big picture regarding any theme for each segment of the GS syllabus and an efficient
in one's mind at the time of study. Thus the preparation method to study them. The basic strategy at the start of
strategy is to follow a mix of both micro and macro level preparation of the GS syllabus for both Pre and Main
approaches. exam is to refer to NCERT books on related topics. Refrain

The first step in this direction is the comprehension of the in the beginning from reading guidebooks, as they load

syllabus in its true orientation. One must look at the you with information, but with very little concept.

syllabus closely and in detail, and try to understand its THE DYNAMIC SIDE OF GENERAL STUDIES –
nature. RELEVANCE OF CURRENT AFFAIRS

The basic characteristic of the GS syllabus is that it is inter- The basic cue to understand the scope of the syllabus or
disciplinary as well as multi-disciplinary. The social, its difficulty level and analyse the direction of study has
economic, political and administrative aspects must merge been provided by UPSC in its notification:
to analyze a topic. For example, study of India's economic The nature and standard of questions in the General
growth will require good knowledge of disparities in society, Studies papers (Paper II to Paper V) will be such that a
administrative issues relating to implementation of well-educated person will be able to answer them
programmes, external factors, etc. In other words, the without any specialized study. The questions will be such
syllabus being inter disciplinary requires a synthesis of as to test a candidate's general awareness of a variety of
ideas. subjects, which will have relevance for a career in Civil

Download from:- www.UPSCPDF.com


https://t.me/UPSC_PDF www.UPSCPDF.com https://t.me/UPSC_PDF

Services. The questions are likely to test the candidate's suffice.


basic understanding of all relevant issues, and ability to For example: Let us take the issue o AADHAAR
analyze, and take a view on conflicting socio- economic
A lot of controversy was created when the government
goals, objectives and demands.
passed the Aadhaar Bill as the Money Bill. Given below
Now, the “variety of subjects, which will have relevance are some of the questions that may be framed on the
for a career in Civil Services” have been very well laid out issue:
by UPSC in its detailed syllabus for the examination. The
ü Why Aadhaar Bill was needed? (Related to Polity and
scope of developing expertise in all these subjects has
Governance topic)
been suggested by UPSC to be basic. So, what we need to
work towards after gathering information and ü Discuss the concerns related to the new Aadhaar Bill.

understanding in all these subject areas is the ability to (Related to Polity and Governance topic)

understand the relevant issues in them. ü Why has the government passed Aadhaar Bill as a

These relevant issues can be more or less defined as the Money Bill? Is it going against the federal character of

Dynamic Side of General Studies or Current Affairs. our constitution? (Related to Polity and Governance

Majority of all such issues will be actively dealt in the topic)

classrooms, and also captured and analysed in the ü What are the benefits of Aadhaar? (Related to Social
institute's monthly current affairs publication, FOCUS. Issues and Economy topics)
However, by the end of the course and after gathering all ü How Aadhaar may make subsidies more targeted
the information necessary and requisite tools of analysis, and be helpful in delivery of services? How it may
it is necessary to develop the ability to independently help in framing and implementing social security
figure all such issues, analyse them, and take a view on schemes? (Related to Social Issues and Economy
conflicting socio- economic goals, objectives and topics)
demands.
ü How may Aadhaar be useful for the intelligence
On analysis of previous years' question papers, we find agencies in their fight against terrorism and
that not only a good number of questions come directly radicalism? (Related to Internal Security topic)
from current affairs but also for the rest of the questions
ü What is snooping? Does Aadhaar create such a
current affairs provides a context on which the questions
possibility? (Related to Science and Technology topic)
are based. This very fact enhances the importance of
current affairs many-folds in respect to the proportion of ü What was Prism surveillance program of the U.S.? Do

marks it is allotted in GS papers, whether it is Prelims or you think Aadhaar may lead to such a program in

Main, as it acts a beacon to help us figure out what is India and breach of privacy? (Related to Science and

important and expected in exam out of the vast syllabus Technology/ International/ Polity and Governance

of GS. topics)

Apart from that current affair always comes handy while ü Aadhaar Bill says that the government agencies may

writing the Essay paper as one of the topics asked access the Aadhaar database in the case of a threat

invariably comes from current issues. It is also pertinent to national security. What are the concerns related to

to note that UPSC explicitly mentions that in interview the this? (Related to Internal Security topic)

candidate must be well aware of the current happenings. So, we observe that questions may be asked from the

Now the question comes, if the current affairs are so related dimensions and not directly from the news itself.

important, what should be the strategy to master it? It is This exemplifies how having command over the current

important to note that UPSC does not ask the questions topics helps in supplementing the preparation towards

directly from the news pieces which are transient in the dynamic parts of various sections of the GS syllabus

nature rather it asks questions from its related aspects (in this example these are Polity and Governance/ Science

(background, interpretation, related facts, etc). So just a and Technology/International/ Social Issue/ Economy/

mere reading of newspapers and magazines will not Internal Security sections).

Download from:- www.UPSCPDF.com


https://t.me/UPSC_PDF www.UPSCPDF.com https://t.me/UPSC_PDF

focus
CRITICAL NOTES FOR THE PREPARATION OF GENERAL STUDIES PAPERS

Chairperson and Managing Director (CMD) : Dear Students,


Mr. V.P. Gupta
Chief Executive Officer : Abhishek Gupta
Executive Editor : Ritesh Singh
FOCUS is the Study Circle's monthly publication of contemporary issues and
Editorial Team : Ritesh Singh, Gajanan Dwivedi, current affairs analysis. It is in fact a collective effort, by experienced
Naweed Akhter, Sanjeev Kumar Pandey, Vikas educators in varied themes, of identifying current affairs in light of the UPSC
Chauhan, Hiralal Mandal, Ankur Sharma, Jasmine
Sokhi, M Burhanuddin, Vrinda Gupta, Maharshi syllabus, supplementing them with background information, explaining
Sharma, Mangal Singh, Nagendra Pratap Singh, their related dimensions, lending them a generalist viewpoint and thus
Pradip Singh
producing critical notes for the preparation of General Studies' syllabus.
Design and Production : Rohit Rajput, Deepti
Gupta
The publication, as the name suggests, focuses solely on issues which are
The purpose of Focus is to expound the applied
relevant to the factual as well as applied aspects of the General Studies'
nature of General Studies to Civil Services
aspirants thereby making them aware of the real syllabus. That is how Focus covers all anticipated issues and themes for the
demand of UPSC exam.
upcoming Civil Services Examination (CSE).
The sources relied for selection and analysis of
issues are: Features
1. The Hindu, The Hindu (Sunday)
2. The Indian Express Part One | Current Affairs Analysis
3. The Times of India
4. Asian Age n This month edition covers analysis of news from the
5. The Tribune
6. The Economic Times
immediately preceding month.
7. Hindu Business Line n All news/issues are categorized and clubbed syllabus-wise
8. Frontline
9. Economic and Political Weekly (International Relations, Polity & Governance, Science & Technology,
10. World Focus Ethics, Integrity & Aptitude and so on) for efficient study.
11. BBC
12. Yojana n Background information has been added to provide context.
13. Kurukshetra
n Related and Additional information-dimensions have been explained
to add depth to your understanding.
n Maps and figures have been provided for associative and retentive
learning.

HEAD OFFICE & NEW DELHI CAMPUS


309, Kanchanjunga Building, Barakhamba Road, Part Two | Contributors Zone
Connaught Place, New Delhi-110 001
Tel : 011-23317293, 23318135-36, 23738906-07 n Articles by Rau’s professors and Research team

BENGALURU CAMPUS n Essays by Rau’s students


2nd Floor, AKS Plaza, 10 Industrial Layout,
Jyoti Niwas College (JNC) Road, 5th Block,
Koramangala, Bengaluru – 560 095 (Karnataka) Part Three | Practice Zone
Tel : 080-25535536, 7, 8, 9, 09916035536
n MCQ’s from the current affairs analysis covered in this edition with
JAIPUR CAMPUS
emphasis on Preliminary General Studies - Paper I
701, Apex Mall, Lal Kothi,
Tonk Road, Jaipur-302 015 (Rajasthan) n Descriptive/Essay type questions from important editorials in this
Tel : 0141-6450676, 093515 28027, 07877036167
edition and Case Studies with emphasis on Main General Studies
email : contact@rauias.com
Papers.
website : www.rauias.com

connect : www.facebook.com/rausias
For further understanding or discussion in any topic, please consult your
NO PART OF THIS PUBLICATION MAY BE REPRODUCED respective professors.
OR TRANSMITTED, IN ANY FORM OR MANNER OR BY
A N Y M E A N S - E L E C T RO N I C , M E C H A N I C A L ,
PHOTOCOPY OR OTHERWISE, OR STORED IN ANY Good Luck!
RETRIEVAL SYSTEM OF ANY NATURE WITHOUT THE RAU'S IAS STUDY CIRCLE
WRITTEN CONSENT OF THE COPYRIGHT HOLDER,
RAU’S IAS STUDY CIRCLE

Download from:- www.UPSCPDF.com


https://t.me/UPSC_PDF www.UPSCPDF.com https://t.me/UPSC_PDF

TABLE OF CONTENTS

Part ONE REAL ESTATE REGULATION IN INDIA – AN ANALYSIS

#ACT #INFRASTRUCTURE #GOVERNANCE 25

Current Affairs Analysis MALFUNCTIONING OF VVPAT

#ELECTIONS #ACT #GOVERNANCE 28

DRAFT SCHEME FOR CAUVERY


02 #FEDERAL ISSUE #ACT #CONSTITUTION 29

SC ON DOMESTIC VIOLENCE LAW FOR DIVORCED


INTERNATIONAL RELATIONS
WOMEN #JUDGMENT #ACT #SOCIAL ISSUE 31
# GS Paper (Prelims) & GS Paper II (Main) PARLIAMENTARY COMMITTEE REPORTS AS EVIDENCE
INDIA AND NETHERLANDS #JUDGMENT #COMMITTEE #CONSTITUTION 32
# BILATERAL RELATIONS 02 NEW BILL: CHANGES SOUGHT IN COMMERCIAL DIVISION
INDIA’S LATIN AMERICAN OUTREACH BENCH #AMENDMENT #ACT #GOVERNANCE 34
#INDIAN FOREIGN POLICY #BILATERAL RELATIONS 05

INDIA & PERU 06


INDIA AND PANAMA 06
INDIA AND GAUTEMALA 07
36
INDIA AND NEPAL

# BILATERAL RELATIONS 07 ECONOMIC DEVELOPMENT


INDIA AND MYANMAR
# GS Paper (Prelims) & GS Paper III (Main)
# BILATERAL RELATIONS 08
SERVICES SECTOR IN INDIA: OPPORTUNTIES AND
INDIA AND RUSSIA: SOCHI SUMMIT CHALLENGES #SECTOR OF ECONOMY 36
# BILATERAL RELATIONS 09
12 CHAMPION SERVICES SECTORS
IRAN NUCLEAR DEAL # SERVICES SECTOR 38
# INTERNATIONAL ISSUE 10
NATIONAL POLICY ON BIOFUELS, 2018
PANMUNJOM DECLERATION # RENEWAL ENERGY 38
# INTERNATIONAL ISSUE 14
NATIONAL WIND-SOLAR HYBRID POLICY
RESPONDING FIRST AS A LEADING POWER # RENEWAL ENERGY 40
# INDIAN FOREIGN POLICY
NATIONAL DIGITAL COMMUNICATIONS POLICY (NDGT)
(Courtesy: MoEA) 17 2018- DRAFT # TELECOMMUNICATION 42

PRADHAN MANTRI JAN VIKAS KARYAKARAM # SCHEME 43

GREEN REVOLUTION- KRISHONNATI YOJANA


20 # SCHEME #AGRICULTURE 45

GREEN SKILL DEVELOPMENT PROGRAMME (GSDP)


CONSTITUTION, POLITY AND GOVERNANCE
# SKILL DEVELOPMENT 47
# GS Paper (Prelims) & GS Paper II (Main)
VAN DHAN VIKAS KENDRAS (VDVK)
ANALYSIS: ROLE OF GOVERNOR # PROJECT # GROWTH AND DEVELOPMENT 48
#CONSTITUTION #ELECTIONS #GOVERNANCE 20
SAMARTH
ASSAM: PROTEST AGAINST CITIZENSHIP AMENDMENT # SCHEME 49
BILL, 2016

#SECURITY #GOVERNANCE #BILL #COMMITTEE 23

Download from:- www.UPSCPDF.com


https://t.me/UPSC_PDF www.UPSCPDF.com https://t.me/UPSC_PDF

FREIGHT VILLAGE IN HARYANA


67
# INFRASTRUCTURE 50

THE STATE/UT AGRICULTURE PRODUCE AND LIVESTOCK SCIENCE & TECHNOLOGY


CONTRACT FARMING AND SERVICES (PROMOTION & # GS Paper (Prelims) & GS Paper III (Main)
ACILITATION) ACT, 2018 # AGRICULTURE 51
AUTOMATED EXTERNAL DEFIBRILLATOR (AEDS)
FARMER PRODUCER ORGANISATION (FPO) #TECHNOLOGY 67
# AGRICULTURE 52
RUTHENIUM’S MAGNETISM
GAS TRADING HUB/EXCHANGE #GENERAL SCIENCE 67
# INFRASTRUCTURE #INDUSTRY 53
COSMIC MICROWAVE BACKGROUND (CMB)
FPIS ALLOWED TO INVEST IN TREASURY-BILLS 55 #SPACE 68
MUTUAL RECOGNITION AGREEMENT 55 GRACE-FO
CAPITAL ADEQUACY RATIO (CAR) 56 #SPACE 69
PM VYAYA VANDANA YOJANA 56 NIPAH VIRUS
PASIGHAT AIRPORT 57 #HEALTH 70
e-SIM 57 ISRO’S NEW PROPELLANT
DIRECTORATE GENERAL OF TRADE REMEDIES (DGTR) 57 #SPACE 71
INDIAN BUREAU OF MINES 58 IPC APPROVES ANIMAL-FREE TESTS FOR DRUGS

#BIOTECHNOLOGY 72

CAN ARTIFICIAL INTELLIGENCE HELP WEED OUT FAKE


NEWS IN A POST-TRUTH ERA? (courtesy: The Hindu) 74
59
SOCIETY AND SOCIAL JUSTICE
# GS Paper I & GS Paper II (Main)
76
SOCIAL AUDIT

# EMPOWERMENT #GOVERNANCE #CITIZEN GEOGRAPHY, ENVIRONMENT, BIODIVERSITY &


PARTICIPATION (Lead Topic) 59 DISASTER MANAGEMENT
SOCIAL INCLUSION
# GS Paper (Prelims) and GS Paper I & III(Main)
# INCLUSIVE GROWTH # EMPOWERMENT 61
OZONE LAYER DEPLETION
SWADHAR GREH #SCHEME #SOCIAL JUSTICE
#ENVIRONMENT CONSERVATION #POLLUTION 76
#WOMEN EMPOWERMENT 61
BONN CLIMATE CHANGE CONFERENCE
NATIONAL NUTRITION MISSION #ENVIRONMENT CONSERVATION 77
#SCHEME #SOCIAL JUSTICE #EMPOWERMENT 62 LAKE VICTORIA: SPECIES UNDER THREAT

GENDER BIAS #BIODIVERSITY #CONSERVATION 79

# SOCIAL ISSUE # HEALTH 63 TIGERS: FIRST INTER-STATE TRANSFER

NIDAAN SOFTWARE #BIODIVERSITY #CONSERVATION 80

#HEALTH #TECHNOLOGY (In short) 64 BLACK PANTHER

MORE ON SOCIAL AUDIT #BIODIVERSITY #CONSERVATION 80

# EMPOWERMENT #GOVERNANCE #CITIZEN PROTECTING BEES


PARTICIPATION (Courtesy: The Hindu) 64 #BIODIVERSITY #CONSERVATION #FOOD SECURITY 81

PARTY POPPERS BANNED

#POLLUTION #ENVIRONMENT 82

Download from:- www.UPSCPDF.com


https://t.me/UPSC_PDF www.UPSCPDF.com https://t.me/UPSC_PDF

SEA SPARKLING AND GLOBAL WARMING

#CLIMATE CHANGE 83
104
INDIA’S FRESHWATER STOCKS

#GEOGRAPHY #RESOURCES 84 MISCELLANEOUS


PUSHING FOR GREEN GDP RAFAEL NADAL: DID IT FOR THE 11TH TIME
# SUSTAINABLE DEVELOPMENT # TENNIS #GLOBAL TOURNAMENT 104
(courtesy: The Hindu) 58
NATIONAL FILM AWARDS 2018

# AWARDS 105

LINNEAN MEDAL

87 # AWARDS 105

CHINA’S TRANSFORMATION
HISTORY, HERITAGE & CULTURE # BOOKS AND AUTHORS 106

# GS Paper (Prelims) & GS Paper I (Main)

PREHISTORIC ROCK PAINTINGS IN DANGER

# ANCIENT INDIA 87
107
‘VACHANAS’ USED FOR VOTES

# MEDIEVAL INDIA 90 ETHICS, INTEGRITY & APTITUDE


SADHARAN BRAHMO SAMAJ # GS Paper IV (Main)
# MODERN INDIA 91
POSSIBLE SOURCE I: NEWSPAPER 107
LOKAMANYA TILAK
POSSIBLE SOURCE II: (OPEN PAGE, THE HINDU) 108
# PERSONALITY # MODERN INDIA 92
POSSIBLE SOURCE III: HISTORY 109
MAULANA AZAD
POSSIBLE SOURCE IV: POLITY 110
# PERSONALITY# MODERN INDIA 94
POSSIBLE SOURCE V: EMINENT AND ETHICAL
THE TURNING POINT IN 1932
PERSONALITIES 111
# MODERN INDIA
POSSIBLE SOURCE VI: MORAL SCIENCE STORIES 112
(Courtesy: The Hindu) 96
POSSIBLE SOURCE VII: BOOKS ON LIFE LESSONS 113

POSSIBLE SOURCE VIII: DOHAS 114

POSSIBLE SOURCE IX: ISSUE BASED MOVIES,


98 DOCUMENTARIES AND SONGS 114

POSSIBLE SOURCE X: CARTOON STRIPS FROM COMICS


SECURITY
AND NEWSPAPERS 115
# GS Paper III (Main)
AI IN NATIONAL SECURITY & DEFENCE

# INTERNAL SECURITY # TECHNOLOGY 98

CEASE-OP’S IN J&K
116
# INTERNAL SECURITY 100

NAVAL OUTREACH GLOSSARY


# EXTERNAL SECURITY 101 # Terms
SURYA KIRAN - XIII POPULATION BOTTLENECK
# MILITARY EXERCISE # BILATERAL EXERCISE 102 # BIOLOGY 116
PEACE IN NAGALAND SELF-SERVING BIAS
# SECURITY (Courtesy: The Hindu) 102 #HUMAN PSYCHOLOGY 116

Download from:- www.UPSCPDF.com


https://t.me/UPSC_PDF www.UPSCPDF.com https://t.me/UPSC_PDF

INTROSPECTION ILLUSION

#PSYCHOLOGY 116 PUBLIC PRIVATE PARTNERSHIP

IDENTIFIABLE VICTIM EFFECT #ECONOMY

#PSYCHOLOGY 117 RITESH KUMAR SINGH, FACULTY: G.S. CONTEMPORARY

NOCEBO EFFECT ISSUES, ANSWER WRITING & ESSAY, AND EXECUTIVE

# PSYCHOLOGY; MEDICINE 117 EDITOR 127

APOSEMATISM

# BIOLOGY 117
130
TERRAFORMING

# PHYSICS 117 LEAD ESSAYS


PEACE DIVIDEND ARE WORKING WOMEN IN INDIA GETTING FAIR DEAL?
# ECONOMY 118 ANISHA TIWARI, EX-RAU’S IAS STUDENT 130

PAIN POINT

# ECONOMICS 118

SURPLUS VALUE

#ECONOMICS 118
Part Three
WINDFALL TAX
practiCe ZONE
#ECONOMICS 118

COMPOUNDED ANNUAL GROWTH RATE (CAGR)

#ECONOMICS 118 134


ASSET ALLOCATION

#ECONOMICS 119
MULTIPLE CHOICE QUESTIONS (MCQS)
UNIFIED PAYMENT INTERFACE (UPI) for Prelims GS Paper – I

#ECONOMICS 119

141
Part TWO
ESSAY TYPE QUESTIONS
CONTRIBUTORS ZONE for Mains GS Papers – I, II & III

121 158
LEAD ARTICLES CASE STUDIES
OPERATION GREEN – ALTHOUGH LATE BUT RIGHT
for Mains GS Paper IV: Ethics, Integrity & Aptitude
INITIATIVE #ECONOMY

DR. J. C. SHARMA, FACULTY (GS: ECONOMY AND


OPTIONAL: ECONOMICS) 121

INDIA’S COASTAL BORDER MANAGEMENT

#SECURITY

ANKUR SHARMA, EDITORIAL TEAM

& EDUCATOR: DAILY NEWS SIMPLIFIED 124

Download from:- www.UPSCPDF.com


https://t.me/UPSC_PDF www.UPSCPDF.com https://t.me/UPSC_PDF

Part ONE

Current
affairs
analysis
logical . simple . targeted
analysis & explanation
of all relevant news of the month

Download from:- www.UPSCPDF.com


https://t.me/UPSC_PDF www.UPSCPDF.com https://t.me/UPSC_PDF

INTERNATIONAL RELATIONS
# GS Paper (Prelims) & GS Paper II (Main)

INDIA AND
With the recent visit of Indian Prime Minister to Netherlands, it becomes
NETHERLANDS necessary to understand the importance of this bilateral relationship. Let
us start this by understanding the meaning of Netherlands:
# Bilateral relations

x Netherlands is the term for the country as a


whole, located in Western Europe, bordered by
Germany and Belgium, and with U.K. a close
neighbour separated by North Sea.
x Holland refers to just the two major provinces
of North and South Holland of Netherlands, but
has long been accepted as a nickname for the
country of the Netherlands.
x Dutch is the term used for both the language
and the people of the Netherlands, but is
popularly used to describe anything ‘of the
Netherlands’ such as Dutch government, etc.
x Now, let us understand about India-
Netherlands relations.

INTERNATIONAL COOPERATION

Global Jointly asserted that there is a need to ensure that globalization is open and fair and that there is
Economic a need to reinvigorate global economic growth, accompanied by improved income and wealth
Vision distribution so as to reduce social exclusion, involuntary migration, and unemployment.

x Jointly asserted that global transformation needs to be shaped by appropriate policies and
institutions, guided by responsible leadership and rules based international order.
Global
x This global transformation should support a rules-based international order that upholds
Governance
agreed international norms, global peace and stability, and encourages inclusive growth and
sustainable development and a multipolar world.

x Netherlands has supported India’s accession to the MTCR, Wassenaar arrangement and the
Export Control Australia Group.
Regimes x Netherlands has also reaffirmed its support to India’s membership to the Nuclear Suppliers
Group.

India and Netherlands have asserted that connectivity initiatives must be based on key principles of:
Connectivity
x International norms and Rule of Law;
Infrastructure
x Openness and transparency;
& Governance
x Follow social and environmental standards;

FOCUS | June 2018 | RAU’S IAS 2

Download from:- www.UPSCPDF.com


https://t.me/UPSC_PDF www.UPSCPDF.com https://t.me/UPSC_PDF

International Relations

x Principles of financial responsibility;


x Accountable debt-financing practices;
x Respecting sovereignty and territorial integrity of other countries.
This is seen as an in-direct reference to Chinese connectivity initiatives such as Hambantota port,
China-Pakistan Economic Corridor (CPEC), among other which have violated above said principles.

International 9 Netherlands has signed the framework agreement of the International Solar Alliance.
Solar Alliance 9 Collaboration under ISA would mainly benefit the Dutch Caribbean islands.

COOPERATION ON SECURITY

x Both reaffirmed that terrorism cannot be justified on any grounds whatsoever it may be and
should not be associated with any religion, creed, nationality or ethnicity.
x Both countries also asserted that all countries should work towards rooting out terrorist safe havens
Against and infrastructure, disrupting terrorist networks and their financing channels, and halting cross-
terrorism border movement of terrorists including foreign fighters.
x Both countries would jointly work against Al Qaeda, Daesh/ISIS, Jaish-e-Mohammed,
HizbulMujahideen, Lashkar-e-Tayabba, and their affiliates as well as terrorist groups threatening
peace and security in South Asia and Europe.

x Both have asserted for an early conclusion of negotiations and the adoption of the Comprehensive
Convention on International Terrorism in the United Nations.
CCIT
x CCIT was first proposed by India in 1996 to develop a universal definition of terrorism, to ban all terrorist
groups and to make cross-border terrorism an extraditable offence.

Both have agreed to hold the bilateral cyber dialogue in 2018 with the intention to discuss and
cooperate on:
9 Internet governance

Cyber 9 Data protection


Terrorism 9 Cyber security policy
9 Computer Security Incident Response Team (CSIRT)
An MoU was signed between The Hague Security Delta and Hyderabad Security Cluster, India’s first
Cyber Security Cluster.

Defence Both countries intend to finalize the text on the Agreement on Mutual Protection of Classified
Agreement Information in the field of Defence.

COOPERATION IN SCIENCE AND TECHNOLOGY

x Both celebrated 10th anniversary of the Indo-Dutch collaboration in Science, Technology and
Innovation (STI).

Indo-Dutch x STI is a mission-oriented, impact-focused approach which is aimed at addressing common societal
challenges and open new markets in areas such as water, health, agriculture, and high-tech
STI
systems and IT.
x Both countries would specifically focus further upon affordable medical devices, urban water
systems, Big Data and Internet of Things (IoT).

x India and Netherlands would collaborate in the global fight against Anti-Microbial Resistance (AMR)
Fight against
under India’s National Action Plan on AMR.
AMR
x Both countries have selected Krishna District in the state of Andhra Pradesh for the

FOCUS | June 2018 | RAU’S IAS 3

Download from:- www.UPSCPDF.com


https://t.me/UPSC_PDF www.UPSCPDF.com https://t.me/UPSC_PDF

International Relations

implementation of a Pilot Project using the ‘One Health’ approach under the Indo-Dutch
collaboration on AMR.

India and Netherlands are cooperating in several projects on water management under the
framework of an MoU in the field of water management signed in June 2017. These projects are:
9 Waste2wealth approach to solid waste and waste water in the Hindon Basin;
9 Assisting leather industries in Kanpur and Unnao to adopt eco-friendly technologies;
Water
9 Conservation of water through efficient agriculture practices in the sugarcane industry of Uttar
Management
Pradesh;
9 Exchange of information on river modelling;
9 Re-use of sludge contributing to a transition towards a circular economy;
9 Further cooperation on the Clean Ganga campaign.

The first Indo-Dutch Centre of Excellence (CoE) in Vegetables was operationalized in Baramati,
Maharashtra.
Both countries would further cooperate on:
9 Boosting productivity of apples;
9 Manufacturing food processing equipment;
9 Breeding high-yielding vegetable seeds;
Agriculture
9 Providing capacity building and training to dairy farmers;
9 Cooperating in the areas of bovine genetics and food safety;
9 Supporting sustainable and climate resilient agriculture;
9 End the practice of stubble burning;
9 Assisting in setting up Food Valleys;
9 Investment in cold chain and food processing facilities.

x Both signed an MoU on Technical Cooperation in the Field of Spatial Planning, Water
Urban Management and Mobility Management with focus on smart city development.
Management x Satisfaction on the MoU signed between the State of Maharashtra and the City of Amsterdam in
the area of sustainable urban development, including waste2energy.

ECONOMIC COOPERATION

India and Netherlands launched the Start-Up Link initiative by Invest India and the Netherlands
Embassy in Delhi.
Start-Up’s
The initiative seeks to provide start-ups easy access to mentorship, partners, tools, services and key
information for success in India and the Netherlands.

India and Netherlands reaffirmed their commitment to work together with all members of the
WTO World Trade Organisation (WTO) in strengthening the centrality of the rules-based multilateral
trading system.

Both countries expressed support to re-engage actively towards re-launching of negotiations for a
BTIA comprehensive and mutually beneficial EU-India Broad Based Trade and Investment Agreement
(BTIA).

India & Netherlands would cooperate in stepping-up connectivity between both countries by the
Port
sea route through direct port-to-port contacts, cooperation in the area of port infrastructure
Infrastructure
development and shipping.

FOCUS | June 2018 | RAU’S IAS 4

Download from:- www.UPSCPDF.com


https://t.me/UPSC_PDF www.UPSCPDF.com https://t.me/UPSC_PDF

International Relations

x Netherlands has positioned itself as being India’s ‘Gateway to Europe’ for enhancing India-EU
trade and Indian investment in EU member countries.
x India is the 5th largest source of FDI into the Netherlands and the Netherlands is the 4th largest
Investment
source of FDI for India.
x Both countries have agreed to form a reciprocal Fast Track Mechanism to resolve issues
confronted by businesses and to promote greater two-way bilateral investments.

Companies and institutions from India and the Netherlands signed wide ranging
MoU’s/Agreements, in the field of:
9 Government to Government (G2G)
9 Knowledge institutions
9 Water

MoU’s Signed 9 Agrifood and horticulture


9 Hi-tech/IT/Space
9 Life Sciences & Health
9 Smart Cities
9 Business
9 Sustainability

INDIA’S LATIN AMERICAN OUTREACH


#Indian foreign policy #Bilateral relations
x Latin America is considered as the last
frontier of the Indian foreign policy.
(Latin America is a group of countries and
dependencies where Spanish, French and Portuguese
are spoken. It comprises of South America, Central
American countries and Caribbean islands.)
x In order to give more impetus for
diversified engagement with Latin American
nations, the Vice-President of India visited
Peru in South America and Panama and
Guatemala in Central America.
th
x The visit also commemorated the 55
anniversary of establishment of diplomatic
relations between India and Peru in 1963.
x Let us understand about India’s
engagement with each country within the
framework of India’s outreach to South
America and Central America.

FOCUS | June 2018 | RAU’S IAS 5

Download from:- www.UPSCPDF.com


https://t.me/UPSC_PDF www.UPSCPDF.com https://t.me/UPSC_PDF

International Relations

INDIA AND PERU

Æ International Cooperation

x India has an emerging bilateral relationship with Peru even though bilateral relations were
established 1960’s.
Emerging
x India’s engagement with Peru is mainly through international groups and organisations of which
Relationship
both countries are members of such as United Nations, Non-Aligned Movement (NAM) and G-
77.

x India has expressed its desire to engage more closely with the Pacific Alliance, particularly with
respect to trade facilitation, investment, and to examine prospects of Alliance-wide arrangements.
x Peru has agreed to cooperate in exploring ways and means to strengthen India-Pacific Alliance
relationship in which India currently has observer status.
Pacific
x The Pacific Alliance was created in the 2011 with Declaration of Lima by countries on the coast
Alliance
of Pacific ocean – Peru, Chile, Colombia and Mexico.
x Its stated purpose is to gradually move towards the free movement of goods, services, capital and
people and emerge into one common trading platform, the Latin American Integrated Market
(MILA).

x Peru has reiterated its support for India`s candidature for permanent membership in a
reformed and expanded United Nations Security Council.
x Both countries have emphasized the need for text-based negotiations in UNSC and serious
UN Reforms discussions on text-based negotiations for UNSC reforms.
x The current deliberation on UNSC reform has been conducted in form of informal discussions in
the UN. By having text-based negotiations, there would be formalization of the reform process of
UNSC and thereby produce one text that can be the basis for negotiation on UNSC reform.

Æ Economic Cooperation

India has become Peru’s seventh largest trading partner.


Bilateral
Both countries are currently engaged in talks for a comprehensive trade agreement covering goods,
trade
services and investments and intend for an early conclusion of the Agreement.

Renewable
India and Peru have signed an agreement to promote cooperation in new and renewable energy.
Energy

IT and ITES India has sent an expert team to discuss the Peruvian request for an extension of the activities of the
Education India-Peru Centre for Excellence in Information Technology (IPCEIT).

INDIA AND PANAMA

x Panama has included India as one of the countries for strengthening relations under its ‘Falcon
Policy’ which would streamline visa facilitation for Indians travelling to Panama.
Ease of x India has already e-visa facilitations extended to all Latin American countries including
Interaction Panama.
x Both countries also signed a MoU for Exemption of Visa for Diplomatic and Official Passports
Holders.

Developmental x India has extended Lines of Credit to form two Centres for Biodiversity and Drug Discovery
Projects respectively and a Centre for Innovation & Technology for Panama's economic development.

FOCUS | June 2018 | RAU’S IAS 6

Download from:- www.UPSCPDF.com


https://t.me/UPSC_PDF www.UPSCPDF.com https://t.me/UPSC_PDF

International Relations

x India under its Indian Technical Economic Cooperation Programme (ITEC) has extended
training in the English language for Panamanians, in-addition to previously existing
programmed under ITEC.

Parliamentary Parliamentarians of India and Panama will interact with each other under the Parliamentary
Groups Friendship Group which extends to the entire Latin American region.

x An MoU was signed between India and Panama in the field of agricultural research and
education.
x Both countries agreed to expedite on-going discussions to strengthen cooperation in the fields
MoU’s Signed
of Exchange of Tax Information, Economic Cooperation, Maritime Cooperation, Air Services,
Traditional Medicines, Culture, and Space.
x India has also invited to Panama to join the International Solar Alliance.

x Panama has welcomed India's proposal for setting up a Telemetry, Tracking and Tele
Command (TTC) Earth station in Panama for supporting operations of geostationary
S&T communication and meteorological satellites of the Indian Space & Research Organization
Cooperation (ISRO).
x This can also guide research and analysis by Panamanian experts in the fields of agriculture,
meteorology, disaster management, climate change, etc.

x India has one of the largest diaspora in Central America in Panama at roughly 15,000.
Indian x Most Persons of Indian Origin had travelled to Panama at the end of 19th CE to build their
Diaspora railway lines and then subsequently also contributed to the construction of the Panama Canal
which connects the Atlantic Ocean and the Pacific Ocean.

INDIA AND GAUTEMALA

UN x India and Guatemala have provided each other reciprocal support for the Non-permanent
Reforms membership of the Security Council wherein India for 2021-22 and Guatemala for 2031-32.
x Guatemala has also extended support to India’s membership of an expanded UN Security Council.

Solar x India has been training and providing other assistance to the Solar Mamas as part of India’s
Energy cooperation development assistance to Guatemala.
x India has also invited Guatemala to join the International Solar Alliance.

Wildlife x A wildlife agreement is under consideration between India and Guatemala.


Agreement x Both countries would cooperate in sharing knowledge, technologies and approaches with regard to
India’s preservation of tiger population and similar Guatemalan experience with Jaguars.

Post the visit of K.P. Oli, Nepal’s Prime Minister in April this year, the
INDIA AND
Prime Minister of India in a reciprocal gesture visited Nepal in May, 2018.
NEPAL The contours of K.P. Oli’s visit to India were covered in detail in May
edition of FOCUS. Henceforth, here we are covering the further additions
# Bilateral relations on the basis of the Indian P.M. visit.

COOPERATION IN CONNECTIVITY AND COMMERCE

Nepal has expressed concern over the growing trade deficit with India.
Trade
To address this concern, both countries have formed an Inter-Governmental Committee meeting on
Deficit
Trade, Transit and Cooperation. The purpose:

FOCUS | June 2018 | RAU’S IAS 7

Download from:- www.UPSCPDF.com


https://t.me/UPSC_PDF www.UPSCPDF.com https://t.me/UPSC_PDF

International Relations

9 To control unauthorized trade;


9 To jointly initiate a comprehensive review of the bilateral Treaty of Trade;
9 To consider amendments to the Treaty of Transit and related Agreements;
9 To further facilitate Nepal's access to the Indian market;
9 To facilitate Nepal's transit trade to other international markets.

Air India and Nepal would expand cooperation in civil aviation sector, including early technical discussion
Connectivity on additional air entry routes to Nepal.

x Both jointly laid the foundation stone of 900 MW Arun-III hydro-electric projects in Nepal.
x Arun III is the largest hydropower project to be undertaken in Nepal.
Arun - III x Both countries have agreed to further enhance bilateral cooperation in power sector in line with the
bilateral Power Trade Agreement.
x Currently, India sells electricity to Nepal and not purchase from it.

CULTURAL COOPERATION

x India and Nepal would further strengthen the close religious and cultural ties wherein a Nepal-India
Ramayana Ramayana Circuit connecting Janakpur, the birthplace of Sita, with Ayodhya and other sites
Circuit associated with the epic Ramayana.
x A direct bus service has been initiated between Janakpur and Ayodhya.

x Prime Minister of India also attended civic receptions in Kathmandu and Janakpur. The civic reception
was held mainly with an audience which comprised of the general public of Nepal.
Civic
x This is different in comparison to the earlier civic receptions held in Madison Square Garden in US
Reception
and similar receptions in other countries which mainly had an audience of Persons of Indian Origins
(PIO) and Indian diaspora.

INDIA AND Recently, External Affairs Minister of India, Sushma Swaraj, visited Myanmar.
During the bilateral meetings, boundary and border related issues, peace and
MYANMAR security matters, developments in the Rakhine State, including return of
displaced persons, India’s development assistance to Myanmar, on-going
# Bilateral
projects, and other issues of mutual interest were discussed.
relations Let us understand about India and Myanmar relations.

FOCUS | June 2018 | RAU’S IAS 8

Download from:- www.UPSCPDF.com


https://t.me/UPSC_PDF www.UPSCPDF.com https://t.me/UPSC_PDF

International Relations

Seven Agreements/MOUs were signed during the visit. The major MoU’s signed were on:
9 Agreement on Land Border Crossing;
MoU’s Signed 9 Restoration and Preservation of Earthquake Damaged Pagodas in Bagan;
9 Assistance to the Joint Ceasefire Monitoring Committee;
9 Forming several Industrial Training Centres.

An agreement on Land Border Crossing will enable people from both countries to cross the land
Land Border
border with passport and visa, including for accessing health and education services, pilgrimage and
Crossing
tourism.

x India reiterated its commitment to help Myanmar in addressing issues related to Rakhine State
under the bilateral Rakhine State Development Programme.
x The first major project under the programme by India is the construction of prefabricated
Rakhine housing in Rakhine State to meet the needs of displaced persons.
Development x Rakhine state of Myanmar is significant for India’s Kaladan Multimodal Transit Transport
Project wherein the project includes a waterways component on Kaladan River from Sittwe
located in Rakhine to Paletwa in Myanmar and then a road component from Paletwa to Zorinpui
on the India-Myanmar border in Mizoram.

x Myanmar reaffirmed India’s request in implementing the Kofi Annan led Rakhine Advisory
Commission’s recommendations with regards to Rohingya refugees.
Rohingya
x India also reiterated its support to an agreement between Bangladesh and Myanmar for the
Refugees
repatriation of the displaced Rohingya refuges to Myanmar that had fled from the Rakhine state
of Myanmar into Bangladesh.

Archaeological Survey of India is undertaking the restoration and preservation of ancient pagodas
Pagoda
and structures in Bagan, the historic Ananda temple and various monuments with the help from
Restoration
Myanmar.

Prime Minister of India and Russian President Vladimir Putin held their first
INDIA AND informal Summit in the city of Sochi in the Russian Federation in May, 2018.
RUSSIA: The Summit is based on the similar format as the informal summit that
occurred between Indian PM & Chinese president Xi Jingping in Wuhan.
SOCHI The Sochi meet intended to provide an occasion for both leaders to deepen
SUMMIT their friendship and to exchange views on international and regional issues.
Sochi Summit was held in the backdrop of the US CAATSA law, so let us also
# Bilateral
understand Indian response to US law and its probable future impact may on
relations Indo-Russia defence relations.

THINGS TO UNDERSTAND

Informal There were no agreements or joint statement that was signed between India and Russia.
Nature The summit is considered ‘informal’ as there was no pre-defined agenda to the interaction between
leaders of India and Russia.

Global India and Russia have designated their relationship as Special and Privileged Strategic Partnership.
Outlook They shared the view that India and Russia have an important role to play in:
9 Contributing to an open and equitable world order;

FOCUS | June 2018 | RAU’S IAS 9

Download from:- www.UPSCPDF.com


https://t.me/UPSC_PDF www.UPSCPDF.com https://t.me/UPSC_PDF

International Relations

9 Recognition of each other’s respective roles as major powers;


9 Common responsibilities for maintaining global peace and stability;
9 Building a multipolar world order.
Both countries have decided to intensify consultation and coordination with each with regard to the
Indo-Pacific Region and continue working together through multilateral organisations such as the
United Nations, SCO, BRICS and G-20.

Terrorism x India and Russia have emphasized the importance of restoring peace and stability in Afghanistan in
in an atmosphere free from the threat of terrorism.
Afghanistan x Both countries have agreed to work together towards achieving this objective by combating
terrorism and radicalisation in all its forms and manifestations.

Economic India and Russia have agreed to institute a Strategic Economic Dialogue between NITI Aayog of India
Dialogue and the Ministry of Economic Development of the Russian Federation, to identify greater synergy in
trade and investment.

CAATSA LAW

x United States under the Countering American Adversaries through Sanctions Act (CAATSA)
mandates US President to impose sanctions on entities which have transactions with listed Russian
About
defence and intelligence entities.
x However, US President can issue waivers on case-by-case basis from US sanctions.

x India has close defence partnership with Russia with several Indian government and private
companies engaged in partnership with Russian defence and intelligence entities. Therefore, Indian
Impact on
entities can come under the sanctions of US CAATSA law.
Indo-Russia
x The sanctions would negatively impact India’s plans to purchase S-400 Triumf air defence systems
Defence
from Russia. Also, India’s legacy weapons systems are of Soviet and Russian origin and it needs to
maintain defence ties with Moscow to keep them operational.

x India has asserted that it won’t allow other countries to dictate terms of India’s defence engagement.
x India has also been lobbying in US for allowing India-specific waivers from the sanctions under the
CAATSA law.
x If US imposes sanctions, they would negatively impact Indo-US trade rather than Indo-Russia trade
Indian because the CAATSA adhering US companies would be restricted to trade with sanctioned Indian
Response entities in terms of joint defence ventures, supply of defence equipment’s, etc.
x The Indian entities, if they choose to ignore US sanctions, can continue to maintain commercial
relations with Russia.
x If imposed on India, CAATSA has the potential to heighten India’s traditional insecurity about the
United States as being a reliable security partner.

IRAN The United States under the Donald Trump administration has recently
withdrawn from the Joint Comprehensive Plan of Action (JCPOA) or what is
NUCLEAR popularly known as the Iranian Nuclear Deal.
DEAL This is bolt from the blue as it has overturned conciliatory process vis-à-vis Iran.
On this, let us understand the immediate cause, global fallout and reactions to
# International the US decision. This would enable you to grasp and comprehend the context of
Issue the on-going coverage and any future changes in the Iranian nuclear deal issue.

FOCUS | June 2018 | RAU’S IAS 10

Download from:- www.UPSCPDF.com


https://t.me/UPSC_PDF www.UPSCPDF.com https://t.me/UPSC_PDF

International Relations

CHRONOLOGY OF EVENTS

CONTOURS OF THE JCPOA

x It is officially called the 'Joint Comprehensive Plan of Action (JCPOA).


x It was signed between Iran and the P5 nations of UNSC which are United States, United Kingdom,
France, China and Russia and plus Germany and European Union in 2015 and was implemented in
January, 2016.
Aim of the
x The deal had two primary aims: first to prevent Iran from building a nuclear weapon by ensuring
deal
curbs on its nuclear programme and the second aim was the lifting of international sanctions on
Iran.
x It pursued the ‘dual-track’ policy of ‘applying pressure in pursuit of a constructive engagement
and a negotiated solution.

x Iran has two main facilities – Natanz and Fordo, for forming enriched Uranium. Iran will have
Uranium
limited research on enrichment at Nantanz till 2024 and no enrichment at Fordo until 2031. Iran
Enrichment
cannot form enriched uranium.

FOCUS | June 2018 | RAU’S IAS 11

Download from:- www.UPSCPDF.com


https://t.me/UPSC_PDF www.UPSCPDF.com https://t.me/UPSC_PDF

International Relations

x Iran had almost 20,000 centrifuges before the deal, but Under the JCPOA, it was limited to installing
no more than 5,060 until 2026.
x Iran's uranium stockpile was reduced by 98% which must not be exceeded until 2031.

x Iran had been building a heavy-water nuclear facility near the town of Arak, wherein spent fuel
from a heavy-water reactor contains plutonium suitable for a nuclear weapon.
x Under the JCPOA, Iran would redesign the reactor so it could not produce any weapons-grade
Plutonium
plutonium, and that all spent fuel would be sent out of the country as long as the modified reactor
Enrichment
exists.
x Iran will not be permitted to build additional heavy-water reactors or accumulate any excess
heavy water until 2031.

International Atomic Energy Agency (IAEA), the global nuclear watchdog would:
9 Continuously monitor Iran's declared nuclear sites;
9 Verifies that no fissile material is moved covertly to a secret location to build a bomb;
Role of IAEA
9 Iran also agreed to implement the Additional Protocol to their IAEA Safeguards Agreement, which
allows inspectors to access any site anywhere in the country they deem suspicious.
9 Iran will have 24 days to comply with any IAEA access request until 2031.

x The time taken for Iran to produce enough weapons-grade enriched uranium fuel for a single
nuclear weapon is called the break-out time.
Break-Out x US under Obama administration had asserted that prior to the deal, Iran if it chooses to, could
Time have built a nuclear weapon in two to three months.
x The Obama administration said the JCPOA would remove the key elements Iran would need to
create a bomb and increase its break-out time to one year or more.

x If Iran refuses to comply with any IAEA access request, an eight-member Joint Commission -
Joint including Iran - will rule on the issue.
Commission
x It can decide on punitive steps based on majority votes, which can including the imposition of
for
sanctions.
resolution
x If the Joint Commission cannot resolve a dispute, it will be referred to the UN Security Council.

x Iran has agreed to the continuation of the UN arms embargo for up to five years, although it could
UN end earlier if the IAEA is satisfied that its nuclear programme is entirely peaceful.
Sanctions x A UN ban on the import of ballistic missile technology will also remain in place for up to eight
years.

x Before the deal, the export of Iranian goods and services esp. oil & gas, Iranian ability to trade in US
Dollar & Euro, investment in Iran such of Indian investment in Chabahar port, etc. were severely
restricted by the US-led sanctions.
Economic
x The estimated loss due to sanctions was more than $160bn in oil revenue from 2012 to 2016 alone.
Impact
x After the deal, sanctions were lifted thereby removing the trade & investment hurdles in Iran and
Iran went from being in economic recession to relatively good growth.
x Under the deal, Iran gained access to more than $100bn in assets frozen overseas.

US DECISION AND ITS FALLOUT

US
US under Trump administration has asserted that the deal had several loopholes and flaws.
reasoning
9 The sunset clause which allows JCPOA termination in 2025 from 10 years after adoption day (18th
for

FOCUS | June 2018 | RAU’S IAS 12

Download from:- www.UPSCPDF.com


https://t.me/UPSC_PDF www.UPSCPDF.com https://t.me/UPSC_PDF

International Relations

withdrawal October, 2015);


9 It did not target Iran's ability to build others forms of ballistic weapons;
9 The deal provides for a short time-period for sub-sunset clauses;
9 There were no concessions to curb Iran's involvement in terrorism through Hezbollah in Israel;
9 The deal does not limit Iran’s involvement in the Yemen & Syrian conflict;
9 Moreover, rise in Iranian economic profile has increased its regional presence which is being
opposed by US & its regional allies Israel & Saudi Arabia.

The European Union may take formal steps such as a blocking statute to protect EU firms from U.S.
sanctions on Iran, by:
9 It forbids EU companies from complying with the extraterritorial effects of US sanctions;

Reaction 9 It allows EU companies to recover damages arising from US sanctions from the person causing
from E3 them;
(France, 9 It nullifies the effect in the EU of any foreign court judgments based on US sanctions.
Germany x European Commission would also remove hurdles for the European Investment Bank (EIB) to
and U.K.) & finance activities in Iran
EU
x UK has said it will continue to implement the deal and has further asserted for US to not to hinder
the other parties from their continued implementation of deal.
x France and Germany have spoken in favour of safeguarding and implementing the deal and
asserted the positive reports from IAEA of Iranian compliance.

x China has said that it will continue to safeguard and implement the deal.
Chinese and
x Russia has said that the US decision was new confirmation of US incompetence, and underscored
Russian
that the US, not Iran, is now technically in violation of the deal.
Response-
Criticism x The US decision allows for China and Russia to increase their clout in Persian Gulf region, due to
the need of both US & Iran to have Russian & Chinese support for their position on the deal.

x Israeli Prime Minister Benjamin Netanyahu has pledged his full support for US decision to
withdraw from the deal and has accused Iran of having secret nuclear facilities.
Israeli and
x Saudi Arabia has also issued its support of US withdrawal from the agreement and the
Saudi
reinstatement of economic sanctions. Saudi Arabia has previously warned it will pursue nuclear
Response-
weapons if Iran does the same.
Support
x The US decision increases the power of hardliners in Iran who had opposed the deal, which may
further increase hostilities between Iran and Israel and Saudi Arabia.

x The JCPOA would continue as long Iran and other signatories remain committed to the deal.
x If Iran would violate any aspect of the deal, the UN sanctions will automatically snap back into place
for 10 years, with the possibility of a five-year extension.
x US can negotiate for a new deal but it would need the support of other countries, which in the
Future of
current scenario, seems very unlikely.
the Deal
x Rules and trust provide the moral foundation of the non-proliferation regime, but the recent US
decision to flout rules based global order would reduce the trust deficit of ensuring rogue
countries such as North Korea from signing on to or abiding by new or existing agreements,
protocols or regimes.

IMPACT ON INDIA

Indian x India has stated that India only recognises UN led sanctions, and not country specific sanctions.

FOCUS | June 2018 | RAU’S IAS 13

Download from:- www.UPSCPDF.com


https://t.me/UPSC_PDF www.UPSCPDF.com https://t.me/UPSC_PDF

International Relations

Response x Moreover, India has also taken a diplomatic position by stating that the Iranian nuclear issue
should be resolved peacefully through dialogue & diplomacy by respecting Iran's right to peaceful
use of nuclear energy and the interest of the international community to maintain the peaceful
nature of Iran's nuclear programme.

x Is yet to be seen on India-Iran trade relations.


x However, Iran is India's third largest oil supplier. The US decision to re-impose secondary sanctions
on Iran may hamper India's ability to conduct oil trade with Iran in US Dollar.
x But, if European countries do not impose secondary sanctions, India can continue to conduct oil
trade in Euro.
Impact on x In addition, India and Iran can further explore the option of trading within their local currencies
trade within the Rupee-Rial trading mechanism. India and Iran had agreed to set up a Joint Committee
to ascertain a Rupee-Rial Arrangement and Asian Clearing Union mechanism, therefore to
establish functional payment channels.
x US decision has further increased volatility of the price of oil thereby increasing impact on India’s
economic resources and any increase in price of oil would negatively India’s fiscal deficit & current
account deficit.

x The pre-deal sanctions caused the Indian investment in Chabahar port to be postponed by roughly
a decade.
Chabahar
x India can negotiate with US on the re-imposition of US sanctions on Iran by asserting that Chabahar
port
port is required for economic development of Afghanistan which is a common goal of India & US.
investment
x However, the impact assessment can only be analysed with further development once the US
sanctions are in full force.

PANMUNJOM DECLERATION
# International Issue

The leaders of North Korea Kim Jong-un and South


Korea Moon Jae-in met in the demilitarised zone
between the two countries to discuss several on-
going conflicts such as the denuclearisation of the
Korean peninsula and the official end to the
Korean War, etc.
Moreover, MoS V.K. Singh also visited North Korea
in May, 2018.
So, let us understand about the Panmunjom
declaration, its impact on global politics and India
and contours of the visit of V.K. Singh to
Democratic People’s Republic of Korea (DPRK) i.e.
North Korea.

OVERVIEW

Background x The 1950-53 Korean War was halted with a truce, not a treaty. So, South Korea and a U.S.-led

FOCUS | June 2018 | RAU’S IAS 14

Download from:- www.UPSCPDF.com


https://t.me/UPSC_PDF www.UPSCPDF.com https://t.me/UPSC_PDF

International Relations

to Korean U.N. force are technically still at war with North Korea.
War x North Korea’s army, the American commander of the United Nations Command and the
commander of the ‘Chinese People’s volunteers’, who were not officially claimed by China at the
time signed an armistice in Panmunjom that led to cessation of hostilities and temporary truce.
x South Korean leaders at the time opposed the idea of a truce that would divide the Korean
peninsula and therefore they did not sign the armistice.
x Therefore, a deal towards an official end to the war requires the consent of all the involved parties
and possibly also China.
x The 1953 armistice formed the Demilitarized Zone (DMZ), which is 240 km along the 38th
parallel.
x Panmunjom is a village located near the DMZ that hosts the Joint Security Area which mainly
serves a purpose of meets between North Korea & South Korea.

x A deal among North Korea, the US, Japan and South Korea in the 1990s was meant to give the
Background to North civilian nuclear power without the ability to build a weapon, but the reactor was never
De- finished.
Nuclearisation x North Korea pledged to relinquish its nuclear programme in 2007 in exchange for sanctions
relief and fuel, but later pulled out of that agreement and expelled inspectors in 2009.

The main objectives that South Korea and North Korea have agreed upon under the Panmunjom
Declaration are:
9 Completely suspend all hostile acts on land, sea and air against each other and work together to
make the Demilitarized Zone (DMZ) an actual peace zone;
9 Make the western Northern Limit Line (NLL) a sea of peace to prevent accidental military
clashes, and take actual measures to guarantee safe fishing activities;
9 Reaffirm non-aggression agreement and work together to strictly observe it;
Objectives of
9 Work together to realize phased arms reduction in line with tension reduction and confidence
recent
building;
Declaration
9 Actively push to hold three-way talks, involving the two Koreas and the United States, or four-
way talks, involving the two Koreas, the U.S. and China, to declare an end to the Korean War;
9 Change the armistice into a peace treaty and establish a lasting, solid peace regime;
9 Reaffirm the shared goal of realizing a nuclear-free Korean Peninsula through the complete
denuclearization;
9 Make joint efforts to further expand the good move towards the continued development of
inter-Korean ties, peace and prosperity on the peninsula and reunification.

x There has been a major shift in the conduct of diplomatic relations by North Korea. The North
Korean international diplomacy was marred heavily with hostilities with threats of nuclear war,
conduct of military tests near Japanese and South Korean territory, naval skirmishes with US,
South Korean & Japanese naval vehicles, threat of military strikes against US territory of Guam,
etc.
North Korean
x However, an invitation of South Korea, Kim Jung-un sent a North Korean delegation to the
Diplomacy
Winter Olympics in South Korea in February, 2018.
x Soon afterwards, Kim Jung-un met the Chinese President Xi Jingping in March, 2018 and
engaged in several high-level diplomatic meetings with the US and South Korea thereby leading
to the setting up of the Panmunjom summit.
x Kim Jong-un diplomatic manoeuvres seem to be to engage with all stakeholders, including the

FOCUS | June 2018 | RAU’S IAS 15

Download from:- www.UPSCPDF.com


https://t.me/UPSC_PDF www.UPSCPDF.com https://t.me/UPSC_PDF

International Relations

U.S. and China, who have competing interests in the region.


x Unlike prior negotiations, North Korea has developed nuclear and missile programmes to
sufficient degree that they can act as a deterrent against any attack and give North Korea
significant negotiating power.

x The US diplomacy under Donald Trump has been marred with aggressive posturing with North
Korean leader Kim Jong-un with threats of annihilation, regime change, etc.
x However, both countries have also engaged in high-level diplomatic meets and talks which led to
US
recent fruitful meeting in Singapore leading to conciliatory tones.
Involvement
x The US President stated halting of joint U.S.-South Korea military exercises.
x Still, the outcomes of the meet is work in progress. More will be covered in forthcoming FOCUS
magazine.

x China is the largest trading partner of both North and South Korea with some 90% of North
Korea’s trade passing through China.
x China has been opposed to military intervention in North Korea since it fears that any war in
Korean peninsula would cause a surge of North Korean refuges into China and justifies further
Chinese
US involvement in East Asia.
Involvement
x It has been argued that China is nudging North Korea to negotiate a US withdrawal of military
troops and bases from South Korea for North Korean de-nuclearisation.
x There is also a fear in China that closer ties between North and South Korea would decrease
Chinese leverage over North Korea.

INDIAN GESTURE: VISIT OF V.K. SINGH TO NORTH KOREA

x India and DPRK established formal diplomatic relations in 1973 and 2018 marks the 45th year of
Reason for establishment of diplomatic relations.
Visit x V.K. Singh is the first Ministerial level visit from India to DPRK after a gap of almost 20 years to
commemorate the anniversary.

Korean India reiterated its support to the joint peace initiative of North Korea and South Korea and their
Peace Talks efforts towards establishment of peace and prosperity on the Korean Peninsula.

x There had been reports that Pakistan aided North Korea’s nuclear program, in exchange for North
Pakistani Korean aid in its missile program.
Proliferation x North Korea has emphasized that as a friendly country it will never allow any action that would
create concerns for India’s security, including with regards to nuclear proliferation.

x India and North Korea have emphasized to explore possibilities of cooperation in areas of mutual
interest including vocational education, agriculture, pharmaceuticals, promotion of Yoga and
Fields of
traditional medicines.
Cooperation
x Both sides agreed to strengthen people to people contacts through educational and cultural
exchange.

FOCUS | June 2018 | RAU’S IAS 16

Download from:- www.UPSCPDF.com


https://t.me/UPSC_PDF www.UPSCPDF.com https://t.me/UPSC_PDF

International Relations

Viewpoint
Responding First as a Leading Power
# Indian Foreign Policy
(Courtesy: MoEA)

Read and ponder!

The emerging concept of India as a “first responder” reflects the country’s growing capability and increasing
willingness to assume the role of a leading power. By contributing its resources to prevent or mitigate regional and
international crises, India is demonstrating its commitment as a responsible actor in the international order. Beyond
narrow self-interest, such contributions help project India’s soft power abroad and portray India in a positive light. They
also reflect India’s expanding sphere of influence and capacity to shape events abroad.

The international order is facing a variety of transnational challenges that occasionally erupt into acute crises. Whether
it is a natural catastrophe, an Internet disruption, or a sudden financial shock, the repercussions are often massive,
with regional and global implications. This is where individual states must take immediate action, to mitigate the crisis
impact and avoid further escalation. By taking the lead, they are providing a public good to the international order,
supporting smaller or less capable states in dire need of assistance.

As the Indian economy surges on and the country emerges as one of the key actors in the international order,
expectations are consequently growing about India’s capacity to provide such support as a first responder to crises
beyond its borders. Commenting on this rising role, ex-Foreign Secretary Dr. S. Jaishankar thus emphasized that India’s
foreign policy dimension is “to aspire to be a leading power, rather than just a balancing power ... (and) a willingness to
shoulder greater global responsibilities.”

While the concept of “first responder” has generally been interpreted quite narrowly, focusing on humanitarian
disasters, a broader definition illustrates how India has played a crucial role in assuming these “global responsibilities”
by responding to a variety of crises in its neighbourhood and beyond. This is particularly apparent in seven issue-
areas given here under:

1. Natural disasters

When the forces of nature unleash their fury on South Asia, the Indian government and military forces have played a
critical role in supporting neighbouring countries in relief operations. After the 2004 tsunami, India deployed 14 Navy
vessels, nearly 1,000 military personnel and several dozen helicopters and airplanes to Sri Lanka. In 2007, in the
aftermath of cyclone Sidr, India was one of the few countries allowed to provide relief to Myanmar and provided critical
rice supplies to address food emergencies there. In 2015, less than six hours after Nepal was rattled by a tragic
earthquake taking almost 9,000 lives, the Indian Air Force flew in National Disaster Response Force (NDRF) rescue
team. Over the next days, India lead on the ground, landing 32 flights with 520 tonnes of relief and more than fifty
medical, Army engineering and other rescue operation teams. In 2014, the Indian Navy was the first to arrive in the
Maldives to provide fresh drinking water to more than 150,000 of its citizens facing an acute supply crisis. More
recently, in response to Cyclone More (2017), India was the first to respond to the devastating floods in Sri Lanka and
Bangladesh. As reflected in relief provided to Pakistan in 2005 and 2010, Indian support transcends political
considerations.

FOCUS | June 2018 | RAU’S IAS 17

Download from:- www.UPSCPDF.com


https://t.me/UPSC_PDF www.UPSCPDF.com https://t.me/UPSC_PDF

International Relations

2. Expatriate evacuation operations

When crises erupt abroad, India is often the first on the ground to protect the lives and assets of its nationals. By
mobilizing its consular officers, New Delhi has also provided safe evacuation to citizens from other countries. In 2015,
for example, India extricated almost 2,000 nationals from 48 different countries, including many from the European
Union, the United States and neighbouring countries. The Ministry of External Affairs, Air India, and the Navy and Air
Force have emerged as key actors in conflict zones, especially in the Indian Ocean and Gulf region, normally operating
as first responders coming to the rescue of thousands of foreign nationals in distress.

3. Non-traditional security challenges

The Indian Navy has emerged as the Indian Ocean’s default first responder to non-traditional security challenges. To
combat piracy in the Gulf of Aden since 2008, it deployed almost thirty warships that have escorted more than 1500
ships and thwarted around thirty piracy attempts. India was a founder-member of the Contact Group on Piracy off the
Coast of Somalia (CGPCS) and has taken a lead role in coordination efforts among different naval forces in the region.
When airplanes or ships go missing in its extended neighbourhood, India has often been among the first responders to
participate in search and rescue missions. In 2014, the Indian Coast Guard deployed around the Andaman and Nicobar
Islands in initial efforts to locate the missing Malaysian Airlines flight MH370.

4. Post-conflict relief and rehabilitation

India has often taken the lead in supporting countries going through post-conflict processes which require expert
resources and significant funding. After the end of the armed conflict in Sri Lanka, in 2009, India provided more than
USD one billion worth in lines of credit and grants for projects in education, health, transport connectivity, and training.
Focusing on relief, rehabilitation and reconstruction, India’s early efforts played a crucial role in facilitating Nepal, Sri
Lanka, and Afghanistan’s return to normalcy years of violent conflict. As one of the largest contributors to United
Nations peacekeeping missions, India’s “blue helmets” have also served as first responders to mitigate dozens of
conflicts around the world, leading efforts on the ground to facilitate stabilization and reconstruction.

5. Regime support

Whenever friendly governments face the risk of a coup or instability threatening regional security, India has often
stepped in as a first support responder. In 1988, for example, in response to a request from the Maldives, India
activated Operation Cactus to deploy its military and ensure regime continuity in Male. Located in one of the world’s
most conflict-ridden regions, whenever requested by neighbouring countries, India has also played a constructive role
in offering its mediation services to ensure peaceful and inclusive settlements. New Delhi is also a democratic first
responder, deploying expert technical support to assist transitioning democracies to design their new constitutions
and hold free and fair elections.

6. Financial assistance

India plays a little-known but crucial role as a first responder in the region to support friendly governments facing
financial crises. Under a special currency swap mechanism instituted in 2012, the Reserve Bank of India has provided
critical assistance to the governments of Bhutan, Maldives and Sri Lanka whenever they face foreign currency liquidity
shortages. By coming to the rescue of their respective monetary authorities, New Delhi has demonstrated its
commitment to financial stability and economic growth in the region, strengthening governance to wither crises.

FOCUS | June 2018 | RAU’S IAS 18

Download from:- www.UPSCPDF.com


https://t.me/UPSC_PDF www.UPSCPDF.com https://t.me/UPSC_PDF

International Relations

7. Refugee flows

Whenever people fear for their lives in South Asia, they often look up to India first. India has consistently provided an
emergency safe haven for refugee and minority populations from across South Asia. Whether they are affected by
violent conflict or any type of persecution, most displaced people’s routes to safety pass through India, taking
advantage of its default open doors policy. Since 1947, this includes people from Tibet, East Pakistan, Afghanistan,
Burma/Myanmar, and Sri Lanka, leading the current UN Secretary General to speak of India’s refugee policy as a model
for other countries.

India’s contribution as a first responder in these areas above will continue to increase as its economy grows in size and
openness. In his message to the heads of Indian missions abroad, in 2015, Prime Minister Modi thus “urged them to use
this unique opportunity to help India position itself in a leading role, rather than just a balancing force, globally.” As India
expands its horizons, it will keep stepping up to take the lead where other countries are reluctant, unwilling or
incapable to do so.

This first respondent tradition must be further studied and promoted, because it reflects India’s deep commitment to
assume a driving role in the international order. And it will also have to be endowed with adequate resources and
capabilities for India to move even quicker and farther beyond its borders.

FOCUS | June 2018 | RAU’S IAS 19

Download from:- www.UPSCPDF.com


https://t.me/UPSC_PDF www.UPSCPDF.com https://t.me/UPSC_PDF

CONSTITUTION, POLITY AND GOVERNANCE


# GS Paper (Prelims) & GS Paper II (Main)

ANALYSIS: ROLE OF GOVERNOR


#Constitution #Elections #Governance
The use of discretionary power of Governor became more visible and prominent after the declaration of
result by the Election Commission for the Karnataka Assembly Election. Bhartiya Janta Party (BJP) emerged
as the single largest party but did not have enough seats to form the government on its own. However, post
declaration of results, Congress gave support to Janta Dal Secular (JDS) and together both political parties
claimed government formation.
However, the Governor decided to call the single largest party to form government and also allowed 15 days
to prove majority. This was opposed by both post-coalition partners and raised constitutional issues on the
misutilisation of discretionary power of the Governor.
In this regard, a case was filed in the Supreme Court in midnight hours challenging the decision of the
Governor. This highlighted the role of the governor of a state who plays a multi-faceted role and is a key
functionary in balancing centre-state relationship.
Here, let us understand about the evolution of the post of Governor in India prior to independence and how
the constitutional functions and limitations of the post of Governor as envisaged by the constitution has
significance in modern times.

HISTORICAL PERSPECTIVE

x The Government of India Act, 1919 introduced dyarchy which was then replaced by The Government of India Act,
1935 (GOI, 1935) which established provincial legislative assemblies elected from a limited franchise.
x GOI, 1935 made elaborate provisions regarding the role and responsibilities of the Governor and also provided him
with certain discretionary powers to control the law and order of his province.
x The Governor of a province was appointed by His Majesty and the executive authority was exercised either directly
by the Governor or through officers’ subordinate to him.

Powers of Governor under GOI, 1935 Discussion on role of Governor in the Constituent Assembly

x The Act gave governor certain x When partition of India was palpable and in the garb of partition,
discretionary powers regarding aid and some provinces and princely states were thinking about leaving
advice by his council of minister. Indian Union. Thus, it was during these times that the discussions on
x The Act also empowered the governor the roles and responsibility of the position of Governor were being
to issue proclamation in case of failure discussed in the Constituent Assembly.
of constitutional machinery of states. x Thought process “…. The Governor being the agency of the Centre is
x The Act provided for certain Special the only guarantee to integrate the various Provinces or States and will
Responsibilities for the Governor see that the Central policy is sincerely carried out. Therefore, the
regarding preservation of law and order: Governor's discretionary powers should not be interfered with.”

9 prevention of any grave menace to the x After much discussions and deliberation, it was agreed upon that
peace or tranquility; the role of governor was crucial and it was necessary to bestow him
with certain discretionary power.
9 safeguarding legitimate interests of
minorities; x Also because, territorially, India was slowly trying to collage as a

FOCUS | June 2018 | RAU’S IAS 20

Download from:- www.UPSCPDF.com


https://t.me/UPSC_PDF www.UPSCPDF.com https://t.me/UPSC_PDF

Constitution, Polity and Governance

9 securing of the peace and good single union comprising of provinces, princely states and
government of partially excluded areas. commissioner’s provinces.
x The Act also provided him to secure x Thus, it was envisaged that governor must be in direct relation
administrative relations between with the Centre to keep the centre informed and updated about
federation provinces and states. any fissiparous tendencies emerging in any states or provinces.

WHAT IS IN THE CONSTITUTION?

x Considering the value of the position of Governor as a crucial link between the Centre and
States, the constitution makers retained discretionary power of the Governor as mentioned in
GOI, 1935.
Discretionary
x Governor’s discretionary power also tilted the balance somewhat in favour of the Centre as
power
certain amount of centralisation of power was necessary to avoid disintegration of provinces.
retained
x However, some concerned members raised the issue of misuse of discretionary power of
Governor. They were assured that the Governor would remain only a constitutional post and
would have no power to interfere in the day-to-day administration of the State.

x This article imposes an obligation upon the Centre to ensure that each state government is
carried on in accordance with the Constitution along with ensuring its protection from external
aggression and internal disturbance.
x So, this article in a way sanctions the governor to use Article 356 in case of breakdown of
constitutional machinery in a state. Article 356 also provides Centre the freedom to act even without
Article 355 Governor’s report in case of breakdown of constitutional machinery in any state.
x Thus, the dual function of the governor is at display when he submits a report to the Centre on
breakdown of constitutional machinery.
x Here, he acts as a representative of the Centre as he is appointed by the President of India
(Article 155) and takes a constitutional oath to preserve, protect and defend the constitution under
Article 159.

x The Executive Power of a State shall be vested in the Governor who shall be the head of the State.
The Governor is appointed by the President and shall hold office during the pleasure of the
President. Thus, effectively the governor acts as the head of the state and also as a
representative of the Centre.
x Controversy at times arises when both functions of the Governor overlap and the question
arises as to which function supersedes which in this era of multi-party system. This dual function
of the Governor in recent times has created enough controversies where different parties enjoy
Controversy power at the Centre and in the States.
due to dual x In most of the recent held Assembly elections in Goa, Manipur, Meghalaya and Karnataka, no
function political party had majority on their own. Hence, in such a situation the role of governor became
crucial as he has discretion to call either -
9 the single largest party,
9 political parties having pre-poll alliance or
9 different political parties who may stitch a post poll alliance
x Use of Article 356 is another ground where the governor can report to the President about the
constitutional failure of state at his discretion.

Note: The use of such discretionary powers at times thus becomes controversial when different political parties are in
government at the Centre and in the state. It is important to note here that discretion given by the constitution has a

FOCUS | June 2018 | RAU’S IAS 21

Download from:- www.UPSCPDF.com


https://t.me/UPSC_PDF www.UPSCPDF.com https://t.me/UPSC_PDF

Constitution, Polity and Governance

purpose and such powers needs to used only to fulfill the mandate of the constitution and not of any political party. In
this regard, let us understand the importance of judgment of S.R. Bommai, Sarkaria Commission and Punchhi
Commission.

S.R. Bommai v Union of India


x In 1989, S.R. Bommai was the Chief Minister (CM) of Karnataka. Some members of the party defected which
raised suspicion on majority for the government.
x The CM suggested the Governor to call the Assembly session for a floor test. The Governor ignored the suggestion
and without trying to find another alternative to form the government asked the President to impose Article
356. Bommai challenged the proclamation in the Karnataka High Court.
x The Karnataka High Court dismissed the petition without questioning the grounds on which proclamation was
issued. The Court also ruled that recourse to floor test was not a pre-requisite to the sending of report to the
President.
x The case was appealed in the Supreme Court where it gave the following orders to be followed in case of
proclamation issued under Article 356:
1. The President exercises his power under Article 356 (1) on the advice of the Council of Minister.
2. The question whether the Chief Minister has lost majority support in the Assembly must be decided on the
floor of the House and not in Governor’s chamber.
3. Governor must try to explore the possibility of an alternative ministry before asking the President to impose
Article 356(1).
4. The material facts on which proclamation was issued can be judicially reviewed to ascertain whether such
facts had any rational nexus to the action proposed or were done in bad faith or by arbitrary decision.
5. The dissolution of State Assembly is not an automatic outcome of every proclamation under Article 356. The
dissolution of the Assembly prior to the approval of the Proclamation under Article 356(3) shall be invalid.
x Thus, the judgment on S.R. Bommai provided certain limitations on the use of discretionary power of the governor
under Article 356.

Sarkaria Commission
The central government set up the Justice R S Sarkaria Commission in June 1983 to examine the relationship and
balance of power between state and central governments. The Commission suggested that in choosing a Chief
Minister, the Governor should be guided by the following principles:
x The party or combination of parties that command the widest support in the Legislative Assembly should be
called to form the government.
x The Governor’s task is to see that a government is formed and not to try to form a government that will pursue
policies that he approves.
x If no party has a majority, the Governor has to invite:
a) a pre-poll alliance,
b) the largest single party that is able to gain majority support,
c) a post-election coalition that has the required members,
d) a post-election coalition in which partners are willing to extend outside support.
x The Commission recommended that whoever is appointed as the Chief Minister, must seek a vote of confidence
in the Assembly on the floor of the House within 30 days of taking over.
x The Governor should not resort to mechanisms where determining of majority of the government is done outside

FOCUS | June 2018 | RAU’S IAS 22

Download from:- www.UPSCPDF.com


https://t.me/UPSC_PDF www.UPSCPDF.com https://t.me/UPSC_PDF

Constitution, Polity and Governance

the assembly.

Punchhi Commission -2007


x It was headed by former Chief Justice of India M. M. Punchhi, and was set up in April 2007 to take a fresh look at
the roles and responsibilities of governments at various levels, and the relations between them.
x The Commission recommended that there should be clear guidelines for the appointment of Chief Ministers, so
that there was some sort of regulation on the discretionary power of the Governor.
x It said that a pre-poll alliance must be treated as one political party, and laid down the order of precedence that
the Governor must follow in case of a hung House:
1. Group with the largest pre-poll alliance commanding the largest number;
2. Single largest party with support of others;
3. Post-electoral coalition with all parties joining the government;
4. Post-electoral alliance with some parties joining the government, and the remaining, including Independents,
supporting from outside.

Several political groups, civil societies groups and residents of


Assam have been protesting against the Citizenship (Amendment)
ASSAM: PROTEST Bill, 2016, which seeks to make foreign illegal immigrants from

AGAINST certain specific communities eligible for Indian citizenship. As per


the protestors, the proposed Bill is in complete violation of Assam
CITIZENSHIP Accord of 1985 as it intends to give legal sanction to some of the
illegal immigrants from Bangladesh.
AMENDMENT BILL,
On this, a Joint Parliamentary Committee (JPC) was constituted to
2016 analyse the provisions of the Bill.

#Security #Governance #Bill In the Brahmaputra valley, many organisations have submitted a
memorandum to JPC opposing the amendments as proposed in
#Committee the Bill. However, the situation seems quite opposite in the Barak
valley where most of the citizens agree on the proposed Bill.

In this backdrop, let us understand about key provisions of Citizenship Act, 1955, changes proposed through
Citizenship (Amendment) Bill, 2016, National Register of Citizens (NRC), key provisions of Assam Accord and how
the Bill contravenes certain provisions of Assam Accord.

INDIAN CITIZENSHIP ACT, 1955 ON ILLEGAL MIGRANTS

x The Indian Citizenship Act, 1955 provides for acquisition and determination of Indian Citizenship.
x As per the 1955 Act, a person may become an Indian citizen if they are born in India or have Indian parentage or
have resided in the country over a period of time.
x However, illegal migrants are prohibited from acquiring Indian citizenship under the 1955 Act.
x As per the Act, an illegal migrant is a foreigner who
9 Enters the country without valid travel documents, like a passport and visa, or
9 Enters with valid documents, but stays beyond the permitted time period.
x Illegal migrants may be imprisoned or deported under the Foreigners Act, 1946 and the Passport (Entry into India)
Act, 1920.

FOCUS | June 2018 | RAU’S IAS 23

Download from:- www.UPSCPDF.com


https://t.me/UPSC_PDF www.UPSCPDF.com https://t.me/UPSC_PDF

Constitution, Polity and Governance

x These Acts empower the Union government to regulate the entry, exit and residence of foreigners in India.

Section 6A of Indian Citizenship Act, 1955


x Section 6A was added through an amendment in the Citizenship Act and it pertains to special provision as to
citizenship of persons covered by the Assam Accord.

x It mentions that all those who came to Assam on or after 1st January 1966 but before 25th March, 1971 from
Bangladesh and since then have been ordinarily resident in Assam, must register themselves with the government
of Assam.

NATIONAL REGISTER OF CITIZENS (NRC) UPDATED

x National Register of Citizens, 1951 was a register prepared after the conduct of the Census of 1951.
x Any citizen who does not find his/her name in NCR shall be considered as foreigner under Foreigner’s Act, 1946
and shall be liable for deportation.
x NRC regards all migrants who have come to India after March, 24, 1971 as illegal irrespective of their religion.
x Such illegal immigrant needs to be deported back to Bangladesh as per Assam Accord.
x The aim of updating NRC list of 1951 is to compile names of genuine citizens residing in Assam and in the process
detect foreigners, who may have illegally entered the state after 24th March, 1971.

ASSAM ACCORD

Assam Accord was signed between Government of India, state government of Assam and the leaders of Assam
movement i.e. All Assam Students’ Union (AASU) and the All Assam Gana Sangram Parishad (AAGSP) on 15th August,
1985. The leaders of Assam movement were against granting of Indian citizenship to illegal migrants from Bangladesh.
x As per the Accord, 1st January, 1966 shall be the base date and year for purposes of detection and deletion of
foreigners.
x Foreigners, who came to Assam after 1st January, 1966 and upto 24th March, 1971 shall be detected in accordance
with the provisions of the Foreigners Act, 1946 and the Foreigners (Tribunals) Order 1964.
x Names of Foreigners so detected will be deleted from the electoral rolls in force.
x Such persons will be required to register themselves before the Registration Officers of the respective districts.
x On the expiry of a period of ten year following the date of detection, the names of all such persons which have
been deleted from the electoral rolls shall be restored.
x Foreigners who came to Assam on or after March 25, 1971 shall continue to be detected, deleted and expelled in
accordance with law.

x The Bill amends the Citizenship Act, 1955 to make illegal migrants who are Hindus, Sikhs, Buddhists,
Jains, Parsis and Christians from Afghanistan, Bangladesh and Pakistan, eligible for citizenship.
x Under the Act, one of the requirements for citizenship by naturalisation is that the applicant must
have resided in India during the last 12 months, and for 11 of the previous 14 years.
Citizenship
x The Bill relaxes this 11 years’ requirement to 6 years for persons belonging to the above
Amendment
mentioned six religions and three countries.
Bill, 2016
x Under the Citizenship Act of 1955, foreigners who come to India without valid travel documents, or
stay in the country beyond their visa period, are considered illegal migrants.
x The Bill however, allows certain illegal migrants to stay in India without being imprisoned or
deported.

FOCUS | June 2018 | RAU’S IAS 24

Download from:- www.UPSCPDF.com


https://t.me/UPSC_PDF www.UPSCPDF.com https://t.me/UPSC_PDF

Constitution, Polity and Governance

CITIZENSHIP (AMENDMENT) BILL, 2016 V. NRC

Citizenship (Amendment) Bill, 2016 NRC

x This Bill allows citizenship status to some illegal x NRC does not discriminate migrants on the grounds of
immigrants who came to India after March 24, religion whereas the Citizenship Amendment Bill
1971 based on religious identities. discriminates on the grounds of religion.
x This means that illegal migrants who are Hindus, x NRC puts the onus on migrants to prove their status of
Sikhs, Buddhists, Jains, Parsis and Christians from residence prior to 1971 based on series of documents
Afghanistan, Bangladesh and Pakistan will not be which will help them in registering themselves as citizens.
deported or imprisoned for being in India without Whereas this is not the case with Citizenship Amendment
valid documents. Bill.

x Acceptance of such illegal immigrants by Bangladesh remains a critical issue.


x Modalities of deportation are still not clear as it will involve negotiations with Bangladesh on various
Unresolved angles including that of security.
issues x Places where these illegal immigrants shall be deported.
x Issue of discrimination on grounds of religion as the provisions of Amendment Bill can be challenged
on grounds of violating Article 14 of the Indian Constitution.

REAL ESTATE REGULATION IN INDIA – AN ANALYSIS


#Act #Infrastructure #Governance
The real estate sector in India caters to the need of infrastructure along with job creation on a sustainable
basis. Post liberalisation saw the emergence of this sector and it involved big players from the Indian
industry. However, investments in real estate have become one of the best ways to park illegal and
unsolicited wealth by the corrupt through benami transactions.
Thus, with an aim to curb black money conversion into real estate and also to protect interest of genuine
home buyers, government of India enacted The Real Estate (Regulation and Development) Act, 2016 which
st
came into effect on 1 May, 2017.
It has now been more than a year since the Act came into effect but its implementation is still far from
satisfactory. Most of the provisions of the Act including framing rules have not been complied by some of the
state governments. In this analysis, let us go through the legislative history of the Act, important provisions
and key challenges which still remains a concern for the effective regulation and development of real estate
sector in India.

The Broad Framework on which 2016 Act has been proposed are:
9 Mandatory registration of projects with the Authority;
9 Mandatory Public Disclosure of all project details with Real Estate Authority;
Broad
9 Mandatory registration of Real Estate Agents;
Framework
9 Establishment of Regulatory Authority/Tribunals at State Level;
9 Provision of penalty and compensation for better consumer protection.
This is expected to promote professionalism, standardisation and orderly growth which will boost

FOCUS | June 2018 | RAU’S IAS 25

Download from:- www.UPSCPDF.com


https://t.me/UPSC_PDF www.UPSCPDF.com https://t.me/UPSC_PDF

Constitution, Polity and Governance

private investment in housing sector.

SALIENT FEATURES OF RERA

x According to RERA, each state and Union territory will have its own regulator and set of rules to
govern the functioning of the regulator.
Regulator
x Government of two or more States or Union territories may establish one single Authority.
x Government may establish more than one Authority in a State or Union territory.

x It shall be mandatory for every promoter/builder or a real estate agent to apply for registration
to state regulator of the real estate project which the builder aims to develop and sell.
Mandatory x The builder/promoter cannot advertise, market, book, sell or invite persons to purchase any
Registration part of the property before applying for registration to such state regulator.
x It shall be the responsibility of each state regulator to register real estate projects and real
estate agents operating in their state under RERA.

x Promoters shall declare original sanctioned plans, changes made later, fresh timeline for
Disclosures completion of ongoing projects to the state regulator.
x The details of all registered projects will be put up on a website for public access.

x Granting of commencement certificate by the authority allows a builder to begin development


Commencement
works on an immovable property as per the sanctioned plan.
& Completion
x Similarly, completion certificate by the authority signifies that the real estate project has been
Certificate
developed according to the sanctioned plan, layout plan and specifications as approved earlier.

x The promoter of a real estate development firm has to maintain a separate escrow account
for each of their projects where minimum 70 per cent of the money from investors and buyers
Separate has to be deposited.
Account x The developer, within three months of applying for registration of a project with the Real
Estate Regulatory Authority shall deposit in a separate bank account 70 per cent of the amount
collected and unused for ensuring completion of ongoing projects.

If the promoter defaults on delivery within the agreed deadline, they will be required to return the
In case of
entire money invested by the buyers along with the pre agreed interest rate mentioned in the
default
contract based on the model contract given by RERA.

In case of delay by the promoter in handing over the house to the owner, developers will be
In case of delay required to refund or pay compensation to the allottees with an additional interest rate within 45
days of the project getting delayed.

x Liability of developers for structural defects for five years.


Imposition of
x Imprisonment of up to three years for developers and up to one year in case of agents and
liability
buyers for violation of orders of Appellate Tribunals and Regulatory Authorities.

IMPLEMENTATION AND ADJUDICATION

Central Advisory
Powers of RERA Real Estate Appellate Tribunal
Council

CAC shall advice on x To impose penalty or interest regarding x The Act provides for the
matters concerning contravention of obligations by the establishment of the Appellate
implementation of the promoters, the allottees or the real estate Tribunal by the State Government

FOCUS | June 2018 | RAU’S IAS 26

Download from:- www.UPSCPDF.com


https://t.me/UPSC_PDF www.UPSCPDF.com https://t.me/UPSC_PDF

Constitution, Polity and Governance

Act, on major questions agents. to hear appeals from the orders or


of policy, protection of x Can refer matters to Competition Commission decisions or directions of the
consumer interest and of India regarding distortion of free Authority and the adjudicating
to foster growth and competition in the real estate market. officer.
development of the real x The Appellate Tribunal is to be
x Can call for information, record reasons and
estate sector. headed by a sitting or retired Judge
conduct investigations on any complaint by
the allottees. of the High Court with one judicial
and one administrative/technical
member.

x Due to stringent norms of the Act, non-serious developers are finding it difficult to sustain in
the competitive real estate market specially regarding the requirement of maintaining a separate
escrow account to deposit 70% of money from investors.
Benefits of
x Non-compliance will eventually force non-serious developers out of the competition which will be
RERA
beneficial for the home buyers.

x It will also make the sector more organised and will help in job creating opportunities at
various levels including skilled, semi-skilled and unskilled jobs.

x So far only 20 states have framed rules under RERA for registration of builders/promoters.

x Some states such as Uttar Pradesh has diluted the provisions under its state rules to benefit
builders.

x Some states have even diluted provisions regarding penalty for non-compliance.

x Only 6 states have set up the Online Portal as per the Act where details about any project are to be
Key uploaded.
challenges x In North-eastern states, RERA has been challenged on certain constitutional grounds as in some
remain cases land belongs to community and autonomous councils.

x Recently, Insolvency Law Committee has proposed to treat homebuyers as “unsecured financial
creditors” in case of insolvency proceedings of a promoter/builder. This may increase the delay of
returning of money to a home buyer by the promoter/builder because as per IBC, it will then
become mandatory for the builder to return money first to “secured financial creditor.” Thus the
conflict between RERA and IBC needs to be solved as it is against consumer interest.

As per a recent notification by Central Government, by June 30, 2018 –


x Every state must establish a permanent regulator.
Recent
x All states have to align their state regulations strictly as per norms provided by Central Government
notification
and do away with any dilutions in their state laws.
for course
correction x Bring all incomplete projects within the preview of RERA including the ongoing projects.
x Websites of RERA in all states must become functional to facilitate proper access by public regarding
information on real estate development projects.

FOCUS | June 2018 | RAU’S IAS 27

Download from:- www.UPSCPDF.com


https://t.me/UPSC_PDF www.UPSCPDF.com https://t.me/UPSC_PDF

Constitution, Polity and Governance

MALFUNCTIONING OF VVPAT
#Elections #Act #Governance
In recently held by-elections for Lok Sabha and Assembly seats, there were reports of malfunctioning of
Voter Verifiable Paper Trail (VVPAT) machines which resulted in delay and even re-polling for some booths.
This incident has raised questions on the efficacy of Electronic Voting Machines (EVMs) and VVPAT for the
upcoming state assembly and Lok Sabha elections of 2019.
There were complaints from Kairana (U.P.). The opposition parties raised doubts about efficacy of such EVMs
and VVPAT machines before the Election Commission (EC) and also demanded restoration of ballot paper for
the upcoming elections.
Malfunctioning of some of the EVMs were also reported in the recently held Gujarat and Karnataka Assembly
elections. Such reports have increased the suspicion regarding tampering of election process across political
spectrum.

x Voter-verified paper audit trail (VVPAT) is a method of providing feedback to voters using a
ballot less voting system.
x VVPAT is intended as an independent verification system for electronic voting machines
that allows voters to verify that their votes are cast as intended and can serve as an additional
barrier to changing or destroying votes.
x Under VVPAT, a printer-like apparatus is linked to Electronic Voting Machine (EVM). When a vote
VVPAT is cast, a receipt is generated showing the serial number, name and symbol of the
candidate. It confirms the vote and the voter can verify the details.
x The receipt, once viewed, goes inside a container linked to the EVM and can only be accessed by
the election officers in rarest of rare cases.
x The system allows a voter to challenge his or her vote on basis of the paper receipt for the first
time. As per a new rule, the booth presiding officer will have to record the dissent of the voter,
which would have to be taken into account at time of counting.

x In a Public Interest Litigation (PIL) in the year 2013, the Supreme Court directed the Election
Commission of India (ECI) to introduce VVPAT in a phased manner.
x Thus, accordingly ECI started using VVPAT at selected constituencies for elections to State
S.C. on VVPAT
Assemblies and also in Parliament Election for 2014.
x ECI has also confirmed that VVPAT shall be used in all the parliamentary constituencies for the
conduct of elections to Lok Sabha in 2019.

x The preliminary fact finding reports submitted to EC have indicated that two sensors of EVM
namely Contrast Sensor and Length Sensor malfunctioned due to excessive exposure to
Reasons of
illumination.
malfunctioning
x Thus, sensitivity to extreme weather conditions and exposure to light is being considered
disclosed by
as one of the main reasons for malfunctioning of EVMs.
E.C.
x EC also blamed the relative inexperience of polling officers handling VVPAT, compared to the
ballot and control units for the electronic voting machines (EVMs) that have been in use for
much longer.

x The EC has asked the Manufacturers - Electronics Corporation of India Limited (ECIL) and
Suggestions
Bharat Electronics Limited (BEL) and Technical Expert Committee (TEC) to suggest
for
additional design improvements including suggestions on any layout changes at the polling

FOCUS | June 2018 | RAU’S IAS 28

Download from:- www.UPSCPDF.com


https://t.me/UPSC_PDF www.UPSCPDF.com https://t.me/UPSC_PDF

Constitution, Polity and Governance

improvement stations to prevent excessive exposure to heat in future.


x Manufacturers have also been asked to do a detailed technical analysis of VVPAT machines.
x EC has decided to make stricter checking process prior to its approval for use in elections.
x EC has also adopted improvement in hardware of EVM as recommended by TEC of the
Commission to prevent auto shutdown of VVPATs due to excessive light and heat.
x EC has also suggested to further strengthen and streamline the training programmes of
polling officials to minimise failures due to human errors in handling EVMs and VVPAT for the
upcoming elections to be held in India.

x Upcoming general elections in 2019 in the summer months of April and May.
x It is to be seen whether VVPAT will withstand the heat and scale of the elections.

Key challenge x Another challenge is proper training – There is a need for awareness along with proper training
of the polling officials in terms of understanding the technicality of the EVMs and its handling
during pre and post polling process across the country. Polling officials at booth level are key to
the holding of smooth and fair elections in India.

Î RELATED INFORMATION : BY-ELECTION

x Section 149 of the Representation of People’s Act, 1951 provides for by-election in case of casual vacancies in the
House of the People.
x When the seat of a member elected to the House of the People becomes vacant or is declared vacant or his
election to the House of the People is declared void, then Election Commission by notification can hold an election
to fill the vacancy.
x However, if vacancy is caused on a seat reserved for Scheduled Caste or Scheduled Tribe, then person filling the
seat through election must be from the same category.
x By-elections to fill the vacancy shall be held within a period of six months from the date of the occurrence of the
vacancy. However, by-election shall not be held if:
9 If less than one year remains for the conduct of next general elections. (Eg. Election is to be held in December and
vacancy arose in February of the same year)
9 If the Election Commission in consultation with the Central Government certifies that it is difficult to hold the bye-
election within six months.

DRAFT SCHEME FOR CAUVERY


#Federal Issue #Act #Constitution
The Supreme Court in its judgment of February, 2018 had directed the Central Government to draft a
scheme under Section 6A of the Inter-State River Water Dispute Act, 1956 within a span of six weeks to
implement the Tribunal’s Award.
This section authorises the Central Government to make schemes to implement decision of Tribunal
constituted for adjudication of disputes. Hence, the Central Government as per the directions of the
honourable Court has submitted the Cauvery draft Water Management Scheme.
Article 262 of the Indian Constitution provides for adjudication of disputes relating to distribution and
control of waters of inter-state rivers or river valleys. Thus, the Parliament under Article 262(1) legislated the
Inter-State River Water Dispute Act, 1956 (ISWDA).

FOCUS | June 2018 | RAU’S IAS 29

Download from:- www.UPSCPDF.com


https://t.me/UPSC_PDF www.UPSCPDF.com https://t.me/UPSC_PDF

Constitution, Polity and Governance

NEW DRAFT SCHEME

x The Central Government has submitted a Draft Scheme to the Supreme Court to implement the
final decision of the Cauvery Tribunal on water sharing between the riparian states of Karnataka,
Kerala, Tamil Nadu and Puducherry.
x The draft scheme has proposed to create nine-member Authority to monitor the
implementation of Cauvery Tribunal’s final award.
x The said Authority shall be a 2-tier structure comprising of-
What? 9 An Apex Body having power to ensure compliance with the final award, and
9 Cauveri Water Regulation Committee (CWRC) that will monitor the field situation and water flows
x The draft scheme has made Central Government the final arbiter in disputes between states
regarding water sharing. However, every state has a right to approach a Court of law if they are not
satisfied with the Centre’s decision.
Note* The concept of having an Authority and a Water regulation Committee have been borrowed
from the recommendations of the Cauvery Water Disputes Tribunal (CWDT) in 2007.

x The authority will have five nominees of the Centre and four of the parties to the dispute and
will supervise the operation of reservoirs and regulate water release with the assistance of CWRC.
x The CWRC is tasked with collecting details about inflows and storage positions of reservoirs

Powers & across the states.

Functions x Powers of Authority would extend to apportionment, regulation and control of Cauvery waters,
of the supervision of operations of reservoirs and regulation of water releases.
Authority x Authority’s decision shall be final and binding.
x However, if the authority finds that any one of the States is not cooperative, then it can seek the
Centre’s help and the Centre’s decision will be final and binding.
x The authority would also decide on the “distress formula” in water-sharing among riparian states.

The Centre can help the fighting states in two ways:

Help from 9 By soliciting and suggesting suitable ways for speedy resolution of the dispute.
central 9 By intervening on behalf of states.
government Note* Intervention by the Centre can at times turn controversial given the multi-party set up of our
democracy.

DIFFERING VIEWPOINTS

Cauvery Management Board Authority


(as proposed by Tribunal) (As suggested by the Central Government )

Tribunal favoured the chairperson x The chairperson could be a senior and eminent engineer with wide
being an irrigation engineer with not experience in water resources management or
less than 20 years of experience in x An officer in the rank of Secretary or Additional Secretary to the Union
water resources management. government.
x Thus, the centre under the draft scheme has more discretion in terms of
appointment of a Chairperson of the Authority.

Note: Thus, for the proposed authority to function independently, it is important that they remain autonomous and
not under the control of central government. It is equally important for all riparian states to agree unanimously on the
proposed draft scheme for its effective implementation.

FOCUS | June 2018 | RAU’S IAS 30

Download from:- www.UPSCPDF.com


https://t.me/UPSC_PDF www.UPSCPDF.com https://t.me/UPSC_PDF

Constitution, Polity and Governance

The Supreme Court has held that law on prevention of domestic


SC ON DOMESTIC violence extends to all man-woman relationship including

VIOLENCE LAW FOR divorced women as the term “domestic violence” cannot be
restrained to marital relation alone. The case of domestic
DIVORCED WOMEN violence can continue even after divorce as the husband could
still resort to violence. The law in focus is Protection of Women
#Judgment #Act #Social Issue from Domestic Violence Act, 2005.

PROTECTION OF WOMEN FROM DOMESTIC VIOLENCE ACT, 2005

The Act provides for more effective protection of the rights of women guaranteed under the Constitution who are
victims of violence of any kind occurring within the family. The key words associated with the Act are:

Aggrieved Means any woman who is, or has been, in a domestic relationship with the respondent and who
Person alleges to have been subjected to any act of domestic violence by the respondent.

Means a relationship between two persons who live or have, at any point of time, lived together in a
Domestic
shared household, when they are related by consanguinity, marriage, or through a relationship in the
Relationship
nature of marriage, adoption or are family members living together as a joint family.

Means a household where the person aggrieved lives or at any stage has lived in a domestic
Shared
relationship either singly or along with the respondent and includes such a house whether owned or
Household
tenanted either singly or jointly.

a) Harms or injures or endangers the health, safety, life, limb or well-being, whether mental or
physical, of the aggrieved person or tends to do so and includes causing physical abuse, sexual
abuse, verbal and emotional abuse and economic abuse; or
Domestic
b) Harasses, harms, injures or endangers the aggrieved person with a view to coerce her or any other
Violence
person related to her to meet any unlawful demand for any dowry or other property or valuable
includes:
security; or
c) Has the effect of threatening the aggrieved person or any person related to her by any conduct as
mentioned above.

The State Government shall appoint such number of Protection Officers in each district as it may
Protection
consider necessary and shall also notify the area or areas within which a Protection Officer shall
Officers
exercise the powers and perform the duties conferred on him.

Shelter A place which provides shelter to the aggrieved person and the Protection Officer shall help such
Homes aggrieved person to find shelter to the aggrieved person in the shelter home.

DIFFERENT CONNOTATION OF THE WORD “DOMESTIC RELATIONSHIP AND DOMESTIC VIOLENCE” AS


EXPLAINED BY SUPREME COURT

Domestic Includes any relationship between two persons who either live at the present moment or have at any
Relationship point of time in the past lived together in a shared household.

Domestic violence can occur or continue to occur even after divorce. Thus the meaning of the word
domestic violence shall not be constrained only;
Domestic x Absence of subsisting domestic relationship in no manner prevents the court from granting certain
violence reliefs specified under the Act.
x The term ‘domestic violence’ cannot be restrained to marital relations alone as domestic
relationship includes consanguinity, marriage, a relationship in the nature of marriage, adoption or

FOCUS | June 2018 | RAU’S IAS 31

Download from:- www.UPSCPDF.com


https://t.me/UPSC_PDF www.UPSCPDF.com https://t.me/UPSC_PDF

Constitution, Polity and Governance

as family members living together as a joint family.


x If the aggrieved person had been in domestic relationship at any point of time even prior to coming
into the force of the Act and was subjected to domestic violence, the person is entitled to invoke
the remedial measures provided under the Act.”

LEGAL PROVISIONS

x Article 15(3) - Nothing in this article shall prevent the State from making any special provision for
women and children.
x Article 39A - The State shall secure that the operation of the legal system promotes justice, on a
Constitution basis of equal opportunity, and shall, in particular, provide free legal aid, by suitable legislation or
of India schemes or in any other way, to ensure that opportunities for securing justice are not denied to
any citizen by reason of economic or other disabilities.
x Article 42 - The State shall make provision for securing just and humane conditions of work and
for maternity relief.

x Section 498A of Indian Penal Code provides for cruelty by husband or relatives of husband
x 498A- Husband or relative of husband of a woman subjecting her to cruelty.—Whoever, being the
husband or the relative of the husband of a woman, subjects such woman to cruelty shall be
punished with imprisonment for a term which may extend to three years and shall also be liable to
Indian Penal fine.
Code x The section explains Cruelty. It means -
9 Any willful conduct which is likely to drive the woman to commit suicide or to cause grave injury or
danger to life, limb or health (whether mental or physical) of the woman; or
9 Harassment of the woman coercing her or any person related to her to meet any unlawful demand
for any property or valuable security

PARLIAMENTARY COMMITTEE REPORTS AS EVIDENCE


#Judgment #Committee #Constitution
A five-judge Constitution Bench of the Supreme Court in the case of Kalpana Mehta & Others v Union of India
& Others has declared that reports of Parliamentary Standing Committees can be referred for the purpose of
interpretation and better clarity and is admissible as evidence. The bench also held that looking into the
reports of Parliamentary Privilege for clarity and reference does not constitute a Breach of Parliamentary
Privilege.
Kalpana Mehta had filed public interest litigation (PIL) where she had challenged the use of two vaccines
Gardasil and Cervarix on tribal women of Andhra Pradesh and Gujarat. In this, she quoted a Report of
Parliamentary Standing Committee where the committee had concluded that the use of Human Papilloma
Virus (HPV) vaccine by state government of Gujarat, Andhra Pradesh in collaboration with PATH
International, constituted a serious breach of medical ethics and a clear cut violation of human rights.

Need for a x Whether the Supreme Court under Article 32 or Article 136 can refer to and place reliance upon the
constitution report of the Parliamentary Standing Committee; and
bench x Whether looking into such reports by the Court amounts to breach of Parliamentary Privileges?

FOCUS | June 2018 | RAU’S IAS 32

Download from:- www.UPSCPDF.com


https://t.me/UPSC_PDF www.UPSCPDF.com https://t.me/UPSC_PDF

Constitution, Polity and Governance

x How does the constitution conceive the delicate balance between Article 105, 121 and 122?
x Whether reports of Parliamentary Committees can be accepted as evidence under Section 57 (4) of
Indian Evidence Act, 1872?

CONSTITUTIONAL PROVISIONS INTERPRETED

Article 105 Powers, privileges, etc., of the Houses of Parliament and of the members and committees

Article 121 Restriction on discussion in Parliament

Article 122 Courts not to inquire into proceedings of Parliament

Article 136 Special Leave to Appeal by the Supreme Court

PROVISIONS OF INDIAN EVIDENCE ACT INTERPRETED

x Section 57 - Facts of which courts must take judicial notice


x Section 57(4) - The course of proceeding of Parliament of the United Kingdom, of the Constituent
Assembly of India, of Parliament and of the legislatures established under any laws for the time
Indian being in force in a Province or in the States.
Evidence x Section 74 – Public Documents
Act, 1872 1. documents forming the acts or records of the acts –– (i) of the sovereign authority, (ii) of official
bodies and tribunals, and (iii) of public officers, legislative, judicial and executive, of any part of
India or of the Commonwealth, or of a foreign country;
2. (2) public records kept in any State of private documents.

RECENT JUDGEMENT

x According to Article 105(2) of Constitution of India, no Member of Parliament can be held liable
On for anything said by him in Parliament or in any committee. The reports submitted by Members
Parliamentary
of Parliament are also fully covered by protection extended under Article 105(2) of the
Privileges
Constitution of India.

Reliance on x Such Parliamentary Reports can be relied upon in proceedings under Article 32 or 136.
Reports of x Help can be taken from reports of Parliamentary Standing Committee for the purpose of
Parliamentary interpretation of a statutory provision for sake of clarity wherever it is required.
Committee x Reports of the Committee can be taken note of as existence of a historical fact.

Breach of Once the report of a Parliamentary Committee has been published, reference to it in the course of
Privilege judicial proceedings will not constitute a breach of parliamentary privilege.

Right to know The Court held that the general public are interested in knowing about the parliamentary
about reports proceedings including parliamentary reports which are steps towards the governance of the
of country. The right to know about the reports only arises when they have been published for use of
Committees the public in general.

x Fair comments on the reports by citizens are fully protected under Article 19(1)(a).
Reports in x However, outright personal attacks on individual member of Parliament or members of House
Public Domain including use of offensive language tarnishing their image would amount to breach of
Parliamentary Privilege which is not allowed.

x Judicial notice can be taken of the Parliamentary Standing Committee report under Section 57(4)
Admissible as
of the Evidence Act and it is admissible as public document under Section 74 of the Evidence
Evidence
Act.

FOCUS | June 2018 | RAU’S IAS 33

Download from:- www.UPSCPDF.com


https://t.me/UPSC_PDF www.UPSCPDF.com https://t.me/UPSC_PDF

Constitution, Polity and Governance

x There is no requirement of any permission of Speaker of Lok Sabha for producing such
documents as evidence in Courts.

Can the x The Parliamentary Reports carry Parliamentary Privilege and its contents cannot be questioned
contents of outside the Parliament.
the reports be x However, the Courts can accept Parliamentary Reports on record before it provided they do not
questioned? question its content.

The ‘Powers, Privileges and Immunities of Parliament and its Members’ as envisaged in Article 105 of the
Constitution leave room for a large number of un-codified and un-enumerated privileges to continue. Assess
the reasons for the absence of legal codification of the ‘parliamentary privileges’. How can this problem be addressed?
UPSC GS PAPER – II (MAINS – 2014)

NEW BILL: The Union Cabinet has approved the Commercial Courts,
Commercial Division and Commercial Appellate Division of High
CHANGES SOUGHT Courts (Amendment) Bill, 2018 for introduction in the Parliament.
The Bill amends the Commercial Courts, Commercial Division and
IN COMMERCIAL Commercial Appellate Division of High Courts Act, 2015.
DIVISION BENCH The Bill aims to reduce the pecuniary limit for commercial
disputes to be heard by such specialized Courts and also allows
#Amendment #Act for mandatory mediation as a process to reconcile differences
#Governance between parties to such disputes.

NEED

x With the rapid economic development there has been considerable increase in commercial activities and
consequent steep rise in number of commercial disputes at domestic and international level.
x Increase of Foreign Direct Investment (FDI) and overseas commercial transactions have further contributed to a
significant increase of commercial disputes.
x Dispute Resolution is one of the important criteria for determining ease of doing business as prepared by
World Bank.
x These suggested amendments will help in faster disposal of cases and will also help in reducing pendency of cases
clogged in Courts. It will eventually help in bettering the Ease of Doing Business index of India.
x A high ease of doing business ranking reflects a better regulatory environment which is more conducive to the
starting and operation of business.
x Thus, the Commercial Courts, Commercial Division and Commercial Appellate Division of High Courts Act, 2015 was
enacted to resolve disputes by specialized courts and in a faster manner.
x Let us understand the suggested changes as proposed through the Amendment Bill of 2018.

x Prior to amendment, commercial courts and commercial divisions in High courts could decide
commercial disputes having a specified value of at least 1 crore rupees.
Pecuniary
x The amendment proposes to reduce this limit to three lakh rupees.
Limit
x This would bring down the time taken in resolution of commercial disputes of lesser value and
will help in improving India's ranking in the Ease of Doing Business.

Commercial x As per 2015 Act, no commercial court could have been constituted for the territory over which the
Courts at High Court had ordinary original civil jurisdiction.(Ordinary Original Civil Jurisdiction means when the
district level Court has the authority to try fresh cases before itself from the beginning.)

FOCUS | June 2018 | RAU’S IAS 34

Download from:- www.UPSCPDF.com


https://t.me/UPSC_PDF www.UPSCPDF.com https://t.me/UPSC_PDF

Constitution, Polity and Governance

x The amendment removes the bar and provides for establishment of Commercial Courts at
district Judge level for the territories over which respective High Courts have ordinary original
civil jurisdiction. i.e. in the cities of Chennai, Delhi, Kolkata, Mumbai and State of Himachal
Pradesh.
x Prior to amendment, these cities could not have Commercial Courts.

Designation of Commercial In areas where High Courts do not have ordinary original civil jurisdiction, state
Appellate Courts governments, may notify commercial appellate courts at the district judge level.

x A provision for mandatory mediation has been provided in those cases where no urgent relief
(such as an injunction) is being sought by the parties to the dispute.
x Mediation provides an opportunity to the parties to resolve the commercial disputes outside the
ambit of the courts through the authorities such as the National and District Legal Services
Pre- Authority constituted under the Legal Services Authorities Act, 1987.
Mediation x This will also help in reinforcing investor's confidence in the resolution of commercial disputes.
x The mediation process is required to be completed within a period of three months (which may
be extended by another two months).
x A signed settlement between the parties will have the same effect as an Arbitral Award under
the Arbitration and Conciliation Act, 1996 and shall be enforceable like an order of court of law.

FOCUS | June 2018 | RAU’S IAS 35

Download from:- www.UPSCPDF.com


https://t.me/UPSC_PDF www.UPSCPDF.com https://t.me/UPSC_PDF

ECONOMIC DEVELOPMENT
# GS Paper (Prelims) & GS Paper III (Main)

SERVICES SECTOR IN INDIA: OPPORTUNTIES AND CHALLENGES


#Sector of Economy
The services sector represents a vibrant and expanding component of the global economy. It is the key
driver of India’s economic growth since 1991. Today, the sector dominates in terms of employment, foreign
investment, export, value addition, productivity and innovation. According to an estimate, the sector
contributes around 54.0 percent of India’s Gross Value Added in 2017-18 and employed 28.6 percent of the
total labour force.
The services sector covers a wide variety of activities such as trade, hotel and restaurants, transport, storage
and communication, financing, insurance, real estate, business services, community, social and personal
services, and services associated with construction.

OPPORTUNITY

The large young talent pool, a billion mobile phone users; 500 million citizens on the Internet; and a
Expanding
determined push for financial inclusion using technology as well as for digital payments have
Market
collectively provided an opportunity for expanding service sector in the country.

x The pace of technology is intensifying the contribution of services in other sectors – including
agriculture, infrastructure and manufacturing.
x Digital economy and Fourth Industrial Revolution are further enhancing the synergy of technology
and services. This will further create an opportunity for the services sector.
Technology
Fourth Industrial Revolution is characterized by a fusion of technologies. It is marked by emerging
technology breakthroughs in a number of fields, including robotics, artificial intelligence, blockchain,
nanotechnology, quantum computing, biotechnology, The Internet of Things, 3D printing and
autonomous vehicles.

India’s global services exports stood at 3.4 percent in 2016 and it has set the target of a 4.2% of global
Export
services by 2022 under foreign trade policy. This will create ample of scope to grow further for the
target
services sector.

x With the rapid expansion of its economy and impressive GDP growth rate, India represents an
Expanding attractive opportunity for global investors and businesses. The services sector boom is a key driver.
economy x By 2025, India hopes to double the size of its GDP to US$ 5 trillion. Of this, the services sector could
be a US$ 3 trillion contributor.

CHALLENGES

Hindrances due x Recent policies such as Demonetisation and implementation of GST have had short-term
to Recent policy impact on Indian economy as a whole and services sector was not an exception.
measures x Lack of project completion, costlier services due to the higher rate under GST, complicate GST
registration rules had affected the growth rate of Services sector in short-term.

Domestic x Restricted Foreign direct investment in some of the services sector such as retail, insurance and
policies, such banking.

FOCUS | June 2018 | RAU’S IAS 36

Download from:- www.UPSCPDF.com


https://t.me/UPSC_PDF www.UPSCPDF.com https://t.me/UPSC_PDF

Economic Development

as: x Higher tax in some of the sectors such as entertainment and aviation sector especially on
imported aircraft parts.

x The strict domestic regulations such as licensing requirement, licensing procedure,


Domestic qualification procedure, technical standards in service sectors are hampering the growth of
regulation the sector and exports.
x These are also responsible for India’s poor performance in Ease of Doing Business index.

x There are many market access barriers in India’s trading partner countries. Some of them for
major services include Visa issues in different countries for Professionals and the Buy American
Market access provisions in the US.
barrier x Another kind of market barrier can be subsidies provided by the partner countries to its
domestic industry affecting services export from India.
x Imposition of higher tariff by the trading partner is another kind of Market Access barrier.

Free trade agreement India’s FTAs, such as India-Asean FTA, have benefitted its trading partners more than India.

Employment in India’s services sector comprises a high share in income and relatively low share in employment,
Services while in China, the shares of both services income and services employment are relatively low.

EFFORTS BY GOVERNMENT

Policy supports in the forms of Start-up India and the Mudra Yojana have seeded a culture of
Entrepreneurship
entrepreneurship, largely in services.

Promotion of Digital India and digital payment have unleashed creative and commercial
Digital India
energies in the services sector.

The Government has eased foreign investment regulations in several sectors, including in
Easing FDI
services such as insurance, retail, aviation and banking.

The introduction of the Goods and Services Tax has tried to convert India into a single market,
GST with both goods and services placed on the same tax platform. This in itself has been tried to
impart a new momentum to the economy.

The project will enhance the competitiveness of India's service sectors and hence, will promote
12 Champion
the Indian services domain and enlarge and deepen India’s engagement with the global services
Service Sector
sector.

x Participation of state governments, as part of Business Reforms Action Plan, has been
ensured.
x Starting a business and obtaining permits, as well as trading across borders, has become
Ease of Doing
easier.
Business
x The Insolvency and Bankruptcy Code has made it simpler for the exit of businesses.
x And the national Intellectual Property Rights policy aims at strengthening the IPR regime in
line with global best practices.

Under the Mid-Term Review of Foreign Trade Policy (2015-20), the Central Government increased incentives
SEIS
provided under Services Exports from India Scheme (SEIS) by two percent.

WAY FORWARD

x Further liberalise the FDI across the services sector.


x Address the issue of multiple levies & duties in various service sectors.
x Rationalise the high taxes and tariffs on various service sectors.

FOCUS | June 2018 | RAU’S IAS 37

Download from:- www.UPSCPDF.com


https://t.me/UPSC_PDF www.UPSCPDF.com https://t.me/UPSC_PDF

Economic Development

x The government needs to enhance the visibility of India in services through trade fairs, buyers-sellers meets and
setting up convention centres.
x Modernise port infrastructure on a priority basis.
x Address the restrictions on inter-state movement of goods.
x Strengthen competition policy for services and regulatory body for services other than banking, insurance, telecom
and ports.

12 CHAMPION In order to promote their development and realize their


potential, the Union cabinet has approved the proposal of
SERVICES SECTORS Ministry of Commerce to give focused attention to 12 identified

# Services Sector Champion Services Sectors.

KEY FACTS

6. Audio Visual Services


1. Information Technology & Information
7. Legal Services
Technology enabled Services (IT & ITeS)
8. Communication Services
Sectors 2. Tourism and Hospitality Services
9. Construction and Related Engineering Services
included 3. Medical Value Travel
10. Environmental Services
4. Transport and Logistics Services
11. Financial Services
5. Accounting and Finance Services
12. Education Services

A dedicated fund of Rs. 5000 crores have been proposed to be established to support initiatives for
Funds
sectoral Action Plans of the Champion Sectors.

x The project will enhance the competitiveness of India's service sectors through the implementation
of focused and monitored Action Plans, thereby promoting GDP growth, creating more jobs and
Benefits promoting exports to global markets.
x It will promote the Indian services domain and enlarge and deepen India’s engagement with the
global services sector.

In order to promote biofuels in the country, the Union Cabinet


NATIONAL POLICY has approved National Policy on Biofuels – 2018. The policy
expands the scope of raw material for ethanol production,
ON BIOFUELS, 2018 thereby aims to reduce dependency on crude oil and at the
# Renewal Energy same time enhance the income of farmers, besides promoting a
cleaner environment and providing health benefits.

SALIENT FEATURES

x The Policy categorises biofuels as "Basic Biofuels" and “Advanced Biofuels”.


Category of
x Basic Biofuels comprise First Generation (1G) bioethanol and biodiesel.
Biofuels
x Advanced Biofuels comprise- Second Generation (2G) ethanol, Municipal Solid Waste (MSW) to

FOCUS | June 2018 | RAU’S IAS 38

Download from:- www.UPSCPDF.com


https://t.me/UPSC_PDF www.UPSCPDF.com https://t.me/UPSC_PDF

Economic Development

drop-in fuels + Third Generation (3G) biofuels, bio-CNG etc.


x This category has been done to enable the extension of appropriate financial and fiscal incentives
under each category.
Drop-in fuels are those renewable fuels which can be blended with petroleum products, such as
gasoline, and utilized in the current infrastructure of pumps, pipelines and other existing equipment.

The Policy expands the scope of raw material for ethanol production by allowing use of
9 Sugarcane Juice,
9 Sugar containing materials like Sugar Beet,
9 Sweet Sorghum, Starch containing materials like Corn,
Scope of raw
9 Cassava,
material
9 Damaged food grains like wheat,
9 broken rice,
9 Rotten Potatoes,
9 unfit for human consumption for ethanol production.

x The policy allows the use of surplus food grains for production of ethanol for blending with
Usage of
petrol with the approval of National Biofuel Coordination Committee.
Surplus food
x This measure will benefit farmers who can now get appropriate price for their produce during the
production
surplus production phase.

With a thrust on Advanced Biofuels, the policy indicates a viability gap funding scheme for 2G ethanol
Viability gap
Bio-refineries of Rs.5000 crore in 6 years in addition to additional tax incentives, higher purchase price
funding
as compared to 1G biofuels.

Supply chain The Policy encourages setting up of supply chain mechanisms for biodiesel production from non-
mechanisms edible oilseeds, Used Cooking Oil, short gestation crops.

Roles and Roles and responsibilities of all the concerned Ministries/Departments with respect to biofuels have
responsibilities been captured in the policy document to synergise efforts.

BENEFITS

The policy will entail more supply of ethanol which will reduce the pressure for importing crude oil
Saving of forex
resulting in saving of forex in the country.

x The policy promotes the ethanol production, usage of which helps in controlling emission of CO2
Cleaner in the environment.
Environment x Further, by reducing crop burning and conversion of agricultural residues/wastes to biofuels
there will be a further reduction in Green House Gas emissions.

x Prolonged reuse of Cooking Oil for preparing food, particularly in deep-frying is a potential health
Health hazard and can lead to many diseases.
benefits x Used Cooking Oil is a potential feedstock for biodiesel and its use for making biodiesel will
prevent diversion of used cooking oil in the food industry.

Infrastructural
investment in Addition of 2G bio refineries across the country will spur infrastructural investment in the rural areas.
Rural Areas

Employment One 100klpd 2G bio refinery can contribute 1200 jobs in plant operations, Village Level Entrepreneurs
Generation and Supply Chain Management.

FOCUS | June 2018 | RAU’S IAS 39

Download from:- www.UPSCPDF.com


https://t.me/UPSC_PDF www.UPSCPDF.com https://t.me/UPSC_PDF

Economic Development

x By adopting 2G technologies, agricultural residues/waste which otherwise are burnt by the


farmers can be converted to ethanol and can fetch a price for these wastes if a market is
Additional
developed for the same.
Income to
x Also, farmers are at a risk of not getting appropriate price for their produce during the surplus
Farmers
production phase. Thus, conversion of surplus grains and agricultural biomass can help in price
stabilization.

Address
The policy primarily tries to address supply-side issues that have discouraged the production of
supply-side
biofuels within the country.
constraints

Availability of more raw material to be used as inputs to produce ethanol will reduce the cost of
Cost reduction producing biofuels and improve affordability for consumers, particularly during times when oil prices
reach discomforting levels.

Biofuels in India are of strategic importance as it augers well with the ongoing initiatives of the
Strategic Government such as Make in India, Swachh Bharat Abhiyan, Skill Development and offers great
importance opportunity to integrate with the ambitious targets of doubling of Farmers Income, Import Reduction,
Employment Generation, Waste to Wealth Creation.

Studies revealed that solar and winds are almost complementary to


NATIONAL WIND- each other and hybridization of two technologies would help in

SOLAR HYBRID optimal utilization of the infrastructure including land and


transmission system. In light of this view, Ministry of New &
POLICY Renewable Energy has issued National Wind-solar Hybrid Policy.
The policy provides a framework for the promotion of large grid-
# Renewal Energy connected wind-solar PV hybrid system.

WHAT IS WIND-SOLAR HYBRID SYSTEM?

x Under the category of wind-solar hybrid power plants, Wind Turbine Generators (WTGs) and Solar PV systems
are configured to operate at the same point of grid connection.
x A wind-solar plant will be recognized as hybrid plant if the rated power capacity of one resource is at least 25% of
the rated power capacity of other resource.

NEED FOR THE POLICY

x India has set an ambitious target of reaching 175 GW of installed capacity from renewable energy sources by the
year 2022, which includes 100 GW of solar and 60 GW of wind power capacity.
x While at the end of 2017-18 the total renewable power installed capacity in the country was almost 70 GW. Hence,
the county needs to find avenues to accelerate its renewable energy production.
x The existing wind farms have the scope of adding solar PV capacity and similarly, there may be wind potential in
the vicinity of existing solar PV plant.
x Suitable policy interventions are, therefore, required not only for new wind-solar hybrid plants but also for
encouraging hybridization of existing wind and solar plants.

KEY FEATURES OF THE POLICY

Aims and x To provide a framework for the promotion of large grid-connected wind-solar PV hybrid system

FOCUS | June 2018 | RAU’S IAS 40

Download from:- www.UPSCPDF.com


https://t.me/UPSC_PDF www.UPSCPDF.com https://t.me/UPSC_PDF

Economic Development

Objectives for
9 optimal and efficient utilization of transmission infrastructure and land,
9 reducing the variability in renewable power generation, and
9 achieving better grid stability.
x To encourage new technologies, methods and way-outs involving the combined operation of
wind and solar PV plants.

x The policy provides for the integration of both wind and solar energy sources.
x It provides for the procurement of power from a hybrid project on tariff-based competitive
Implementation
bids.
strategy
x It also permits the use of battery storage in the hybrid project for optimising the output and
further reduce the variability.

x The Central Electricity Authority and CERC shall formulate necessary standards and regulations
including;
9 metering methodology and standards,
Regulatory 9 forecasting and scheduling regulations,
environment
9 Renewable Energy Certificate (REC) mechanism,
9 grant of connectivity and
9 sharing of transmission lines, etc. for wind-solar hybrid systems.

x All fiscal and financial incentives available to wind and solar power projects may also be made
available to hybrid projects.
Incentives
x Low-cost financing for hybrid projects may be made available through IREDA and other
financial institutions including multilateral banks.

x The government will support the technology development projects in the field of wind-solar
Research &
hybrid systems.
Development
x Besides, support will be provided for the development of standards for hybrid systems.

BENEFITS

Cost savings x The hybrid project will gain in terms of cost saving on land acquisition.

x The policy also allows the existing wind farms to expand their portfolio and add solar PV capacity
Expand and vice versa provided they have 25 percent installed generation capacity of the other source.
capacity x This will help the sector players as they can lower costs and raise generation at their existing
project sites.

x Hybrid projects are likely to be competitive in tariffs compared to individual wind or solar energy
projects.
Competition
x This will happen due to the benefits associated with hybrid projects — mainly in respect of
lower capital cost, optimisation of transmission infrastructure and higher generation expected.

High The generation profile will also be relatively higher than standalone wind or solar project due to lower
Generation variability in generation profile.

Address the
Lower variability (fluctuation) in generation profile would partially address the concerns of distribution
concern of
utilities over the grid stability arising due to the intermittent nature of wind or solar.
distribution

FOCUS | June 2018 | RAU’S IAS 41

Download from:- www.UPSCPDF.com


https://t.me/UPSC_PDF www.UPSCPDF.com https://t.me/UPSC_PDF

Economic Development

WAY FORWARD

x Policy alone may not help in generating benefits. The Central Electricity Authority and the Central Electricity
Regulatory Commission will have to come out with norms that can allow the sale of hybridised power generated
from existing projects.
x There needs to be clarity on how Power Purchase Agreements will be reworked in case a project decides to add
another generation source.
x The project developer will definitely want to gain better margins by lowering generation costs and increasing
volumes. It will be important if the renewable purchase obligation is extended for these projects.

Î RELATED INFORMATION: POWER PURCHASE AGREEMENTS

x PPA (Power Purchase Agreement) is a contract between the purchaser of electricity and electricity generator
setting out the terms and price for supplying electricity. PPAs are signed based on these pre-determined prices for a
number of years.
x Earlier, tariffs were fixed by state electricity regulatory commissions based on which PPAs were made with power
generators. In the case of renewable energy, state electricity regulatory commissions set the Feed-in Tariffs for the
purchase of electricity from these sources.
x The recent auctions for solar power procurement have led to the discovery of very low tariffs. This is, though a
welcome news, possibly contributed to some demands for the renegotiation of the already signed PPAs

NATIONAL DIGITAL
The Department of Technology (DoT) has issued the draft
COMMUNICATIONS ‘National Digital Communications Policy (NDCP) 2018. The
POLICY (NDGT) 2018- draft paper outlines three main action paths, viz. ‘Connect
India’, ‘Propel India’ and ‘Secure India’ in its approach to
DRAFT addressing issues in the digital arena.

# Telecommunication

Connect India looks at broadband as a tool for socio-economic development, while Propel India aims at harnessing
the power of new digital technologies like 5G, artificial intelligence (AI) and Big Data. And finally, Secure India seeks to
ensure the sovereignty, safety and security of digital communication in the country.

OBJECTIVES OF THE POLICY

x Increasing digital communication sector’s GDP contribution to 8% from current 6%;


x To take India to the top 50 nations in International Telecommunication Unit’s (ITU) ICT Development Index from 134
in 2017;
x Enhancing India’s contribution in the global digital value chain;
x Ensuring digital sovereignty.

KEY FEATURES OF THE POLICY

x The new policy aims to make sure that every citizen has access to broadband running at least
Broadband
50Mbps by 2022.
for all
x It seeks to ensure connectivity to unconnected areas through channelising the Universal Service

FOCUS | June 2018 | RAU’S IAS 42

Download from:- www.UPSCPDF.com


https://t.me/UPSC_PDF www.UPSCPDF.com https://t.me/UPSC_PDF

Economic Development

Obligation Fund (USOF).

x The policy seeks to implement collaborative models to improve infrastructure sharing between
Fibre first
public, local and private entities.
initiative
x This will increase access to fibre optic cables in municipalities, rural areas and national highways.

x A central authority, called the National Fibre Authority, will be put in place to handle utility
National between new initiatives being launched under the NDCP.
Digital Grid x The authority would coordinate access, standardisation of costs and timelines with respect to the
National Digital Grid between the centre, states and local bodies.

Employment creation The draft paper has stated creation of 4 million additional jobs in the Digital arena by 2022.

x The NDCP aims to attract $100 billion foreign investment for the telecom sector by 2022.
Investment x It aims to catalyse such investment by providing an impetus to research and development, start-
ups and local manufacturing.

Re-skilling The policy envisages re-training and re-skilling 1 million people with ‘new age skills’

Expansion of The draft envisages expanding the ‘Internet of Things’ network by simplifying the licensing and
IoT Network regulatory framework.

x The policy states review of licenses fee and spectrum user charges.
Spectrum
x Further, it mentions the development of a fair, flexible, simple and transparent method system for
allocation
spectrum allocation.

Next
x The policy promotes adoption of next-generation access technologies by the service providers.
Generation
x This will ensure cost optimisation, service agility and new revenue streams for service providers.
Technologies

Broadband x The draft aims at creating Broadband Readiness Index for States/UTs.
Readiness x This will promote standard measurement mechanism which, in turn, will help attract investments
Index and address challenges from the rest of the world.

x The aim of the policy is to establish a strong, flexible and robust data protection regime so that
each citizen and enterprise can operate with autonomy and be given the freedom of choice.
Secure India
x For this, the NDCP suggests a Telecom Testing and Security Certification (TTSC) to enforce
security standards that are at par will global standards with consideration for local requirements.

The Cabinet Committee on Economic Affairs has approved


PRADHAN MANTRI the proposal for renaming and restructuring of Multi-
Sectoral Development Programme (MsDP) as Pradhan Mantri
JAN VIKAS Jan Vikas Karyakram (PMJVK). It has also approved the
KARYAKARAM # Scheme continuation of the programme during the remaining period
of the 14th Finance Commission (2015-2020).

ABOUT PMJVK

The Programme aims to address the development deficits in the identified minority concentration
Aim
areas.

Identification x The identification of minority concentration areas has been done on the basis of presence of a

FOCUS | June 2018 | RAU’S IAS 43

Download from:- www.UPSCPDF.com


https://t.me/UPSC_PDF www.UPSCPDF.com https://t.me/UPSC_PDF

Economic Development

substantial population of notified minority communities based on Census, 2011.


x The criteria for identification of Minority Concentration Towns (MCTs) and Clusters of Villages
have been rationalized:
9 Earlier only those towns which were found backwards in terms of both in Basic Amenities and
Socio-economic parameters were taken up as MCTs. Now, the towns which are found
backwards in either or both of the criteria have been taken up as MCT.
9 Earlier only those clusters of villages which were having at least 50% population of minority
community were taken. Now the population criteria have been lowered to 25%.

x Unit area of implementation has been broadened by including minority concentration districts
headquarters in addition to the minority concentration blocks, minority concentration towns and
the cluster of minority concentration villages.
Geographical x As compared to MsDP, the PMJVK would now cover five more States/UTs namely Himachal
coverage Pradesh, Tamil Nadu, Nagaland, Goa and Pondicherry. All other States/UTs earlier covered under
MsDP have also been included under PMJVK.
x Overall, the PMJVK will cover 308 districts of the country compared to only 196 districts under the
MsDP.

x 80% of the resources under the PMJVK would be earmarked for projects related to education,
Finance health and skill development.
Mechanism x 33 to 40% of resources under the PMJVK would be specifically allocated for women-centric
projects.

All the implementing agencies are brought under Public Finance Management System (PFMS) for
Monitoring
better monitoring.

IMPACT

x The restructured programme would provide better socio-economic infrastructure facilities to the minority
communities particularly in the field of education, health & skill development
x The flexibility introduced in the programme will result in speedier implementation leading to the greater
inclusiveness of the minority communities.
x The geographical coverage has been broadened under the scheme leading to substantial increase in the number
of beneficiaries.

Î RELATED INFORMATION: BACKGROUND OF MSDP

x MsDP has been identified as one of the “Core of the Core Schemes”.

x Important to note here is that there at the central level there are two types of schemes-core schemes and optional
schemes.

x Core Schemes require compulsory participation by States, whereas, amongst the Optional Schemes, States could
choose some or all of them. Among the core scheme, MGNREGA + Scheme for Social Inclusion are considered as
“the Core of the Core” and hence, are the first charge on funds available for the National Development Agenda.

x The programme was launched in the year 2008-09 in 90 identified Minority Concentration Districts (MCDs) having at
least 25% minority population and below the national average with respect to one or both of the backwardness
parameters with the objective of developing assets for socio-economic and basic amenities.

FOCUS | June 2018 | RAU’S IAS 44

Download from:- www.UPSCPDF.com


https://t.me/UPSC_PDF www.UPSCPDF.com https://t.me/UPSC_PDF

Economic Development

The Cabinet Committee on Economic Affairs has approved


GREEN REVOLUTION- "Green Revolution – Krishonnati Yojana" in agriculture sector
beyond 12th Five Year Plan (2012-17) for the period from
KRISHONNATI 2017-18 to 2019-20. The Umbrella scheme comprises of 11

YOJANA Schemes/Missions. These schemes look to develop the


agriculture and allied sector in a holistic and scientific
# Scheme #Agriculture manner to increase the income of farmers by enhancing
production, productivity and better returns on produce.

FOLLOWING SCHEMES ARE PART OF THE UMBRELLA SCHEME- GREEN REVOLUTION-KRISHONNATI YOJANA

Schemes Objectives

Mission for Integrated x To promote holistic growth of horticulture crops


Development of x To enhance horticulture production
Horticulture crops x To improve nutritional security and income support to farm households

x To increase production of rice, wheat, pulses, coarse cereals and commercial crops,
National Food Security through area expansion, productivity enhancement
Mission including
x To restore soil fertility and productivity at the individual farm level and
National Mission on oil
x To enhance farm level economy (i.e. farm profits) to restore confidence amongst the
seeds and Oil Palm
farmers.
(NMOOP)
x To augment the availability of vegetable oils and to reduce the import of edible oils.

x To promote sustainable agriculture practices best suitable to the specific agro-


ecology focusing on
National Mission for
9 integrated farming,
Sustainable Agriculture
9 appropriate soil health management and
9 synergizing resource conservation technology.

x To strengthen the ongoing extension mechanism of state governments, local bodies


etc., achieving food and nutritional security and socio-economic empowerment of

Sub-mission on farmers

Agriculture Extension x To institutionalize programme planning and implementation mechanism;


(SMAE) x To forge effective linkages and synergy amongst various stake-holders;
x To support Human Resource Development interventions;
x To promote pervasive and innovative use of electronic / print media, inter-personal
communication and ICT tools, etc.

x To increase production of certified / quality seed


x To increase Seed Replacement Ratio/Rate
x To upgrade the quality of farm saved seeds
Sub-Mission on
x To strengthen the seed multiplication chain
Seeds and
Planting x To promote new technologies and methodologies in seed production, processing, testing etc.

Material (SMSP) x To strengthen and modernize infrastructure for seed production, storage, certification and
quality etc.
Seed Replacement Rate is the percentage of area sown out of total area of crop planted in the
season by using certified/quality seeds other than the farm saved seed.

FOCUS | June 2018 | RAU’S IAS 45

Download from:- www.UPSCPDF.com


https://t.me/UPSC_PDF www.UPSCPDF.com https://t.me/UPSC_PDF

Economic Development

x To increase the reach of farm mechanization to small and marginal farmers and to the
regions where availability of farm power is low
x To promote ‘Custom Hiring Centres’ to offset the adverse economies of scale arising due to
Sub-Mission on small landholding and high cost of individual ownership
Agricultural
x To create hubs for hi-tech and high value farm equipment,
Mechanisation
(SMAM) x To create awareness among stakeholders through demonstration and capacity building
activities, and
x To ensure performance testing and certification at designated testing centers located all over
the country.

x To minimize loss to quality and yield of agricultural crops from the ravages of insect pests,
Sub-Mission on diseases, weeds, nematodes, rodents, etc.
Plant Protection
x To shield our agricultural bio-security from the incursions and spread of alien species
and Plan
x To facilitate exports of Indian agricultural commodities to global markets, and
Quarantine
(SMPPQ) x To promote good agricultural practices, particularly with respect to plant protection strategies
and strategies.

x To undertake the agriculture census, study of the cost of cultivation of principal crops;
Integrated
Scheme on x To undertake research studies on agro-economic problems of the country;
Agriculture x To fund conferences/workshops and seminars involving eminent economists, agricultural
Census, scientists, experts and to bring out papers to conduct short term studies;
Economics and x To improve agricultural statistics methodology; and
Statistics
x To create a hierarchical information system on crop condition and crop production from
(ISACES)
sowing to harvest.

x To provide financial assistance for improving the economic conditions of cooperatives


Integrated
x To remove regional imbalances and to speed up - cooperative development in agricultural
Scheme on
marketing, processing, storage, computerization and weaker section programmes
Agricultural
Cooperation x To help cotton growers fetch remunerative price for their produce through value addition
besides ensuring supply of quality yarn at reasonable rates to the decentralized weavers.

x To develop agricultural marketing infrastructure;


x To promote innovative and latest technologies and competitive alternatives in agriculture
Integrated marketing infrastructure;
Scheme on
x To provide infrastructure facilities for grading, standardization and quality certification of
Agricultural
agricultural produce;
Marketing
(ISAM) x To establish a nationwide marketing information network;
x To integrate markets through a common online market platform to facilitate pan-India trade
in agricultural commodities, etc.

x To bring farmer centricity and service orientation to the programmes;


x To enhance reach and impact of extension services;
National e-
x To improve access of farmers to information and services throughout crop-cycle;
governance Plan
(NeGP-A) x To build upon, enhance and integrate the existing ICT initiatives of Centre and States;
x To enhance efficiency and effectiveness of programs through making available timely and
relevant information to the farmers for increasing their agriculture productivity.

FOCUS | June 2018 | RAU’S IAS 46

Download from:- www.UPSCPDF.com


https://t.me/UPSC_PDF www.UPSCPDF.com https://t.me/UPSC_PDF

Economic Development

Note: All these schemes / missions have been under implementation for varying duration during past few years. In
2017-18, it has been decided to club all these schemes / missions under one umbrella scheme 'Green Revolution -
Krishonnati Yojana'.

GREEN SKILL Realising that the Green Skills will go a long way in reaping the
demographic dividend in the country, the Ministry of Environment and
DEVELOPMENT Forest and Climate Change aims to impart green skills at a large scale

PROGRAMME in the country through its Green Skill Development Programme (GSDP).
The programme has been conceptualised and developed by the
(GSDP) Ministry of Environment, Forest and Climate Change (MoEF&CC) in
consultation with the National Skill Development Agency (NSDA).
# Skill Development

NEED FOR GSDP

Demand- There exists a demand-supply gap of skill sets at various levels in the environment/forest field in the
supply gap country.

Demographic India is bestowed with a large working population. It has advantage of reaping this demographic
dividend dividend through green skills.

Greener Green skilling is crucial for making a transition from energy and emissions – intensive economy to
production cleaner and greener production and service patterns.

Quality of It also prepares people for green jobs that contribute to preserving or restoring the quality of the
environment environment while improving human well-being and social equity.

Green skills will make monitoring and managing activities effective in the field of waste energy,
Monitoring
energy efficiency and so on.

ABOUT GSDP

x The pilot project of GSDP was launched in June 2017. Based on the feedback, the GSDP is being scaled
up to an all-India level.
Launch
x More than 30 skilling programmes have been identified to be implemented in 84 institutions across
the country during 2018-19.

x The objective of GSDP is skilling the youth of India, especially dropouts and in increasing the
availability of skilled workforce.
Objective
x The programme endeavours to develop green skilled workers having technical knowledge and
commitment to sustainable development.

x The skilling programmes cover a wide range of fields such as pollution monitoring, waste
management, forest management, wildlife management, bamboo management, livelihood generation
ext.
Course
x All courses will be National Skills Qualifications Framework (NSQF) compliant.
x In the first stage, a pool of Master Trainers/Specialists are being created, who can further train the
youth across the country.

FOCUS | June 2018 | RAU’S IAS 47

Download from:- www.UPSCPDF.com


https://t.me/UPSC_PDF www.UPSCPDF.com https://t.me/UPSC_PDF

Economic Development

BENEFITS

x Such skilling efforts will strengthen the resolve to conserve and protect the environment.
x It will help in the attainment of the Nationally Determined Contributions (NDCs), Sustainable Development
Goals (SDGs), National Biodiversity Targets (NBTs), as well as Waste Management Rules (2016).
x According to an estimate by the ministry, the number of people to be employed under GSDP will be increased to
cover 2.25 lakh people next year and to about 5 lakh people by the year 2021.

VAN DHAN VIKAS The Government of India proposes to expand Van Dhan Vikas

KENDRAS (VDVK) Kendras in Tribal Districts across the country. This initiative is
aimed at mainstreaming the tribal community by promoting
# Project # Growth and primary level value addition to Multi-Fibre Product (MFP) at the
grassroots level.
Development

KEY HIGHLIGHTS OF THE INITIATIVE

About 3000 Van Dhan Kendras are proposed to be set up in two years in the forested Tribal Districts
Target
of the country.

The initiative is proposed to be taken up on priority in the 39 Districts with more than 50% tribal
Coverage
population and then gradually be expanded to other Tribal Districts in India.

At the unit level, aggregation of produce would be done by SHGs having about 30 members each
Aggregation
forming Van Dhan Vikas ‘Samuh’.

x At central level- Ministry of Tribal Affairs as Nodal


Î TRIFED
Department
x The Tribal Cooperative Marketing
x National Agency- Tribal Cooperative Marketing Development Federation of India
Development Federation of India Limited (TRIFED) came into existence in 1987.
(TRIFED)as Nodal Agency at the national level x It is a national-level apex organization
Nodal x At state level- At State level, the State Nodal functioning under the administrative
agency control of Ministry of Tribal Affairs,
Agency for MFPs and the District collectors are
Govt. of India.
envisaged to play a pivot role in scheme
x TRIFED has its registered and Head
implementation at grassroots level.
Office located in New Delhi and has a
x At local level- VDVKs are proposed to be managed network of 13 Regional Offices located
by a Managing Committee (an SHG) consisting of at various places in the country.
representatives of Van Dhan SHGs in the cluster.

Through this initiative, the share of tribals in the value chain of Non-Timber Forest Produce is
Benefit
expected to rise from the present 20% to around 60%.

MFP CENTRIC TRIBAL DEVELOPMENT

x MFP is more aptly referred as Non-Timber Forest Products (NTFPs). These are the primary source of
What is income and livelihood for tribal people in the country.
MFP? x They include fruits and nuts, vegetables, fish and game, medicinal plants, resins, essences, honey,
and a range of barks and fibres such as bamboo, rattans, and a host of other palms and grasses.

FOCUS | June 2018 | RAU’S IAS 48

Download from:- www.UPSCPDF.com


https://t.me/UPSC_PDF www.UPSCPDF.com https://t.me/UPSC_PDF

Economic Development

x It provides essential nutrition to people living in forested areas.


x These are used for household purposes, thus forms an important part of tribals’ non-cash
income.
x For many tribal communities who practice agriculture, MFPs are also a source of cash income,
especially during the slack seasons.
Importance x Tribals are dependent on MFPs for their livelihood and employment opportunities.
x MFPs are especially important for the poorest households living in forested areas, especially
women. Research suggests the high involvement of women in the NTFP economy.
x Tribal have enormous traditional skills in the processes involved in collection and value addition
of MFPs. Hence, based on their local skills and resources, the MFPs based tribal development is
considered as an ideal model.

x 1996 & 2006- Both Panchayats (Extension to Scheduled Areas) Act, 1996 and the Scheduled Tribes
and Other Traditional Forest Dwellers (Recognition of Forest Rights) Act, 2006 which conferred
Government ownership rights on tribal Gram Sabhas in respect of MFP found in their area.
efforts
x 2014- The scheme of Minimum Support Price for selected MFPs were introduced.
x 2018-Van Dhan Vikas Kendra is proposed to be established in the districts across the country.

x Trade related to MFPs is unorganised in nature


x Low returns to MFP gatherers due to its being unorganised
Problem
x High wastage due to limited value addition
x Lack of backward and forward linkages of MFP supply chain

A more holistic approach with robust institutional mechanisms is required for strengthening the
Way
backward and forward linkages of MFP supply chain and particularly for mainstreaming the tribal
forward
community.

The Ministry of Textile has recently released the guidelines on Samarth-


SAMARTH Scheme for Capacity Building in Textile Sector. The scheme targets to train 10
lakh persons (9 lakh in organised and 1 lakh in the traditional sector) over a
# Scheme
period of 3 years (2017-20).

KEY PROVISIONS

x To skill the youth for gainful and sustainable employment in the textile sector covering
the entire value chain of textiles, excluding spinning and weaving.
x To provide demand-driven, placement oriented National Skills Qualifications Framework
(NSQF) compliant skilling programmes.
Objective
x To enable provision of sustainable livelihood either by wage or self-employment to all
sections of the society across the country.
x To promote skilling and skill upgradation in the traditional sectors of handlooms,
handicrafts, sericulture and jute.

x Skilling target and assessment of skill requirement will be done on the basis of ‘Job-role ‘at
Skilling target various levels (such as entry-level) under the scheme.
x The scheme also includes Recognition of Prior Learning (RPL), Training of Trainers,

FOCUS | June 2018 | RAU’S IAS 49

Download from:- www.UPSCPDF.com


https://t.me/UPSC_PDF www.UPSCPDF.com https://t.me/UPSC_PDF

Economic Development

Entrepreneurship Development.

Focus on Skilling requirement in the traditional sectors such as handlooms, handicrafts, jute, silk etc. will
traditional sector be considered as special projects.

Support for skill Skill upgradation will be supported for entrepreneurial development through the provision of
upgradation MUDRA loans.

Web-based monitoring will be adopted for steering every aspect of the implementation of the
Monitoring
programme.

x The scheme will be implemented for the benefit of all sections of the society across the
country including rural, remote, Left Wing Extremism (LWE) affected, North East, J&K by
Beneficiaries imparting skills in the identified job roles.
x Preference will be given to various social groups, SC, ST, differently abled, minorities and
other vulnerable groups.

The Cabinet Committee on Economic Affairs has given its approval


FREIGHT VILLAGE for the development of Freight Village at Nangal Chaudhary,
Haryana. A freight village is a defined area within which all activities
IN HARYANA relating to transport, logistics and the distribution of goods, both for

# Infrastructure national and international transit are carried out by various


operators.

ABOUT FREIGHT VILLAGE

Development of a Freight Village has been considered necessary to achieve the following objectives:
x Improving efficiency of operations of existing industries
x Increasing attractiveness of the region as an investment destination for various categories of
industries, warehousing and logistics operators;
x Tapping potential traffic along the Western Dedicated Freight Corridor (DFC) and of Northern
Objective states.
x Serving as a world-class multi-modal facility that provides for efficient storage/transitioning of
goods to/from DFC.
x Offering an attractive destination to freight companies and customers to boost industrial
development in the region and creating economic opportunities.
x Creating various value-added services.

x The construction of trunk infrastructure will be undertaken in two phases. Phase I will be
implemented by the financial year 2020-21.
x For the implementation of proposed Freight Village, an Special Purpose Vehicle (SPV) has been
incorporated as a 50:50 Joint Venture of Government of India through National Industrial
Key Corridor Development & Implementation Trust (NICDIT) and Government of Haryana through
features Haryana State Industrial & Infrastructure Development Corporation Limited (HSIDC)under the
name "DMIC Haryana Multi Modal Logistic Hub Project Limited".
x The Freight Village will be connected through Western Dedicated Freight Corridor (DFC) at Dabla.
x This logistics hub integrated with the DFC would have the following components:
9 High-speed connectivity to key ports and Industrial Centres;

FOCUS | June 2018 | RAU’S IAS 50

Download from:- www.UPSCPDF.com


https://t.me/UPSC_PDF www.UPSCPDF.com https://t.me/UPSC_PDF

Economic Development

9 Single point solution for all logistics needs like loading/aggregation/packaging/ storage;
9 Last mile gateway port connectivity from the hinterland etc.

x Synergies in logistics processes – e.g. storage, packaging – Freight village warehouses more
productive
x Synergies/Sharing in infrastructure – e.g. Connections to the road, transshipment equipment,
railway sidings
x Reductions in wasted movements, fewer transport links – Internalization of freight movements – a
regional network of freight villages can extend these benefits further
x A firmer basis for coordinated urban distribution
x Benefits encourage intermodal movements

Benefits x A boost for inland regions


x The economic benefits of the project will include
9 creation of employment,
9 reduction in fuel costs,
9 boosts to exports,
9 reduction in vehicle (trucks) operating cost,
9 reduction in accident-related costs,
9 increase in collection of taxes by the State Government,
9 reduction in pollution, etc.

Î RELATED INFORMATION: DMIC

x The Government of India (Gol) is developing the Delhi Mumbai Industrial Corridor, as a global manufacturing and
investment destination around 1,504 km long Western Dedicated Freight Corridor (DFC) as the backbone.
x It includes 24 industrial regions, eight smart cities, two international airports, five power projects, two mass rapid
transit systems, and two logistical hubs.
x Eight Investment Regions and Industrial Areas have been identified for development in this corridor, across six
States namely Gujarat, Haryana, Madhya Pradesh, Maharashtra, Rajasthan and Uttar Pradesh.
x It is with the technical and financial assistance from Japan.
x It was formalized in the year 2007.

THE STATE/UT AGRICULTURE PRODUCE AND LIVESTOCK


CONTRACT FARMING AND SERVICES
(PROMOTION & ACILITATION) ACT, 2018
# Agriculture
The final Model Act “The State/UT Agricultural Produce and Livestock Contract Farming and Services
(Promotion & Facilitation) Act 2018” has been recently approved and released by the Agriculture Minister.
The preparation of “Model Contract Farming Act” was earlier announced by the Finance Minister in the
Budget.
The act is aimed to integrate farmers with bulk purchasers including exporters, agro-industries etc. for
better price realization through mitigation of market and price risks of farmers and ensuring smooth agro
raw material supply to the agro-industries.

FOCUS | June 2018 | RAU’S IAS 51

Download from:- www.UPSCPDF.com


https://t.me/UPSC_PDF www.UPSCPDF.com https://t.me/UPSC_PDF

Economic Development

SALIENT FEATURES OF THE ACT

x The act lays special emphasis on protecting the interests of the farmers, considering them as weaker of the two
parties entering into a contract.
x In addition to contract farming, services contracts all along the value chain including pre-production, production
and post-production have been included.
x “Registering and Agreement Recording Committee” or an “Officer” for the purpose at district/block/ taluka level
for online registration of sponsor and recording of agreement provided.
x Contracted produce is to be covered under crop / livestock insurance in operation.
x Contract framing to be outside the ambit of APMC Act.
x No permanent structure can be developed on farmers’ land/premises.
x Promotion of Farmer Producer Organization (FPOs) / Farmer Producer Companies (FPCs) to mobilize small and
marginal farmers has been provided. It can be a contracting party if so authorized by the farmers.
x No rights, title ownership or possession to be transferred or alienated or vested in the contract farming sponsor
etc.
x Ensuring buying of entire pre-agreed quantity of one or more of agricultural produce, livestock or its product of
contract farming producer as per contract.
x Contract Farming Facilitation Group (CFFG) for promoting contract farming and services at village / panchayat at
level provided.
x Accessible and simple dispute settlement mechanism at the lowest level possible provided for quick disposal of
disputes.
x It is a promotional and facilitative Act and not regulatory in its structure

The Model Agriculture Produce and Livestock Contract


FARMER PRODUCER Farming and Services (Promotion & Facilitation) Act, 2018 has

ORGANISATION (FPO) emphasised on the promotion of Farmer Producer


Organisation (FPO) to mobilize small and marginal farmers.
# Agriculture In this context, let us understand about the FPO.

ABOUT FPO

x A Producer Organisation (PO) is a legal entity formed by primary producers, viz. farmers, milk
producers, fishermen, weavers, rural artisans, craftsmen.
What? x When such entity is created by farmers, it is called as Farmer Producer Organisation.
x It can be a producer company, a cooperative society or any other legal form which provides for
sharing of profits/benefits among the members.

x It ensures better income for farmers through an organisation of their own.


x Small producers do not have the volume individually (both inputs and produce) to get the benefit of
Need for
economies of scale. FPO allows them to achieve economies of scale to farmers through aggregation.
PO
x It also allows them to have better bargaining power vis-à-vis the bulk buyers of produce and bulk
suppliers of inputs.

Essential x It is formed by a group of producers for either farm or non-farm activities.


features of x It is a registered body and a legal entity.
PO x Producers are shareholders in the organization.

FOCUS | June 2018 | RAU’S IAS 52

Download from:- www.UPSCPDF.com


https://t.me/UPSC_PDF www.UPSCPDF.com https://t.me/UPSC_PDF

Economic Development

x It deals with business activities related to the primary produce/product.


x It works for the benefit of the member producers.
x A part of the profit is shared amongst the producers.
x Rest of the surplus is added to its owned funds for business expansion.

Ownership The ownership of the PO is with its members. It is an organization of the producers, by the producers
of PO and for the producers.

DIFFERENCE BETWEEN FPO AND COOPERATIVE SOCIETY

Parameters Cooperative Society Farmer Producer Organisations (FPO)

Registration Cooperative Societies Act Indian Companies Act

Objectives Single object Multi-object

Area of operation Restricted Entire Union of India

Any individual, group, association, producer of


Membership Individuals and cooperatives
goods or services

Not tradable but transferable; limited to


Share Non-tradable
members at par value

Profit Sharing Limited dividends on shares Commensurate with volume of business

One member, one vote, but Government


One member, one vote. Members not having
Voting rights and Registrar of Cooperatives hold veto
transactions with the company cannot vote
power

Highly patronized to the extent of


Government control Minimal, limited to statutory requirements
interference

Fully autonomous, self-ruled within the


Extent of autonomy Limited in “real world scenario”
provisions of Act

Restricted as per bye-law. Any Borrowing limit fixed by Special Resolution in


amendment to bye-law needs to be general meeting. Companies have more
Borrowing powers
approved by the Registrar and time freedom to raise borrowing power
consuming.

GAS TRADING The Petroleum and Natural Gas Regulatory Board (PNGRB) and
HUB/EXCHANGE the Central government are moving ahead to make gas trading
hub a reality by the end 2018.
# Infrastructure #Industry

ABOUT GAS TRADING HUB?

x Natural gas hubs tend to be at the heart of gas infrastructure networks such as pipelines and
liquefied natural gas (LNG) terminals.
What is it?
x The hub is used as a central pricing point for the network’s natural gas. It functions on the lines of
the power exchange, facilitating trade in natural gas.

FOCUS | June 2018 | RAU’S IAS 53

Download from:- www.UPSCPDF.com


https://t.me/UPSC_PDF www.UPSCPDF.com https://t.me/UPSC_PDF

Economic Development

x The most important feature is its role in arriving at the price of the commodity by a market-based
mechanism as against the current price mechanism which is fixed by the government.

x The move would ensure that a market-based pricing mechanism is in place for the commodity.
This also minimizes regulatory interventions by the government.
x A hub is used as a central pricing point for a network that could aid better price discovery for
domestic as well as imported gas.
x This move will help in boosting the consumption of natural gas in the country.
Benefits x The move will help India ushering towards the gas-based economy. Important to note that the
Government has envisaged ushering into a gas-based economy by increasing the share of natural
gas in the primary energy mix of the country from current level of about 6 percent to 15 percent by
2030.
x The mechanism will induce competition among the players, which in turn, will help in reducing the
prices of gas for consumers.

India lacks conducive environment for Gas Trading Hub. It has lack of multiple buyers, sellers, traders,
Challenges
marketers, and infrastructure providers which are essential for promoting gas trading hub.

x Some of the things it will need to decide on are


9 the volume of gas to be traded (removal of allocation of gas mechanism),
9 price deregulation in the purest sense (no cross subsidies or price)
9 rationalise transport tariff (entry-exit systems are more conducive for the better trading
Way environment); and
forward 9 correct the tax regime (local taxes vary from State to State).
x A digital platform is needed to bring producers, suppliers, marketers, traders, infrastructure
providers and customers together and integrate the market in a real time and in an open and
transparent manner.
x There is also a need for unbundling — transport and gas marketing businesses.

x India is not only country launching trading hub.


Main Gas
x China plans to launch a natural gas trading hub in Chongqing this year.
trading
x The world's biggest natural gas hub is the Henry Hub in the US state of Louisiana.
hubs
x Britain has National Balancing Point (NBP) as the main gas hub

Î RELATED INFORMATION: PNGRB

x The Petroleum and Natural Gas Regulatory Board (PNGRB) was constituted under The Petroleum and Natural Gas
Regulatory Board Act, 2006.
x Functions:
9 To protect the interests of consumers and entities engaged in specified activities relating to petroleum, petroleum
products and natural gas;
9 To promote competitive markets;
9 To regulate the refining, processing, storage, transportation, distribution, marketing and sale of petroleum,
petroleum products and natural gas excluding production of crude oil and natural gas;

FOCUS | June 2018 | RAU’S IAS 54

Download from:- www.UPSCPDF.com


https://t.me/UPSC_PDF www.UPSCPDF.com https://t.me/UPSC_PDF

Economic Development

In-Short
FPIs ALLOWED TO INVEST IN TREASURY-BILLS

x The Reserve Bank of India (RBI) has allowed foreign portfolio investors (FPIs) to invest in treasury bills
issued by the central government.
x The central bank’s move comes close on the heels of foreign portfolio investors being permitted to
Why in
invest in corporate bonds with a minimum residual maturity of above one year.
news?
x However, such investment in securities of any category [G-secs, State Development Loans or
corporate bonds] with residual maturity below one year shall not exceed 20% of total investment by
an FPI in that category.

x Treasury bills, generally shortened as T-bills, are money market instruments. These are issued by the
government to raise money from the market. These are presently issued in three maturities,
namely, 91-day, 182-day and 364-day.
x Treasury bills are zero coupon securities and pay no interest. Rather, they are issued at a discount
(at a reduced amount) and redeemed (given back money) at the face value at maturity.
What are
x For example, a 91-day Treasury bill of Rs.100/ (face value) may be issued at say Rs. 98.20, that is, at a
T-Bills?
discount of say, Rs.1.80 and would be redeemed at the face value of Rs.100/-.
x This means that one can get a hundred-rupee treasury bill at a lower price (When he/she buys) and
can get Rupees hundred at maturity.
x The return to the investors is the difference between the maturity value or the face value (that is
Rs.100) and the issue price.

MUTUAL RECOGNITION AGREEMENT

x A mutual recognition agreement (MRA) is an international agreement by which two or more


countries agree to recognize one another's conformity assessments.
x Conformity assessment is testing or other activities that determine whether a process, product, or
services comply with the requirements of a specification, technical standard, contract, or regulation
What is
of other countries.
MRA?
x MRAs have become increasingly common since the formation of the World Trade Organization in
1995.
x MRAs are most commonly applied to goods, such as various quality control MRAs. However, the term
is sometimes applied to agreements on the recognition of professional qualifications as well.

Why in The government has approved the Mutual Recognition Agreement between the Institute of Chartered
news? Accountants of India (ICAI) and The South African Institute of Chartered Accountants (SAICA).

The Agreement will:


Benefit 9 facilitate recognition of Indian Accountancy Professionals with local Accountancy qualification in
addition to the existing ICAI qualification, which will increase their professional avenues in South

FOCUS | June 2018 | RAU’S IAS 55

Download from:- www.UPSCPDF.com


https://t.me/UPSC_PDF www.UPSCPDF.com https://t.me/UPSC_PDF

Economic Development

African markets.
9 foster strong working relations between the two accounting institutes.
9 increase the mobility of professionals at either end and would herald a new dimension for small and
medium businesses in both the countries.

CAPITAL ADEQUACY RATIO (CAR)

x The Reserve Bank of India (RBI) has barred state-run Allahabad Bank from extending fresh loans,
under its prompt corrective action (PCA) framework.
Why in x This was done due to the failure of the bank to meet minimum capital adequacy ratio (CAR)
News? requirement.
x The bank’s CAR stood at 8.69% as on March end against the regulatory minimum requirement of
9%.

ABOUT CAR

x The Capital Adequacy Ratio (CAR) is a measure of a bank's available capital expressed as a
percentage of a bank's risk-weighted credit exposures. This is also known as capital-to-risk
weighted assets ratio (CRAR).
x This is measured by using the following formula:
Tier One Capital + Tier Two Capital
What is it? CAR
Risk Weighted Assets

x Tier one capital is the one which can absorb losses without a bank being required to cease
trading.
x Tier two capital is the one which can absorb losses in the event of a winding-up and so provides a
lesser degree of protection to depositors.

x The reason minimum capital adequacy ratios (CARs) are critical is to make sure that banks have
enough cushion to absorb a reasonable amount of losses before they become insolvent and
consequently lose depositors’ funds.
x The capital adequacy ratios ensure the efficiency and stability of a nation’s financial system
Why CAR? by lowering the risk of banks becoming insolvent.
x During the process of winding-up, funds belonging to depositors are given a higher priority than
the bank’s capital, so depositors can only lose their savings, if a bank registers a loss exceeding the
amount of capital it possesses. Thus, the higher the bank’s capital adequacy ratio, the higher
the degree of protection of depositor's assets.

PM VYAYA VANDANA YOJANA

Why in x As part of commitment for financial inclusion and social security, the Government has given its
News? approval for extending the investment limit from Rs 7.5 lakhs to Rs 15 lakhs under the Pradhan Mantri
Vaya Vandan Yojana (PMVVY).
x It will enable up to Rs.10000 Pension per month for Senior Citizens.

FOCUS | June 2018 | RAU’S IAS 56

Download from:- www.UPSCPDF.com


https://t.me/UPSC_PDF www.UPSCPDF.com https://t.me/UPSC_PDF

Economic Development

About x The PMVVY is being implemented through Life Insurance Corporation of India (LIC) to provide social
Scheme security during old age and protect elderly persons aged 60 years and above against a future fall in
their interest income due to uncertain market conditions.
x The scheme provides an assured pension based on a guaranteed rate of return of 8% per annum for
ten years, with an option to opt for pension on a monthly / quarterly / half yearly and annual basis.
x The differential return, i.e. the difference between the return generated by LIC and the assured return
of 8% per annum would be borne by Government of India as subsidy on an annual basis.

PASIGHAT AIRPORT
x With the Pasighat Airport coming into operation, Arunachal Pradesh has become the last of the eight north-
eastern states to be on India’s commercial flight service map.
x The Pasighat airport is situated in East Siang district of Arunachal Pradesh and now has become India’s easternmost
airport with civilian operations. Earlier, Mohanbari airport near Dibrugarh in Assam was the easternmost civilian
airport in India.
x Pasighat airport, one of six operational advance landing grounds (ALGs) in Arunachal Pradesh primarily for military
use, was laid in 1952 but was virtually abandoned after the China-India war in 1962, until the Indian Air Force took it
over in 2010.
x Earlier, in April, the UDAN scheme connected Meghalaya’s Umroi airport near State capital Shillong. Air connectivity
for Sikkim was also opened in March this year with the operation of Pakyong Airport near Gangtok.
x Assam is the best air-connected State in the northeast, with Guwahati being the communication hub. The other
civilian airports with commercial flight service are at Dibrugarh, Silchar, Jorhat, north Lakhimpur and Tezpur.
x The busiest airports after Guwahati in Northeast are Imphal and Agartala, followed by Lengpui near Mizoram’s
capital Aizawl, and Nagaland’s commercial hub of Dimapur.

e-SIM
x To cater to the needs of modern technological developments in Machine 2 Machine (M2M)/Internet of Things (IoT),
the Department of Telecommunication has decided to permit the use of embedded-subscriber identity module
(eSIM)’
x The eSIM is a virtual equivalent of normal SIM cards and come embedded in the device. These cannot be
removed from the device but can be updated over-the-air.
x While the service can also be rolled out for smartphones — enabling users to port from one service provider to
another without changing physical SIMs, the primary focus is to enable machine-to-machine communication.
x The department also issued instructions for use of SIMs for connected devices such as cars and appliances. These
mobile connections will be issued in the name of the entity providing M2M service. SIMs will have restrictive
features compared to traditional SIMs for person-to-person communication.
x The details of all customers of M2M services will have to be maintained by the M2M service provider.

DIRECTORATE GENERAL OF TRADE REMEDIES (DGTR)


x The Ministry of Commerce has recently issued a notification for a unified Directorate General of Trade
Remedies (DGTR).

FOCUS | June 2018 | RAU’S IAS 57

Download from:- www.UPSCPDF.com


https://t.me/UPSC_PDF www.UPSCPDF.com https://t.me/UPSC_PDF

Economic Development

x DGTR will be the apex national authority for all trade remedial measures including anti-dumping,
countervailing duties and safeguard measures.
x It would subsume the Directorate General of Anti-dumping and Allied duties, Directorate General of
Safeguards and some functions of the Directorate General of Foreign Trade.
x The body will also provide trade defence support to domestic industry and to exporters in trade remedy
investigations instituted by other countries.

INDIAN BUREAU OF MINES


x On the recommendation of the committee- ‘Review and Restructuring of the Functions and Role of IBM’, the
Ministry of Mines has restructured the Indian Bureau of Mines (IBM).
x IBM was set up by the Government of India on 1st March 1948 under the Ministry of Works, Mines and Power.
x It is primarily an advisory body. It helps in the formulation of policy and legal framework for the mining sector and
advising Central and State Governments on development and utilization of mineral resources.
x The role and responsibility of IBM have changed with emerging needs of the sector as facilitator and regulator of
mining sector (other than coal, petroleum and atomic minerals).

FOCUS | June 2018 | RAU’S IAS 58

Download from:- www.UPSCPDF.com


https://t.me/UPSC_PDF www.UPSCPDF.com https://t.me/UPSC_PDF

SOCIETY AND SOCIAL JUSTICE


# GS Paper I & GS Paper III (Main)

For the effective implementation of schemes, citizen


SOCIAL AUDIT participation in governance becomes very important. In this
# Empowerment #Governance regard, beneficiary participation for targeted implementation of
schemes through social audit becomes an important cog. This
#Citizen Participation improves (transparency cycle) by removing paralysis of
(Lead Topic) implementation in the entire process of schemes roll out.

x Social audit is a process of reviewing official records and determining whether state reported
expenditures reflect the actual money spent on the ground.
Meaning of x According to Food and Agriculture Organisation (FAO), a social audit is a way of measuring,
Social Audit understanding, reporting and ultimately improving an organization's social and ethical
performance. A social audit helps to narrow gaps between vision/goal and reality, between
efficiency and effectiveness.

x So far, social audits of government programmes have been done at the initiative of civil society organisations. These
social audits had no official sanction. But to give it a formal shape, through legislation, the state of Meghalaya has
become the leading light by passing first social audit law, The Meghalaya Community Participation and Public
Services Social Audit Act, 2017’.
x This legislation will be applicable for around 20 schemes.

BENEFITS

x Course correction of the schemes during its rolling out so as to remove all bottlenecks such as
First delay, poor implementation, corruption, lack of information or ignorance of actual beneficiary etc.
x This will help in plugging the gaps to increase efficacy.

It gives people a direct say in how money will be spent and fills an information gap for officers as they
Second
are directly in touch with the ground.

Social audits have been civil society initiatives rather than government-mandated. By this legislation, it
Third
has now become part of the system.

Fourth It has set an example for other states to emulate and to roll out citizen friendly governance.

Note: To know more on this extraordinary initiative, please follow the highly informative article given below:

THE MEGHALAYA EXAMPLE


(courtesy: The Hindu)
As the first State to pass a social audit law, its experience is instructive on how to increase awareness of entitlements.

Nearly 300 people, mostly women, gathered in a community hall on a sunny afternoon in Iewshillong village,
Meghalaya waiting for the “social audit meeting” to begin. They wondered what the ‘social audit’, that had
unfolded in their village over the past five days, was going to amount to.

FOCUS | June 2018 | RAU’S IAS 59

Download from:- www.UPSCPDF.com


https://t.me/UPSC_PDF www.UPSCPDF.com https://t.me/UPSC_PDF

Society and Social Justice

Talking it over

When the distribution register of foodgrains under the National Food Security Act was read out, there was an uneasy
silence. One woman raised her hand to say that the dealer had been charging her more than what was recorded. The
dealer, the wife of the village headman, reminded people that they had agreed to pay more to make up for
transportation losses. When the whole hall erupted in cheers of unanimous support for the dealer and disapproval for
her act of apparent disloyalty in saying what she did, she stood and softly said on the mike, “I am not complaining. I
am only stating the truth”. The soft voice of Margaret Shabong was the loudest affirmation of what social audits stood
for — speaking truth to power. Her individual act of stating the truth led to an administrative decision of preventing
rollover of transportation costs of foodgrains to the citizen. The village eventually applauded her, as she spoke again
and stood her ground, and a mature deliberative democracy demonstrated its potential in a remote Khasi village.

In April 2017, Meghalaya became the first State in the country to pass a social audit legislation, the Meghalaya
Community Participation and Public Services Social Audit Act. This Act mandated social audits across 21 schemes and
11 departments.

Later in the year, the Meghalaya government decided to pilot social audits in a campaign mode to unpack the
modalities that would have to be institutionalised across the State for meeting the mandate of the legislation.
Eighteeen villages representing Garo, Khasi and Jaintia Hills were selected for the pilot. The process began in the third
week of November 2017, and culminated with public hearings in 18 villages, including Iewshillong.

The Meghalaya exercise demonstrated how social audits can be developed as an ongoing process through which
citizens participate in the planning, implementation and monitoring of the programme. Meghalaya is a Sixth Schedule
Area, so the audits had to be built on traditional tribal institutions, leveraging their inherent strengths and facilitating
their engagement with contemporary democratic practices. The audits were deliberately positioned to be a platform
for sharing information about schemes, and enhancing awareness amongst people about their entitlements; detecting
beneficiaries who were eligible, but had been left out; recording people’s testimonies; identifying priorities for inputs
for planning; registering of grievances; and pinpointing systemic shortcomings. The critical requirement of recording
financial and procedural irregularities and deviations between fact and record remained a core part of the exercise.

The audits helped identify and bring about evidence-based policy changes. More than 21 issues were identified based
on pilots alone that needed a change in policy, in the interest of the community. For instance, several instances of local
discretion in drawing up pension beneficiary lists for the National Social Assistance Programme (NSAP) and the Chief
Minister Pension Programme were recorded, because the CM pension provided twice as much remuneration as the
NSAP. At a culmination meeting, the government announced parity between the two schemes, benefiting thousands of
pensioners.

Citizen oversight

In India today, there is a growing acknowledgement of social audits as a credible means of institutionalising citizen
oversight. There is therefore an urgent need to come up with a working protocol for facilitating social audits across a
range of interventions. The experience of Meghalaya has taught us how social audit is intrinsically related to processes
of community participation and grievance redress. The Meghalaya pilots have also helped formulate a practical
framework through which that can be done. Draft rules were prepared on the basis of consultation.

By passing and rolling out a social audit law, Meghalaya has made a breakthrough in the framework of accountability
to the people. Social audit is much more than just a tool of “good governance”. Knowing the reluctance of most
government establishments to share power or become accountable, this initiative is unlikely to spread or become
robust, unless driven by citizens groups. Civil society needs to shape the social audit campaign, be a watchdog, and
staunchly protect the independence of the process. Social audits must become part of the demand for effective
legislation for the whole country.

FOCUS | June 2018 | RAU’S IAS 60

Download from:- www.UPSCPDF.com


https://t.me/UPSC_PDF www.UPSCPDF.com https://t.me/UPSC_PDF

Society and Social Justice

SOCIAL INCLUSION
# Inclusive growth # Empowerment
Social inclusion leads to more participation and inclusion of disadvantaged individuals, groups, and sections
in the overall growth and development process. It lays down the process of improving the terms on which
individuals and groups take part in society for improving their ability, opportunity, and dignity.
It brings better access to education, health, land, water and mineral resources, employment and economic
rights along with increased political participation.
In this regard and taking into consideration high growth regions of Asia-Pacific, a UN Economic and Social
Survey of Asia and the Pacific 2018 by United Nations Economic and Social Commission for Asia and the
Pacific (ESCAP) has urged the regional powers to invest in inclusive and sustainable growth.

x It is the regional development arm of the


United Nations for the Asia-Pacific region.
x It has 53 Member States and 9 Associate
Members.
x Its geographical scope stretches from Turkey
in the west to the Pacific island nation of
ESCAP Kiribati in the east, and from the Russian
Federation in the north to New Zealand in the
south.
x This region is home to 4.1 billion people, or two thirds of the world’s population.
x It was established in 1947 in Shanghai, and is headquartered in Bangkok, Thailand.
x It provides a forum for its member States to promote regional cooperation and collective action in
pursuit of the 2030 Agenda for Sustainable Development

x The UN Economic and Social Survey of Asia and the Pacific 2018 report advised the governments
of countries in the Asia-Pacific region to take advantage of the currently favourable economic
conditions in order to address vulnerabilities and enhance the resilience, inclusiveness and
sustainability of their economies.
x The report was launched at the Indian Council for Research in International Economic Relations
Highlights of
(ICRIER). ICRIER was established in 1981 as a not-for-profit research organisation to provide a
the report
strong economic basis for policy making.
x The report described South and Southwest Asia as the fastest growing sub region of the Asia-
Pacific region and urged the countries to increase social spending.
x For less developed countries, it identified that the role of external sources of finance, such as
official development assistance, and global development partnership remains critical.

This is for providing institutional support for rehabilitation to


SWADHAR GREH disadvantaged women i.e. women victims of difficult circumstances.
The Ministry of Women and Child Development is implementing it.
#Scheme #Social Justice The Scheme envisages providing shelter, food, clothing and health as
#Women Empowerment well as economic and social security for these women. It is in
implementation from 2015.

FOCUS | June 2018 | RAU’S IAS 61

Download from:- www.UPSCPDF.com


https://t.me/UPSC_PDF www.UPSCPDF.com https://t.me/UPSC_PDF

Society and Social Justice

Under the Scheme, Swadhar Greh is to be set up in every district with capacity of 30 women with the
following objectives:
9 To cater to the primary need of shelter, food, clothing, medical treatment and care of the women
in distress and who are without any social and economic support.
9 To enable them to regain their emotional strength that gets hampered due to their encounter with
unfortunate circumstances.
Objectives
9 To provide them with legal aid and guidance to enable them to take steps for their readjustment in
family/society.
9 To rehabilitate them economically and emotionally.
9 To act as a support system that understands and meets various requirements of women in
distress.
9 To enable them to start their life afresh with dignity and conviction.

For big cities and other districts having more than 40 lakh population or those districts where there is a need for
additional support to the women, more than one Swadhar Greh could be established.

The benefit of the component could be availed by women above 18 years of age of the
following categories:
9 Women who are deserted and are without any social and economic support;
9 Women survivors of natural disasters who have been rendered homeless and are without any
social and economic support;
Beneficiaries 9 Women prisoners released from jail and are without family, social and economic support;
9 Women victims of domestic violence, family tension or discord, who are made to leave their homes
without any means of subsistence and have no special protection from exploitation and/ or facing
litigation on account of marital disputes; and
9 Trafficked women/girls rescued or runaway from brothels or other places where they face
exploitation and Women affected by HIV/AIDS who do not have any social or economic support.

Recently, Government of India signed an agreement with


NATIONAL NUTRITION the World Bank today for a USD 200-million loan for the
National Nutrition Mission (POSHAN Abhiyaan) to achieve
MISSION its goal of reducing stunting in children aged 0-6 years
#Scheme #Social Justice from 38.4 per cent to 25 per cent by 2022.
The POSHAN Abhiyaan was launched in March 2018 in
#Empowerment
Jhunjhunu in Rajasthan.

POSHAN ABHIYAAN

x It is a flagship programme of the Ministry of Women and Child Development (MWCD), Government of
India.
x It ensures convergence with various programmes i.e.,
Ministry
9 Anganwadi Services,
9 Pradhan Mantri Matru Vandana Yojana (PMMVY),
9 Scheme for Adolescent Girls (SAG) of MWCD Janani Suraksha Yojana (JSY),

FOCUS | June 2018 | RAU’S IAS 62

Download from:- www.UPSCPDF.com


https://t.me/UPSC_PDF www.UPSCPDF.com https://t.me/UPSC_PDF

Society and Social Justice

9 National Health Mission (NHM),


9 Swachh-Bharat Mission,
9 Public Distribution System (PDS), Department Food & Public Distribution,
9 Mahatma Gandhi National Rural Employment Guarantee Scheme (MGNREGS),
9 Ministry of Drinking Water and Sanitation

x To achieve improvement in nutritional status of Children from 0-6 years, Adolescent Girls, Pregnant
Women and Lactating Mothers in a time bound manner.
x The programme through the targets will strive to reduce the level of stunting, under-nutrition, anemia
Goals and low birth weight babies.
x Overall target: NNM targets to reduce stunting, under- nutrition, anemia (among young children,
women and adolescent girls) and reduce low birth weight by 2%, 2%, 3% and 2% per annum
respectively.

According to Lancet Global Health, there have been 2.39 lakhs “excess
GENDER BIAS deaths” per year of girls under the age of five in India. This works out
# Social Issue # Health to about 2.4 million deaths in a decade.

FOCUS | June 2018 | RAU’S IAS 63

Download from:- www.UPSCPDF.com


https://t.me/UPSC_PDF www.UPSCPDF.com https://t.me/UPSC_PDF

Society and Social Justice

x Biased mentality, preference for boys and illegal sex determination tests.
x This is despite Pre-Conception and Pre-Natal Diagnostic Techniques (Prohibition Of Sex
Major reasons Selection) Act (PCPNDT Act) to improve the regulation of the technology used in sex selection.
x Many deaths of GIRLS under five are partly due to unwanted child bearing and subsequent
neglect.

x Northern India, where the four largest States in the region, Uttar Pradesh, Bihar, Rajasthan,
and Madhya Pradesh, accounted for two thirds of the total excess deaths of females under
Worst
five.
performers
x The worst affected areas were all rural, agricultural areas with lower levels of education, high
population densities, low socio-economic development and high levels of fertility.

In-Short
NIDAAN SOFTWARE
#Health #Technology

Why in The government of Rajasthan has launched a new software, Nidaan, for presumptive diagnosis and
news? monitoring of seasonal and non-communicable diseases.

x To facilitate online entries of many diseases and healthcare services available for their treatment in
Purpose different institutions.
x To help in formulation of specific action plans for control of diseases.

Viewpoint

More on Social Audit


# Empowerment #Governance #Citizen Participation
(Courtesy: The Hindu)

Read and ponder!


The breakdown of institutions has underlined the fact that democracy — and especially public funds — need eternal
public vigilance. But in India, the elites close ranks to neutralise voices of dissent and alarm, thus preventing public
vigilance.
Democratic governance needs the citizen to be legally empowered to ask questions, file complaints, and be a part of
the corrective process. Social audits, as they have begun to evolve in India, can potentially become a powerful
democratic method by which transparency can be combined with an institutionalised form of accountability to the
people.

FOCUS | June 2018 | RAU’S IAS 64

Download from:- www.UPSCPDF.com


https://t.me/UPSC_PDF www.UPSCPDF.com https://t.me/UPSC_PDF

Society and Social Justice

In the mid-1990s, the Mazdoor Kisan Shakti Sangathan (MKSS) experimented with, and began to conceptualise, village-
based Jan Sunwais (public hearings) on development expenditure. These helped establish the Right to Information (RTI)
as a potent, usable people’s issue and, in parallel, the institutionalised form of social audits.

INFORMATION IS EMPOWERING

In a Jan Sunwai campaign, organised in five different development blocks of central Rajasthan, people learnt by doing.
They realised that information is at the core of their empowerment. The process of verification, inquiry and auditing of
records was demystified. Public readings of informally accessed development records had dramatic outcomes. As the
names were read out from government labour lists, the responses were immediate and galvanised the people.
Information about payments made to dead people and non-workers propelled residents to testify in the Jan Sunwai.
These included serving government and armed forces personnel and names randomly copied in serial order from
electoral lists. Even animals absurdly enough found their way into the lists of workers. Unfinished buildings without
doors, windows or a roof were shown as audited and ‘complete’. Ghost names and ghost works were exposed. Fake
development works paid for and ‘completed’ on paper enraged local residents.

The people made four sharply focussed demands and circulated them in a pamphlet: full and open access to records
of development expenditure; the presence and accountability of officials who are responsible to answer people’s
questions; the immediate redress of grievances, including the return of defalcated money to its intended purpose; and
mandatory ‘social audits’ .

Amitabh Mukhopadhyaya, then an officer of the IA&AS, who visited, watched and contributed to the architectural
growth of the process till he passed away a year ago, remarked that this was “audit returning to its roots”: the word
audit comes from the Latin word audiere, which means “to hear”. The Jan Sunwai facilitated the reading of
information and recorded the people’s response. The effective institutionalisation of this platform could be a
fundamental breakthrough in the attempt to give people and communities real monitoring powers. One of the defining
slogans of the RTI movement that emerged from these Jan Sunwais and people’s agitations — “hamaara paisa,
hamaara hisab” (our money, our accounts) — succinctly encapsulated the concept of a social audit.

The RTI Act brought into effect the first prerequisite for social audits — giving citizens access to government records.
The last 13 years of its use have demonstrated its salutary effect, but also made it obvious that information itself is not
enough. Contemporary discourse on the RTI reflects frustration when ordinary people are armed with information but
are unable to obtain any redress. The social audit places accountability in the centre of its frame, and transfers the
power of scrutiny and validation to the people: a citizen-centric mode of accountability.

THE POWER OF SOCIAL AUDITS

The social audit is conceptually simple. Information is to be proactively shared amongst people so that they can “
performance audit” a service or programme, from planning, to implementation and evaluation. This is, however,
easier said than done.

An independent facilitation structure needs to be set up, fleshed out, legally empowered and mandated to ensure that
social audits are conducted. The relationship between the powerful and the powerless has to shift from patronage to
rights, and from inequality to equality, making the right to question sacrosanct. Specific methods of sharing
information, recording comments and acting on findings have been worked out. They now need to be acted upon.

The Mahatma Gandhi National Rural Employment Guarantee Act (MGNREGA) was the first law to mandate social audit
as a statutory requirement. However, even within the MGNREGA, social audits made painfully slow progress. They
faced their most trenchant opposition in Rajasthan, where the concept was born. Elected representatives and officials
reacted with intimidation, violence and pressure on the political leadership to stall and neutralise the process. The
notable exception was undivided Andhra Pradesh which institutionalised social audits and drew significant positive
outcomes. There have been innovative efforts in States like Sikkim, Tamil Nadu and Jharkhand. Nationally,
institutionalised social audits have begun to make real progress only recently, with the interest and support of the

FOCUS | June 2018 | RAU’S IAS 65

Download from:- www.UPSCPDF.com


https://t.me/UPSC_PDF www.UPSCPDF.com https://t.me/UPSC_PDF

Society and Social Justice

office of the Comptroller and Auditor General (CAG), and the orders of the Supreme Court. In what was a social audit
breakthrough in 2017, Meghalaya became the first State to pass and roll out a social audit law to cover all departments.

The Office of the CAG developed social audit rules for the MGNREGA in 2011, conducted a performance audit in 2015,
and finally a year later formulated social audit standards in consultation with the Ministry of Rural Development — the
first time in the world. If these are followed, it can be ensured that the social audit process is viable, credible and true
to first principles of social accountability.

The Supreme Court has recently passed a series of orders to give social audits the robust infrastructural framework
they need. Citing the statutory requirements in the MGNREGA and the National Food Security Act, the court has
ordered that the CAG-formulated Social Audit Standards be applied to set up truly independent state-supported State
Social Audit units. It has also ordered that social audits be conducted of Building and other Construction Workers Cess,
and the implementation of the Juvenile Justice Act. Social audits, if properly implemented, will help address the
impunity of the system in delivery and implementation.

The current dispensation makes a cursory mention of social audits in its manifesto. But there has been no delivery on
legal accountability frameworks such as the Lokpal Bill and the Whistle Blowers Protection Bill. The system of social
audits needs synergetic endorsement and a push by multiple authorities to establish an institutionalised framework
which cannot be undermined by any vested interests. It is now an opportune time for citizens groups to campaign to
strengthen social audits, and make real progress in holding the political executive and implementing agencies to
account.

FOCUS | June 2018 | RAU’S IAS 66

Download from:- www.UPSCPDF.com


https://t.me/UPSC_PDF www.UPSCPDF.com https://t.me/UPSC_PDF

SCIENCE & TECHNOLOGY


# GS Paper (Prelims) & GS Paper III (Main)

Automated External Defibrillators are portable electronic medical


AUTOMATED devices that detect life-threatening cardiac activity from irregular
EXTERNAL heartbeats to pulse. It is capable of reviving the heart by
defibrillation.
DEFIBRILLATOR Defibrillation is the application of electricity to restore cardiac
rhythm. Failure to use a defibrillation within 4 to 6 minutes of
(AEDs)
suffering cardiac arrest is a major cause of over 95 % of sudden
#Technology deaths.

THINGS TO UNDERSTAND

x During a cardiac arrest, the heart stops


functioning due to some problem with electrical
activity of the heart, which disrupts pumping
action.
x Once a cardiac arrest occurs, a person drops
down unconscious and has no pulse within
seconds. If not attended, a person may suffer
death and within minutes.
x Emergency services in India take time and hence
first-aid measures have a crucial role to play to
save lives.
x So installation of AEDs in public places can play a
game changing role.
x Being small in size, the device can be installed in
buildings, markets malls, security guard rooms
and at entry and exits.
x AEDs are easy to use and come with preloaded
audio and video tutorials. This means any lay
person can also use the AED in the hour of need.

RUTHENIUM’S The scientists from the University of Minnesota, US has discovered that
Ruthenium (Ru) is the fourth element now to possess unique magnetic
MAGNETISM properties at room temperature after Iron (Fe), Cobalt (Co), and Nickel (Ni)

#General Science showed ferromagnetism at room temperature.

FOCUS | June 2018 | RAU’S IAS 67

Download from:- www.UPSCPDF.com


https://t.me/UPSC_PDF www.UPSCPDF.com https://t.me/UPSC_PDF

Science & Technology

ABOUT RUTHENIUM

Chemical x Ruthenium is an element in the periodic table with atomic number 44.
Element x The name is the latin word for Russia.

Rare x Ruthenium, together with rhodium, palladium, osmium, iridium, and platinum form a group of
Transition elements referred to as the platinum group metals (PGM).
metal x It is rare transition metal. Being a member of the platinum group, it is inert to most other chemicals.

Discovery It was discovered in 1844 by Russian scientist Karl Ernst.

Ruthenium is one of the rarest metals on Earth. It is found as the free metal, sometimes associated with
Location
platinum, osmium and iridium, in North and South America, and in South Africa.

x The metal find use in the electronic industry (50%) and the chemical industry (40%), with smaller
amounts being used in alloying.
x Ru is mainly used to make electric contacts and thick-film resistors.
Use
x The discovery of magnetism will help researchers to improve sensors, devices in computer memory
and logic industry.
x Interestingly, Ru is resistant to oxidation and additional has high thermal stability.

x Ferromagnetism is a physical phenomenon in which electrically neutral materials strongly


attract other elements.
Ferromagnetism
x Today, ferromagnetic materials are used in a wide variety of devices essential to everyday life
like electric motors and generators, transformers, telephones, and loudspeakers.

Indian scientists are conducting experiments to confirm


COSMIC MICROWAVE occurrence of unusual space signals in the spectrum of
Cosmic Microwave Background (CMB) radiation.
BACKGROUND (CMB)
The location of the experiment is Timbaktu village in Andhra
#Space Pradesh. Let us understand the significance.

ABOUT CMB

x CMB is a weak electromagnetic radiation from the early universe when matter was about to be
formed.
x This radiation does not have its origin in cosmic enitities that are observed in the universe, like stars
or galaxies instead comes from things that are not formed yet.
What? x CMB is a relic from an early universe when matter and radiation were still in thermodynamic
equilibrium.
x CMB holds the key from the early universe when matter and radiation were still dormant. Thus
when it is observed it is similar to looking at the period in universe after the big bang and before
the present objects were formed.

Indian Indian scientists are conducting experiments to confirm occurrence of unusual space signals of Cosmic
attempt Microwave Background (CMB) radiation.

FOCUS | June 2018 | RAU’S IAS 68

Download from:- www.UPSCPDF.com


https://t.me/UPSC_PDF www.UPSCPDF.com https://t.me/UPSC_PDF

Science & Technology

x By studying the properties of CMB, scientists have come to conclude that the early universe was
Significance filled with hot dense and extremely uniform gas with hydrogen making the most of it.
of research x The most accepted theory of the origin of the universe, the big bang, can be proved with the
findings from CBM.

x Timbaktu of Andhra Pradesh is surrounded by hills with peculiar rock formations which allows least
Why contact with the outside world and minimal interference of digital technology.
Timbaktu? x The atmospheric conditions at timbaktu are ideal with clean air, clear sky and no noise. These
conditions make it radio quiet allowing it to detect electromagnetic signals from space.

The GRACE-FO or Gravity Recovery and Climate Experiment (Follow-On) is joint

GRACE-FO initiative between NASA and German Research Centre for Geosciences (GFZ). It is
launched to track Earth’s water Cycle. It is subsequent mission to original GRACE
#Space mission, which has already mapped Earth’s water and ice by measuring changes
in Earth’s gravitational field for a period starting from 2002 to 2017.

THINGS TO UNDERSTAND

GRACE-FO will take the work GRACE forward and continue study of rising sea levels, melting of
Role
glaciers and polar ice caps and other changes in distribution of water on Earth.

Launch vehicle The GRACE-FO mission was launched on Elon Musk’s promoted SpaceX rocket Falcon 9.

GRACE-FO comprises of 2 satellites which will orbit at 490 Km near the Polar-orbit orbiting the
Components
earth in 90 minutes.

x Earth does not have a uniform land form. Simply put, some places have mountains while some
areas are oceans. Depending on the land form, the gravitational pull exerted on the satellites
changes.
Interferometer x So, the satellites are equipped with Laser Ranging Interferometer which can easily measure
changes in gravity based on the change in the distance between the 2 satellites.
x This is because as the gravitational pull on any satellite would change, the distance between the
satellites would change.

x Among its innovations, GRACE was the first mission to measure the amount of ice being lost
from the Greenland and Antarctic ice sheets.
x The mission improved our understanding of the processes responsible for sea level rise and
Achievements
ocean circulation.
of GRACE
x It provided insights into where global groundwater resources are shrinking or growing,
showed where dry soils are contributing to drought, and monitored changes in the solid Earth,
such as from earthquakes.

FOCUS | June 2018 | RAU’S IAS 69

Download from:- www.UPSCPDF.com


https://t.me/UPSC_PDF www.UPSCPDF.com https://t.me/UPSC_PDF

Science & Technology

The Health Department of Kerala has confirmed Nipah virus (NiV) infection
NIPAH VIRUS for the first time in the State with the blood and body fluid samples of two
persons who died of viral fever in the Kozhikode district indicating the
#Health presence of the zoonotic disease.

NIPAH INFECTION CYCLE

ABOUT NIPAH VIRUS

x Nipah virus was first recognized in 1999 during an outbreak among pig farmers in, Malaysia.
Past x It was then recognized in Bangladesh in 2001, and nearly annual outbreaks have occurred in that
Outbreak country since.
x The disease has also been identified periodically in eastern India.

Virus Nipah virus (NiV) is transmitted from animals to humans and can also be transmitted through
contaminated food or directly between people.

Hosts x Fruit bats of the family Pteropodidae are the natural hosts for Nipah virus.
x However, there is no disease in fruit bats.

Antibodies African fruit bats of the genus Eidolon(not to be remembered) were found positive for antibodies
against Nipah.

FOCUS | June 2018 | RAU’S IAS 70

Download from:- www.UPSCPDF.com


https://t.me/UPSC_PDF www.UPSCPDF.com https://t.me/UPSC_PDF

Science & Technology

x In infected people, it causes a range of illnesses from infection to respiratory illness and fatal
encephalitis.
Impacts
x The virus can also cause severe disease in animals such as pigs, resulting in significant economic
losses for farmers.

x Infected people initially develop influenza-like symptoms of fever, headaches, myalgia (muscle pain),
vomiting and sore throat.
x This can be followed by dizziness, drowsiness, altered consciousness, and neurological signs that
Symptoms indicate acute encephalitis.
x Some people can also experience atypical pneumonia and severe respiratory problems, including
acute respiratory distress. Encephalitis and seizures occur in severe cases, progressing to coma
within 24 to 48 hours.

The incubation period, interval from infection to the onset of symptoms, is believed to range from 4 to
Incubation
14 days. However, an incubation period as long as 45 days has been reported.

The case fatality rate is estimated at 40% to 75%. This rate can vary by outbreak depending on local
Fatality
capabilities for epidemiological surveillance and clinical management.

Diagnosis Treatment Prevention

x Nipah virus infection


can be diagnosed with x There are currently x Routine and thorough cleaning and disinfection of pig
clinical history. no drugs or vaccines farms with appropriate detergents may be effective in
specific for Nipah preventing infection.
x The main tests used are
real time polymerase virus infection. x If an outbreak is suspected, the animal premises should
chain reaction (RT- x WHO has identified be quarantined immediately.
PCR) from bodily fluids Nipah as a priority x Culling of infected animals may be necessary to reduce
and antibody detection disease for Research the risk of transmission to people.
via enzyme-linked and Development. x Restricting or banning the movement of animals
immunosorbent assay x Intensive supportive from infected farms to other areas can reduce the
(ELISA). care is recommended spread of the disease.
x Other tests used include to treat severe x As Nipah virus outbreaks have involved pigs and/or fruit
polymerase chain respiratory and bats, establishing an animal health/wildlife surveillance
reaction (PCR) assay, neurologic system is essential in providing early warning for
and virus isolation by complications. veterinary and human public health authorities.
cell culture.

ISRO is developing an environment-friendly propellant blend based on


hydroxylammonium nitrate (HAN) to power satellites and spacecrafts of
ISRO’s NEW the future. HAN is a monopropellant formula that consists of ammonium
PROPELLANT nitrate, methanol and water. The new propellant will soon replace the
conventional hydrazine rocket fuel. HAN is a greener alternative for future
#Space missions. Moreover, it is cost effective, re-usable, recoverable and reliable
for space launches of ISRO.

FOCUS | June 2018 | RAU’S IAS 71

Download from:- www.UPSCPDF.com


https://t.me/UPSC_PDF www.UPSCPDF.com https://t.me/UPSC_PDF

Science & Technology

THINGS TO UNDERSTAND

Role of
The methanol is added in the HAN formulation as it helps to decrease the combustion instability.
Methanol

Freezing The HAN-based propellant has a lower freezing point than the conventionally used. Further, the
point propellant allows better control over the burn rate.

Being a mono-propellant, it does not require use of separate oxidizer. Simply put, it does not require air
Uniqueness
to burn thus allowing its use in satellites to correct their orbits and orientation.

Presently, ISRO uses Hydrazine as it has a very high specific impulse.


Current
Specific impulse is a measure of fuel efficiency which is dependent on the amount of energy that can be
Propellant
obtained for the satellite launch vehicle when 1 Kg of propellant is burned for 1 second.

Problems x The use of hydrazine has various problems. The propellant is highly corrosive and so reduces the life
with of the components.
Hydrazine x Further, it is carcinogenic and highly toxic in nature, so requires extremely careful handling.

IPC APPROVES ANIMAL-FREE TESTS FOR DRUGS


#Biotechnology
Research on animals has been practiced since around 500 BC. The animals are used to develop medical
treatments, determine the toxicity of medications, the safety of the medicines, and other biomedical,
commercial, and healthcare uses. Animal testing helps the researchers to study numerous diseases.
Diseases are artificially produced in laboratories in an attempt to mimic the human disease allowing
researchers to develop a new drug by testing on animals and to obtain cues on how the disease develops in
the human body.
To prevent more exploitation and misery to animals, the Indian Pharmacopoeia Commission (IPC) has
approved modern, animal-free tests for drug manufacturers to free animals from painful drug testing.

FOCUS | June 2018 | RAU’S IAS 72

Download from:- www.UPSCPDF.com


https://t.me/UPSC_PDF www.UPSCPDF.com https://t.me/UPSC_PDF

Science & Technology

1. Pyrogen Test: The Pyrogen Test is performed to check the presence or absence of pyrogens that
can cause adverse side-effects. These tests are generally performed on Rabbits as their body
Type of
temperature increases when pyrogen is introduced.
tests done
2. Abnormal toxicity test: The abnormal toxicity test is a form of quality control conducted to detect
on animals
any potential hazardous biological contamination prior to approval for direct administration in
humans. The test is conducted on mice or guinea pigs.

x The Indian Pharmacopoeia Commission (IPC) is an autonomous institution of the Ministry of Health
and Family Welfare, Government of India.
x The sets standards for drugs in the country and to update regularly the standards of drugs
About IPC
commonly required for the treatment of diseases prevailing in this region.
x It publishes official documents for improving Quality of Medicines by adding new and updating
existing guidelines in the form of Indian Pharmacopoeia (IP).

x Indian Pharmacopoeia (IP), the Commission has provided guidelines on tests for drugs
manufactured and marketed in India.
IPC’s new
x The Commission has replaced the pyrogen test carried out on rabbits and the abnormal toxicity test
guidelines
carried out on guinea pigs and mice, with tests that can be done in test tubes.
x The new guidelines will come into effect from 1st July, 2018.

x The move will be beneficial for saving animals and increase focus on alternative methods.
Significance x Moreover, the animal testing is costly. Animal-free testing would ultimately result in reducing the
cost of drugs.

x Developed countries like the USA and European pharmacopeia have already removed the abnormal
toxicity test.
x It was in 2013 that India banned the testing of cosmetic products making it first South Asian
Way ahead
country to do so.
x With the new guidelines of IPC, India is in a position to lead several nations to the path of ethical
health care development.

Other x In India, the Drugs and Cosmetics Act, 1940 regulates the manufacture, distribution, imports of

FOCUS | June 2018 | RAU’S IAS 73

Download from:- www.UPSCPDF.com


https://t.me/UPSC_PDF www.UPSCPDF.com https://t.me/UPSC_PDF

Science & Technology

Regulations cosmetics and drugs in India.


in India x Its aim is to ensure that the drugs and cosmetics sold in the country are safe, effective, and meet
standards of quality.

Viewpoint

CAN ARTIFICIAL INTELLIGENCE HELP WEED OUT FAKE NEWS


IN A POST-TRUTH ERA?
(courtesy: The Hindu)

Earlier this year, while Myanmar and Sri Lanka were caught in the throes of social media-fuelled communal violence,
watchdogs and analysts studying the phenomenon of misinformation also turned their attention to Karnataka, where
things seemed to be headed the same way. Last month’s state elections were considered to be a preview of sorts for
the 2019 national elections, with opposing factions wielding the power of the Internet to compete with each other.

A so-called BBC poll indicating that the BJP would win 135 seats — fake. A 20-second clip of Congress President Rahul
Gandhi “exposing” former Karnataka chief minister Siddaramaiah’s corruption (retweeted by minister Smriti Irani) — a
2013 video taken completely out of context.

These were just two of the claims negated by BOOM, a fact-checking website that partnered with Facebook during the
lead up to the elections. “Facebook created a dashboard, where you can see what users are flagging as misinformation,
and then we would fact check that,” shares Govindraj Ethiraj, co-founder of the Mumbai-based organisation.

FACTS VS EMOTIONS

Late last month, Facebook released Facing Facts, a 12-minute video directed by Academy Award winner Morgan
Neville. Half bashful confession, half cheerful PR stunt, the film features employees emphasising their commitment to
fighting the spread of false news. This includes larger fact-checking teams, artificial intelligence, and machine-learning
systems. One interviewee confided that the problem was the News Feed. It was algorithmically designed to incite
emotional responses from users. But it turned out to be so good at its job that it led to the spread of highly-targeted,
incendiary, and false content.

“This is a rapidly transforming, no, transmogrifying entity,” says Ethiraj, about the widening spread of misinformation
across India, going on to state that, as the Karnataka elections revealed, Facebook was not the largest source of false
news. The credit went to another beast, also owned by the Internet giant. As a recent Washington Post article stated,
“Elections in India are now fought and won on WhatsApp.”

Krish Ashok, techie and blogger, points out that with its end-to-end encryption that often makes it impossible to trace
the source of a viral forward, WhatsApp poses a bigger problem than Facebook in a country which, with 200 million
users, forms its largest market. “It is literally controlled by small groups of people, promoting their own biases and their
own fake news,” he says. “There are no algorithms at play at all. In some sense, WhatsApp is a reflection of existing
polarisations in society, weaponised by communications technology.”

TECH TO THE RESCUE

Can AI and machine-learning, promoted by Internet giants as part of their new manifestos, help solve the
spread of misinformation? In some ways, machine-learning algorithms helped create the very targeting system that
was successfully exploited in the run up to the 2016 US elections. The technology also works more successfully with

FOCUS | June 2018 | RAU’S IAS 74

Download from:- www.UPSCPDF.com


https://t.me/UPSC_PDF www.UPSCPDF.com https://t.me/UPSC_PDF

Science & Technology

flagging clickbait rather than false news, because the former is much easier to identity. However, Vinay Anand, co-
founder of Pipes, an AI-based news aggregator app, believes there is some hope. “AI can come in to stem the spread of
fake news,” he says, “But before that, we need to have a number of reliable and reputable sources to fact check
against.”

As Sanjana Hattotuwa, founding editor of Groundviews, the Sri Lankan civic media platform points out, “There is no
way out of employing AI, since the volume of content production is exponentially growing, and simply scaling up
human reviewers will not help. What AI can and must do is algorithmic red-flagging.”

In an interview with Wired, Eduardo Ariño de la Rubia, data science manager at Facebook, says, “Misinformation can
come from any place that humans touch, and humans touch a lot of places.” Even as artificial intelligence is poised to
play a big role in curbing the spread of misinformation, it is impossible to deny that fake news is a human construct,
born out of existing biases.

HOW TO CREATE AN AI MIND

Æ Dr Andreas Vlachos (lecturer, right) and James Thorne (PhD student, left) from the Department of Computer
Science, University of Sheffield, UK, weigh in.

Æ Combating the spread of misinformation poses particular challenges in India, which has 22 official languages and
many beyond the list. This linguistic diversity, combined with an increase in smartphone use, can cause
misinformation to spread quickly, and on a huge scale.

Æ Human fact-checkers try their best to flag and address dubious information, but there are just too many false
stories and too few people. Could AI (artificial intelligence) fill this gap? Automated fact-checking, using natural
language processing to identify whether the statistics being used in a particular claim are true or false, has been an
area of focus in our research.

Æ Together with colleagues at Amazon Research Cambridge, we have collaborated on a project called FEVER (Fact
Extraction and VERification), using AI to fact-check information. We created a database of 2,00,000 claims written by
humans, with each claim labelled with evidence selected from Wikipedia.

Æ We designed and introduced this database with the aim of pushing the bounds of current Natural Language
technologies: requiring computers to reason about whether claims can be supported or refuted from evidence.

Æ We have made the FEVER database publicly available (fever.ai) so that computer scientists, researchers, social
scientists, journalists and industry can work together against the spread of misinformation. We’re not quite there
yet, but hopefully one day soon, AI will be a key component in solving this major challenge.

FOCUS | June 2018 | RAU’S IAS 75

Download from:- www.UPSCPDF.com


https://t.me/UPSC_PDF www.UPSCPDF.com https://t.me/UPSC_PDF

GEOGRAPHY, ENVIRONMENT, BIODIVERSITY &


DISASTER MANAGEMENT
# GS Paper (Prelims) and GS Paper I & III (Main)

According to a recent study, East Asia with its faster


OZONE LAYER DEPLETION push towards economic growth, is impacting Ozone
layer depletion, due to usage of banned chemicals like
#Environment Conservation #Pollution
CFC-11( Chloro-floro-carbons).

x Ozone layer lies in the stratosphere


between 10-40 kms above earth’s surface.
x It protects life on earth from the deadly
Ultra Violet radiations.
Ozone depleting substances
x Chlorofluorocarbons (CFCs)

Halons
x Carbon tetrachloride (CCl4), Methyl
chloroform (CH3CCl3)
x Hydrobromofluorocarbons (HBFCs)
x Hydrochlorofluorocarbons (HCFCs)

Methyl Bromide (CH3Br)


x Bromochloromethane (CH2BrCl)

KNOW MORE: CONVENTIONS FOR PROTECTION OF OZONE LAYER

x It is multilateral environmental agreement agreed upon at the 1985 Vienna Conference and
entered into force in 1988.

Vienna x It acts as a framework for the international efforts to protect the ozone layer,
Convention these efforts are laid out in the accompanying Montreal Protocol
x It does not include legally binding reduction goals for the use of CFCs
x It is the most widely ratified treaties in UN history. 197 countries have ratified this.

x Finalised in 1987 and effective from 1989.


x It is an international treaty designed to protect ozone layer by phasing out production of
Montreal
numerous Ozone Depleting Substances (ODSs) that are responsible for ozone depletion.
Protocol
x Under it, production and consumption of key ODSs like chlorofluorocarbon (CFCs), Methyl
Chloroform, CTC halons and Methyl Bromide have been phased-out globally.
x It has legally binding obligations on 197 member countries.

Kigali x Kigali Agreement, amended the 1987 Montreal Protocol and aims to phase out

FOCUS | June 2018 | RAU’S IAS 76

Download from:- www.UPSCPDF.com


https://t.me/UPSC_PDF www.UPSCPDF.com https://t.me/UPSC_PDF

Geography, Environment, Biodiversity & Disaster


Management

Agreement Hydrofluorocarbons (HFCs), a family of potent greenhouse gases by the late 2040s.
x CFCs were discontinued under the Montreal protocol
x As an alternative to CFCs, HFCs were introduced in 1990. However, later it was realised that HFCs
are vigorously contributing to the global warming. Thus Kigali agreement aims at banning HFCs.
x Kigali agreement will be a legally binding on member countries from 2019.

Delegates representing various governments met recently in


Bonn, Germany for the next round of UN climate change
Bonn Climate Change negotiations to further develop the guidelines for
Conference implementing the landmark 2015 Paris Climate Change
Agreement. The final decisions are to be taken at the UN
#Environment Conservation Climate Change Conference (COP24) in Poland at the end of
this year.

THINGS TO UNDERSTAND

Conference The Conference of the Parties (COP) is the supreme governing body of the United Nations Framework
of Parties Convention on Climate Change.
(COP) Heads of state and other delegates from UN member countries attend the conference.

WHAT HAPPENED AT BONN?

Aim: The aim was to operationalize the Paris agreement, guidelines, or operating rulebook, to unlock practical actions
to realize the full potential of the agreement.

FOCUS | June 2018 | RAU’S IAS 77

Download from:- www.UPSCPDF.com


https://t.me/UPSC_PDF www.UPSCPDF.com https://t.me/UPSC_PDF

Geography, Environment, Biodiversity & Disaster


Management

UNSETTLED ISSUES

Issues raised by Developing Issues raised by Least Developed Countries Issues raised by
Countries Developed Countries

x They wanted the rulebook to cover x The issues related to loss and damage (L&D) Developed or rich
mitigation targets, adaptation are another thorn in the negotiations. countries would like the
and the means of x L&D is a means to provide assistance to rulebook to be limited to
implementation for the Intended poor countries that experience severe mitigation, the reduction of
Nationally Determined impacts from climate change but have greenhouse gases.
Contributions (INDCs). contributed very little to the greenhouse gases
x At the Copenhagen summit, it was responsible for the warming and its effects.
agreed that from 2020, rich x This is a very important issue for the least
countries would provide a developed countries and for small islands,
minimum of $100 billion each year which are already experiencing the brunt of
to poor and developing countries. sea level rise. But there was little progress on
There is little sign that these funds the funds that could be used to support L&D.
will be available.

BACKGROUND AND MANDATES OF PREVIOUS COPS

COP21 COP 22 COP 23

The COP decided to convene a The President of COP22 and The Presidencies of COP22 and COP23 presented
facilitative dialogue among Parties the Incoming-President of the outcome of their consultations on the
in 2018 to take stock of the COP23 were mandated to dialogue and on this basis, made available the
collective efforts of Parties in undertake inclusive and approach to the dialogue to all Parties.
relation to progress towards the transparent consultations The COP welcomed with appreciation the design
long-term goal of the Paris with Parties on the of the 2018 facilitative dialogue, known as the
Agreement and to inform the organization of the 2018 Talanoa Dialogue, and launched the dialogue.
preparation of nationally facilitative dialogue.
determined contributions.

ABOUT TALANOA DIALOGUE

x Talanoa is a traditional word used in Fiji and across the Pacific to reflect a process of inclusive,
participatory and transparent dialogue.

x The purpose of Talanoa is to share stories, build empathy and to make wise decisions for the collective
Meaning
good. The process of Talanoa involves the sharing of ideas, skills and experience through storytelling.

x Similarly, in the context of the international climate change process, the Talanoa Dialogue will invite
participants to share stories to find solutions for the global common good.

The consultative dialogue will check progress, reaffirm the goals of the Paris Agreement and aim to find
Aim solutions to how countries can increase their ambition now and in the next round of their national climate
action plans, officially termed ‘Nationally Determined Contributions’ (NDCs).

FOCUS | June 2018 | RAU’S IAS 78

Download from:- www.UPSCPDF.com


https://t.me/UPSC_PDF www.UPSCPDF.com https://t.me/UPSC_PDF

Geography, Environment, Biodiversity & Disaster


Management

A report by the International Union for Conservation of


LAKE VICTORIA: SPECIES Nature (IUCN) titled ‘Freshwater biodiversity in the Lake
Victoria Basin’, assessed the global extinction risk of more
UNDER THREAT than 650 freshwater species, including 204 species of fish,
#Biodiversity #Conservation molluscs, dragonflies, crabs, shrimps and aquatic plants
native to the Lake Victoria Basin, in East Africa.

ABOUT THE REPORT

x Cited the example of the African Lungfish (Protopterus aethiopicus).


x The fish is declining in the lake basin largely due to overfishing, poor fishing practices and
environmental degradation as wetlands were being converted to agricultural land.
Example
x Its obliteration could be a disaster as the fish is considered a delicacy by some local communities
cited
and is an important local medicinal product, used to boost the immune system and treat
alcoholism.
x It is also traded at the market, making it important to the local economy.

x Pollution from industrial and agricultural sources, over-harvesting of resources and land
clearance.
x Invasive species present in the lake pose a threat to native biodiversity in the basin, affecting 31
Threats to per cent of all species and 73 per cent of threatened species.
lake’s 9 The invasive species - purple flowered Water Hyacinth (Eichhornia crassipes) was accidentally
Biodiversity introduced to Lake Victoria from South America in the 1980s, and at its peak covered close to 10
per cent of the lake surface.
9 It reduces the oxygen and nutrient availability in the water column, which negatively affects native
biodiversity.

ABOUT LAKE VICTORIA

x Lake Victoria is Africa's largest lake by area,


the world's largest tropical lake, and the
world's second largest fresh water lake by
surface area, after Lake Superior in North
America.
x It is situated along the equator and
borders the countries of Uganda, Kenya,
and Tanzania.
x The Kagera River is the largest river flowing
into this lake.
x Two rivers flow out of the lake. They are the
White Nile (called the "Victoria Nile" where it
leaves the lake), and the Katonga River.
x John Hanning Speke was the first European
to sight the lake. He named the lake after the
queen of England at the time, Queen
Victoria.

FOCUS | June 2018 | RAU’S IAS 79

Download from:- www.UPSCPDF.com


https://t.me/UPSC_PDF www.UPSCPDF.com https://t.me/UPSC_PDF

Geography, Environment, Biodiversity & Disaster


Management

Six tigers are being relocated to the Satkosia Tiger Reserve in

TIGERS: FIRST Odisha’s Angul district from Kanha National Park in Madhya
Pradesh as part of the State government’s plan to revive Tiger
INTER-STATE population in the protected forest. This is India’s first inter-state
relocation.
TRANSFER Before this, Tigers have been relocated in India to repopulate dying
#Biodiversity #Conservation reserves of Sariska (Rajasthan) in 2008 and Panna (Madhya
Pradesh) in 2010, but they were all within the state’s borders.

SATKOSIA TIGER RESERVE

Location x It is situated in the Angul district of Odisha.


x The tiger reserve is located in the Eastern Ghats moist deciduous forests ecoregion..

Flora The major plant communities are mixed deciduous forests including Sal (Shorea robusta), and
riverine forest.

River The Mahandi River passes through this Tiger Reserve.

Unique Satkosia is the meeting point of two bio-geographic regions of India; the Deccan Peninsula and the
Aspect Eastern Ghats, contributing to immense biodiversity.

KANHA NATIONAL PARK

x It is located in Maikal range of Satpuras in Madhya Pradesh, the heart of India that forms the
central Indian highlands.
Location
x The present-day Kanha area is divided into two sanctuaries, Hallon and Banjar.
x It the largest national park in Madhya Pradesh.

x The lowland forest is a mixture of sal (Shorea robusta) and other mixed forest trees,
interspersed with meadows.
Flora
x The highland forests are tropical moist dry deciduous type and of a completely different
nature with bamboo (Dendrocalamus strictus) on slopes.

x Members of the Baiga tribe, a semi-nomadic tribe, lived in 28 villages within the Kanha National
Relocation of
Park until 1968, when they were relocated.
Baiga tribe
x The relocation was part of an effort to maintain a critical tiger habitat.

Unique Aspect The forest is depicted in the famous novel by Rudyard Kipling, The Jungle Book.

BLACK PANTHER The wildlife wing of Odisha’s forest and Environment Department
have recorded the presence of black panthers in a forest in
#Biodiversity #Conservation Sundargarh district.

ABOUT BLACK PANTHER

x Black panther or melanistic leopard is a colour variant of the Indian leopard.


x Black panthers in Asia and Africa are leopards (Panthera pardus), and those in the Americas are black jaguars
(Panthera onca).
x It is as shy as a normal leopard and very difficult to detect.

FOCUS | June 2018 | RAU’S IAS 80

Download from:- www.UPSCPDF.com


https://t.me/UPSC_PDF www.UPSCPDF.com https://t.me/UPSC_PDF

Geography, Environment, Biodiversity & Disaster


Management

Distribution in India:
x Kerala (Periyar Tiger Reserve); Karnataka (Bhadra Tiger Reserve,
Dandeli-Anshi Tiger Reserve and Kabini Wildlife Sanctuary);
Chhattisgarh (Achanakmar Tiger Reserve); Maharashtra (Satara); Goa
(Mhadei Wildlife Sanctuary); Tamil Nadu (Mudumalai Tiger Reserve);
Assam and Arunachal Pradesh.
The recent sighting makes Odisha the ninth State in India where the
elusive and rare big cat has been seen.

One out of every three bites of food we eat is a result of pollinators


like honey bees. Honey bees are so important that farmers often
PROTECTING BEES have bee-hives transported and then placed on their farm to
#Biodiversity #Conservation provide pollination for their crops. In this regard, the UN’s Food and
Agricultural Organisation (FAO) and the European Union (EU) have
#Food Security called for a global action to protect pollinators, and bees in
particular, which are crucial for ensuring food security.

CONTRIBUTION OF HONEY BEES

THINGS TO UNDERSTAND

Importance x Pollinators such as bees, birds, bats, butterflies and beetles are responsible for most of the crops
of and food that we eat. Thus, they provide food security to the masses.
pollinators x For this the role of pollinators becomes even more important.

x Bees help pollinate 90 percent of the world's major crops, but in recent years many have been
dying off from "colony collapse disorder", a mysterious phenomenon blamed partly on
Threats to
pesticides.
Bees
x The UN has warned that 40 percent of invertebrate pollinators -- particularly bees and butterflies -
- risk global extinction.

FOCUS | June 2018 | RAU’S IAS 81

Download from:- www.UPSCPDF.com


https://t.me/UPSC_PDF www.UPSCPDF.com https://t.me/UPSC_PDF

Geography, Environment, Biodiversity & Disaster


Management

x Colony collapse disorder (CCD) is the phenomenon that occurs when the majority of worker
bees in a colony disappear and leave behind a queen, plenty of food and a few nurse bees to
care for the remaining immature bees.
Colony x Such disappearances have occurred throughout the history of apiculture, and were known by
collapse various names -disappearing disease, spring dwindle, May disease, autumn collapse, and fall
disorder dwindle disease.
x The syndrome was renamed colony collapse disorder in late 2006 in conjunction with a drastic
rise in the number of disappearances of western honey bee (Apis mellifera) colonies in North
America.

x The UN has named May 20th as World Bee Day.


x The day marks the birthday of Anton Jansa (1734-1773), a Slovenian pioneer in modern
Global
beekeeping.
Action Plan
x Recently EU countries voted for a ban on the use of three neonicotinoid pesticides in fields,
allowing its use only in covered greenhouses where they cannot get into the environment.

PARTY POPPERS Birthday parties without Party Poppers…


The Central Pollution Control Board (CPCB) has banned party poppers,
BANNED a popular decorative used at parties and celebrations, including at IPL
#Pollution #Environment matches, on the grounds that the devices posed a health risk.

ABOUT PARTY POPPERS

x Party poppers work similar to the popping of a champagne


bottle. According to CPCB, they are harmful to health.
x It is observed that the plastic glittering material and the other
charge chemicals, which are generally low-intensity explosive,
often composed of red-phosphorous, potassium chlorate and
potassium perchlorate, are harmful to human health and
environment.
x They get mixed with food items and may cause severe eye
trauma or facial injuries.
x Explosion is through Armstrong Mixture comprising of red
phosphorus, potassium chlorate, potassium perchlorate and
Sulphur).

ABOUT CPCB

x It is a statutory organisation that was constituted in 1974 under


the Water (Prevention and Control of Pollution) Act, 1974.
x It also provides technical services to the Ministry of
Environment and Forests of the provisions of the Environment
(Protection) Act, 1986.
x Further, CPCB is entrusted with the powers and functions under
the Air (Prevention and Control of Pollution) Act, 1981.
x Aims

FOCUS | June 2018 | RAU’S IAS 82

Download from:- www.UPSCPDF.com


https://t.me/UPSC_PDF www.UPSCPDF.com https://t.me/UPSC_PDF

Geography, Environment, Biodiversity & Disaster


Management

9 To promote cleanliness of streams and wells in different areas.


9 To improve the quality of air and to prevent, control or abate air pollution in the country.

A phenomenon of glowing beaches in the dark has been observed


occasionally in Mumbai. Earlier, it was believed that this glow was due to
SEA SPARKLING industrial pollution.

AND GLOBAL On this, the researchers from the Indian National Centre for Ocean
Information Services (INCOIS)—a Ministry of Earth Sciences (MoES)
WARMING body—and the US’ National Oceanic and Atmospheric Administration
(NOAA) conducted a study on the probable causes of ‘Sea Sparkle’. The
#Climate Change study has found that Global Warming conditions may be responsible for
this phenomenon.

THINGS TO UNDERSTAND

x It is a common name for Noctiluca algae. It is a parasite and occurs in patches or ‘blooms’ in the

Sea Sparkle Northern Arabian Sea.

x They glow at night due bioluminescence, and have earned them the nickname ‘sea sparkle’.

x The patches of sparkle are a sign of decline of fish because they compete with fish for food and
choke their supply.
Impact-
x Noctiluca algae feeds on one of the most important planktonic organisms at the base of the fish-
Killing fishes
food chain, namely diatoms, and also excretes large amounts of ammonia, which is linked
with massive fish mortalities.

ABOUT INCOIS

x INCOIS is a unit of the Earth System Science Organization (ESSO), established as ESSO-INCOIS in 1999
under the Ministry of Earth Sciences (MoES).
What? x OBJECTIVES: To provide the best possible ocean information and advisory services to society,
industry, government agencies and the scientific community through sustained ocean observations
and constant improvements through systematic and focused research.

1. Potential Fishing Zone (PFZ): Identify the potential fishing zones to help the fishermen to get better
catch.

2. Tsunami Early Warning Centre (TEWS): To provide important tsunami advisories to the people
Major living in the coastal areas of the country.
activities 3. Ocean State Forecast (OSF): To predict the surface and sub-surface features of the Indian Ocean in
advance to plan and execute commercial activities safely.

4. Ocean Observation Group (OOG): To measure and monitor the surface temperature and salinity of
the upper 2000 meters of the ocean.

FOCUS | June 2018 | RAU’S IAS 83

Download from:- www.UPSCPDF.com


https://t.me/UPSC_PDF www.UPSCPDF.com https://t.me/UPSC_PDF

Geography, Environment, Biodiversity & Disaster


Management

A study by NASA has found that India’s freshwater stocks are


depleting due to improper human water use, climate change
INDIA’S FRESHWATER and natural cycles. The team of NASA used years of
STOCKS observations from the Gravity Recovery and Climate
Experiment (GRACE) spacecraft mission, a joint project of NASA
#Geography #Resources and the German Aerospace Center, to track global trends in
freshwater in 34 regions around the world.

x India is among the hotspots where overuse of water resources has caused a sharp decline in the
availability of freshwater.
x Areas in northern and eastern India are the worst affected by the overuse of water resources.
x In northern India, groundwater extraction for irrigation of wheat and rice crops has led to depletion,
Concerns
despite rainfall being normal throughout the period studied.
x Rice is not suitable for Punjab, Haryana, and West U.P. as it is depleting ground water resources.
x Marathwada ended as a region of man-made drought due to over-exploitation of water for growing
sugarcane.

Î ALSO READ: Time to shift focus from land to water productivity in farming, says NABARD
(Courtesy: The Hindu)

Indian agriculture needs to stop being “obsessed” with the land productivity and instead start worrying about water
productivity, says a report released by the National Bank for Agriculture and Rural Development (NABARD).
Titled Water Productivity Mapping of Major Indian Crops, the report is part of a research project with ICRIER, mapping a
water atlas for ten major crops — rice, wheat, maize, red gram or tur, chickpea or channa, sugarcane, cotton,
groundnut, rapeseed-mustard and potato. These together occupy more than 60% of the country’s gross cropped area.
Given that Indian agriculture uses almost 80% of all the country’s water resources, which are increasingly under stress,
changing the objective of agriculture development to increasing productivity per unit of water, especially irrigation
water, is crucial, says the report.

WATER GUZZLERS

The most stark differences between


land and water productivity are
seen in rice and sugarcane
cultivation, the report says. Punjab
reports the highest land
productivity for rice, producing four
tonnes per hectare. However, it
only produces 0.22 kg of rice for
every meter cube of irrigation
water. Jharkhand and Chhattisgarh,
on the other hand, produce 0.75
and 0.68 kg for the same amount of
water. However, low irrigation
coverage results in low land
productivity in these States.
Jharkhand has only 3% of its land
under irrigation.

FOCUS | June 2018 | RAU’S IAS 84

Download from:- www.UPSCPDF.com


https://t.me/UPSC_PDF www.UPSCPDF.com https://t.me/UPSC_PDF

Geography, Environment, Biodiversity & Disaster


Management

For sugarcane, another water-guzzling crop, Tamil Nadu reports the highest land productivity, producing 105 tonnes
per hectare. Karnataka, Maharashtra and Andhra Pradesh also have high rates of land productivity. However, all four
States in the water-stressed sub-tropical belt have an irrigation water productivity of less than 5 kg/m3. In fact, an
average of 40 rounds of irrigation are needed in Tamil Nadu. In the Gangetic Plain States of Bihar and Uttar Pradesh,
on the other hand, need five and eight rounds of irrigation respectively.
The report recommends that cropping patterns be re-aligned to water availability, using both demand and supply side
interventions. With water and power subsidies skewing cropping patterns, it also recommends reform in these areas,
with a shift from the price policy approach of heavily subsidising inputs to an income policy approach of directly giving
money farmers on per hectare basis. Prices will then be determined by market forces.

Viewpoint
PUSHING FOR GREEN GDP
# Sustainable development
(courtesy: The Hindu)

India has suffered a cost of $550 billion, due to air pollution, according to a World Bank report; the cost of externalities
such as water pollution and land degradation were possibly far higher. Through commodity exports, we effectively
transfer natural capital to our trade partners, raising the risk of desertification and land being degraded significantly.
Within a century, our food production could see a loss of 10-40% if these trends continue. So when we crow about GDP
growth, we should also consider the decline in natural capital in our national accounts.

ESTIMATION IS A CHALLENGE

The idea of having a national account for wealth accrued over a year is accepted in most major economies. Such
national accounts (GDP, net national product, gross savings) provide a measure of an economy’s performance and
form the basis for socio-economic policies, while highlighting the gap between potential and actual economic output.

GDP computations indicate the economic activity in a country, with rising GDP growth rate often leading to
international prestige. However, such estimates often exclude the variations in natural capital by assuming them to be
constant and indestructible. Such natural capital is often self-generating (water, clean air) but needs to be handled in a
sustainable way in order to avoid depletion.

Natural capital can cover entire ecosystems such as fisheries and forests, besides other hidden and overlooked
services — for example, the regeneration of soil, nitrogen fixation, nutrient recycling, pollination and the overall
hydrological cycle. Valuing such ecosystems can be challenging, with their market value often termed as zero. When
pollution happens, it is actually a depletion of our natural capital as, for example, acid rain damages forests and
industrial seepage affects water quality. In a modern economy, the challenge is to estimate such depreciation to
natural capital.

Consider groundwater. Most groundwater basins in India are subjected to unrestricted extraction until the ‘marginal
value of extracting water is less than the unit extraction cost’ — i.e. when the water table drops so low new borewells
are needed. Effectively, any rents due to groundwater depletion are dissipated indefinitely.

Now many economists have pushed for an “environmental Kuznets curve”, highlighting that the ‘relationship between
GDP per capita and the concentration of sulphur dioxide in the local air’ is an inverted U curve. Such a relationship
leads to the postulation that people from ‘developing countries can’t place a weight on natural environment’ and
should consider pollution as an acceptable side-effect of GDP growth.

FOCUS | June 2018 | RAU’S IAS 85

Download from:- www.UPSCPDF.com


https://t.me/UPSC_PDF www.UPSCPDF.com https://t.me/UPSC_PDF

Geography, Environment, Biodiversity & Disaster


Management

However, this inverted U curve is found primarily for local pollutants that lead to short-term damages (sulphur,
particulates) and not for pollutants that lead to long term and dispersed costs (carbon dioxide). In addition, the
inverted U curve hides systemic consequences of emissions. We are long past treating natural capital as a luxury. In
fact, it is a necessity.

While India might have a GDP of $2.65 trillion in nominal terms, it fails to take into account the externalities of such
economic growth. For example, India routinely suffers from high levels of air pollution that impose costs on local
transport, health and liveability in urban and rural areas. When economic growth leads to the destruction of forests,
wetlands and woodlands for agriculture, mining or even urban expansion, it is typically the poorest of the traditional
dwellers who suffer. Ecological collapse can soon come, examples being the Darfur region in Sudan and countries in
the Horn of Africa. All were subject to rapid socio-economic decline.

SOME ATTEMPTS

India has sought to unveil “green GDP” figures in the past. In 2009, the Centre announced that it would publish a “green
GDP” that would include the environmental costs of degrading and depleting our forests, grasslands and natural stock.
An expert programme, sponsored by the Ministry of Statistics and Programme Implementation, released a
Compendium of Environment Statistics 2013.

The group recommended that India shift to a system of measuring comprehensive national wealth, which includes
items such as human capital, capital equipment and natural capital. However, implementation of such well-founded
recommendations has been constrained by the lack of micro-level data on capital formation, particularly in a natural
context.

While the 12th Five Year Plan undertook groundwater resource mapping at the national level, a similar focus is
essential for data on land usage, forests and mineral wealth. ‘India’s current national accounts incorporate such
environmental considerations in a limited fashion’. GDP includes the value of: minerals extracted; timber, fuelwood
and non-timber forest products; natural growth of cultivated assets for some crops; and the output from dung
manure. In addition, ‘gross fixed capital formation contains output estimates from the improvement of land along with
irrigation works and flood control projects’. However, even in GDP estimates of timber value, there is significant under-
estimation — non-monetised goods and services provided from timber forests are not considered.

India should seek to publish “green GDP” figures that take into account depreciation of natural capital stock due to
economic exploitation and environmental degradation. This can follow the template provided by the UN’s System of
Environmental-Economic Accounting.

A few studies have tried to document the ecological services offered by natural capital in India. But we need a ‘greater
push for empirical studies of the potential value of such ecosystem services’. Adapting our national accounts in line
with this framework will help in incorporating the value of the environment in our growth while helping us to focus on
developing a feasible transition path to a green economy.

FOCUS | June 2018 | RAU’S IAS 86

Download from:- www.UPSCPDF.com


https://t.me/UPSC_PDF www.UPSCPDF.com https://t.me/UPSC_PDF

History, HERITAGE & Culture


# GS Paper (Prelims) & GS Paper I (Main)

Preservation of Prehistoric art is essential as it provides the


PREHISTORIC ROCK important link with our ancient forefathers who gave initial shape
to the emerging society. So, defacing of prehistoric art becomes a
PAINTINGS IN huge concern.

DANGER # ANCIENT INDIA Such defacement is becoming prominent at the prehistoric rock
paintings of Pandavulagutta as it can spoil the prehistoric rock.

ABOUT PREHISTORIC ROCK PAINTINGS OF PANDAVULAGUTTA

The prehistoric rock paintings of Pandavulagutta are located in Jayashankar Bhupalpally district of
Location Telangana.
These rock paintings trace the evolution of human knowledge.

Pandavulagutta is a Treasure trove of the following:

Historical 9 Painted rock shelters dating to 10000 BC-8000 BC,


information 9 An 8th century inscription of the Rashtrakuta period, and
9 Painted frescoes from the 12th century Kakatiya empire.

x The pre-historic rock paintings resemble those at Bhimbetka in Madhya Pradesh, with flora, fauna
and human figures seen in red ochre.
Comparison
x And in the painted frescos, the Kakatiya artists painted scenes from the Mahabharata and of the
elephant-headed Ganesha.

Î ADDITIONAL INFORMATION: ABOUT RASHTRAKUTA

x The dynasty was founded by Dantidurga in 735 A.D. who established his capital at Manyakhet or
Malkhed near modern Sholapur (Maharashtra).
x The Rashtrakutas soon dominated the entire area of northern Maharashtra.
Origin
x They also engaged with the Pratiharas for the over-lordship of Gujarat and Malwa.
x Rashtrakutas also fought constantly against the Eastern Chalukyas of Vengi (Andhra Pradesh) and
in the south against the Pallavas of Kanchi and the Pandyas of Madurai.

x The greatest Rashtrakutas ruler was Amoghavarsha (814-878).


x He was himself an author and is credited with writing the first Kannada book on poetics.
x He was a great builder and is said to have built the capital city Manyakhet.
x His grandson Indra III re-established the empire. He was the most powerful ruler of his times.
Important x According to Al-Masudi who visited India at that time, Rashtrakutas king Balhara or Vallabharaja,
rulers was the greatest king in India and most of the rulers accepted his suzerainty and respected his
envoys.
x Krishna III was the last in a line of brilliant rulers. He was engaged in a struggle against the
Paramaras of Malwa and the eastern Chalukyas of Vengi.
x After his death, all his opponents united against his successor.

FOCUS | June 2018 | RAU’S IAS 87

Download from:- www.UPSCPDF.com


https://t.me/UPSC_PDF www.UPSCPDF.com https://t.me/UPSC_PDF

History, Heritage & Culture

x The Rashtrakutas capital, Malkhed was sacked and burned in 972. This marked the end of the
Rashtrakutas empire.

x The Rashtrakutas rulers were tolerant in their religious views and patronized not only Shaivism
and Vaishnavism but Jainism as well.
x The famous rock-cut temple of Shiva at Ellora was built by one of the Rashtrakutas kings,
Krishna I in the ninth century.
x His successor Amoghavarsha was a Jain but he also patronized other faiths.
Contribution
x The Rashtrakutas allowed Muslims traders to settle and permitted Islam to be preached in their
to Art and
dominions.
culture
x This tolerant policy helped to promote foreign trade which enriched the Rashtrakutas.
x The Rashtrakutas kings were great patrons of arts and letters.
x In their courts we find not only Sanskrit scholars, but also poets and others, who wrote in the
apabhramsha.
x The great apabhramsha poet Svayamhu lived at the Rashtrakutas’ court.

FOCUS | June 2018 | RAU’S IAS 88

Download from:- www.UPSCPDF.com


https://t.me/UPSC_PDF www.UPSCPDF.com https://t.me/UPSC_PDF

History, Heritage & Culture

Î ADDITIONAL INFORMATION : ABOUT KAKATIYAS

x The 12th and the 13th centuries saw the emergence of the Kakatiyas.
Emergence x They were at first the feudatories of the Western Chalukyas of Kalyana.
x Initially, they ruled over a small territory near Warangal (Telangana).

x Prola II, a ruler of this dynasty, ruled in mid twelfth century.


Expansion
x He extended his sway to the south and declared his independence.

x His successor Rudra pushed the kingdom to the north up to the Godavari delta.
Rule of Rudra x He built a fort at Warangal to serve as a second capital and faced the invasions of the Yadavas of
Devagiri.

x The next ruler Mahadeva extended the kingdom to the coastal area.
Ganapati, the
x In A.D. 1199, Ganapati succeeded him.
influential
ruler x He was the greatest of the Kakatiyas and the first after the Satavahanas to bring the entire Telugu
area under one rule.

x Ganapati Deva’s daughter Rudramba succeeded him in A.D.1262 and carried on the
The female
administration.
ruler
x She suppressed the internal rebellions and external invasions with the help of loyal subordinates.

x Prataparudra succeeded his grandmother Rudramba in A.D.1295 and ruled till A.D.1323.
x He pushed the western border of his kingdom up to Raichur (Karnataka).
Rule of
x He introduced many administrative reforms.
Prataparudra
x He divided the kingdom into 75 Nayakships, which was later adopted and developed by the
Rayas of Vijayanagara.

x In time of Prataparudra, the territory constituting Andhra Pradesh had the first experience of
invasion from north.
x In A.D.1303, the Delhi Sultan Ala-ud-din Khilji sent an army to the kingdom. But Prataparudra
defeated them.
x After that, another army under Malik Kafur invaded Warangal, Prataparudra yielded and agreed
Invasion from to pay a large tribute.
Delhi x When Ala-ud-din Khilji died, Prataparudra withheld the tribute. It provoked another invasion of
Sultanate the sultans.
x In another army attack, Ghiaz-ud-din Tughlaq sent a large army under Ulugh Khan to conquer
the area but owing to internal dissensions he called off the siege and returned to Delhi.
x Within a short period, he came back with a much bigger army. In this attack, Prataparudra died.
x Thus ended the Kakatiya rule, opening the gates of the Telugu land to anarchy and confusion
yielding place to an alien ruler.

Golden The Kakatiya period was rightly called the brightest period of the Telugu history. The entire Telugu
period speaking area was under the kings who spoke Telugu and encouraged Telugu.

The most famous strongholds in the Kakatiya period were:


x Giridurgas -Anumakonda and Gandikota
Strongholds x Vanadurgas- Kandur and Narayanavanam
x Jaladurgas -Divi and Kolanu
x Sthaladurgas -Warangal and Dharanikota

FOCUS | June 2018 | RAU’S IAS 89

Download from:- www.UPSCPDF.com


https://t.me/UPSC_PDF www.UPSCPDF.com https://t.me/UPSC_PDF

History, Heritage & Culture

KAKATIYAS ART AND CULTURE

x Saivism was the religion of the masses but intellectuals favoured revival of Vedic rituals.
x They sought to reconcile the Vaishnavites and the Saivites through the worship of Harihara
Religion and (Harihara is the fused representation of Vishnu (Hari) and Shiva (Hara) from the Hindu tradition).
Literature x Tikkana Somayaji wrote some cantos of the Mahabharata.
x It is notable that Sanskrit received encouragement at the hands of the Kakatiyas.
x Prataparudra was himself a writer and he encouraged other literature.

x The Kakatiya dynasty expressed itself best through religious art.


x The Kakatiya temples dedicated mostly to Siva.
x They reveal in their construction a blending of the styles of North India and South India which
influenced the political life of the Deccan.
x The most important of these temples are those at Palampeta, Hanamkonda (Telangana) and the
incomplete one in the Warangal fort.
x The temple at Palampeta is described as the `brightest gem in the galaxy of Medieval Deccan
Architecture temple architecture’.
x It was constructed by Recherla Rudra, a general of Kakatiya Ganapati.
x The figures in the temple are of a heterogeneous character comprising: gods, goddesses,
warriors, acrobats, musicians and mithuna pairs in abnormal attitudes and dancing girls.
x The Thousand-Pillared Temple at Hanamkonda was built by the Kakatiya king Rudra.
x This temple, dedicated to Siva, Vishnu and Surya, is star-shaped.
x After the fall of Kakatiyas, uncertainty prevailed over the region. Several small kingdoms came
into existence, Musunuri Nayakas occupied Warangal from Muslims.

‘VACHANAS’ USED In the recent Karnataka elections, major political parties used

FOR VOTES Vachana to garner public support in their favour.


So, let us understand about the basic facts of Vachana.
# MEDIEVAL INDIA
ABOUT VACHANA

x Vachana literature is a form of writing in Kannada that evolved in the 12th century as a part of the
Veerashaiva movement.
x The word Vachana means ‘Speech’. It also means a verbal promise.
x Vachanas of the Sharanas (Sharana’ means a person who is committed or dedicated to Shiva as the
Meaning supreme godhead) are the means to purify one in word, action, and vision.
x Sharanas have chronicled their experiences and the path towards divinity in this unique form of
literature.
x Vachanas are composed in simple language. And they carry profound philosophy and thought
provoking ideas.

x Vachana prescribe to a format-free writing, not imposing any grammatical or literary


restrictions on the authors. Yet, Vachanas possess musical and emotional quality.
Features
x Sharana convey complex concepts in simple Vachanas. Often, they draw examples from familiar day-
to-day experiences and use them as similes and metaphors to drive home profound ideas.

FOCUS | June 2018 | RAU’S IAS 90

Download from:- www.UPSCPDF.com


https://t.me/UPSC_PDF www.UPSCPDF.com https://t.me/UPSC_PDF

History, Heritage & Culture

x Vachanas cover philosophical, cosmological, scientific, and theological aspects with the same ease as
it handles the interpersonal relationships, emotional turmoil, instincts, and shortcomings of human
beings.
x It takes an extremely scientific and logical approach in dealing with the cosmology, universe, and its
creation.
x Vachanas were primarily targeted at the common person and sought to demystify God, as large
sections of society had been deprived of access to the texts.

x Each Sharana has used a penname that seems be drawn from his/her first divine connection.
x These divine insignia are incorporated towards the end of the Vachana clearly identifying the author.
For instance, Guru Basavanna has used Kudalasagamadeva as his penname drawing from the place
Using of
where he first realized his divine mandate.
penname
x Sharanas have also classified and categorized the Vachanas of major Vachanakaras into step-by-step
levels, giving us a framework to absorb the concepts of the path of Basavanna, the founder of
Lingayat movement.

SADHARAN
Recently, Sadharan Brahmo Samaj (SBS) has entered into a legal battle with
BRAHMO SAMAJ the West Bengal government due to:

# MODERN INDIA

x The state government’s action to dissolve the governing bodies of eight colleges in Kolkata run by the Sadharan
Brahmo Samaj;
x The action was taken following the decision of the State’s Minority Affairs and Madrasah Education Department to
not to grant the SBS the status of a minority religion in an order;
x The order argued that since SBS is not a separate minority religion. So, the related colleges administered by it
should be treated as non-minority Government-aided Colleges.

ABOUT BRAHMO SAMAJ AND SADHARAN BRAHMO SAMAJ

In 1828, Raja Ram Mohan Roy founded a new religious society, the Brahma Sabha, later known as the
Origin
Brahmo Samaj.

x Its purpose was to purify Hinduism and to preach monotheism or belief in one God.

x The new society was to be based on the twin pillars of reason and the Vedas and Upanishads.

x It was also to incorporate the teachings of other religions.

x The Brahmo Samaj laid emphasis on human dignity, opposed idolatry and criticized such social
evils as the practice of Sati.
Objectives x No image, statue or sculpture, carving, painting, picture, portrait or the likeness of anything was to
be allowed in the Samaj building.

x There was no place for priesthood in the Samaj nor sacrifices of any kind were allowed.

x The worship was performed through prayers and meditation and readings from the Upanishads.

x Great emphasis was laid on promotion of charity, morality, piety, benevolence, virtue and
strengthening of the bonds of union between men of all religious persuasions and creeds.

FOCUS | June 2018 | RAU’S IAS 91

Download from:- www.UPSCPDF.com


https://t.me/UPSC_PDF www.UPSCPDF.com https://t.me/UPSC_PDF

History, Heritage & Culture

x The early death of Ram Mohan Roy in 1833 left the Brahmo Samaj without the guiding soul and
Emergence of a steady decline set in.
Debendranath x After that, it was left to Debendranath Tagore and he gave the theist movement a definite form
Tagore and shape. Earlier, he headed the Tattvabodhini Sabha which was engaged in search of
spiritual truth.

x Another person important for the Samaj was Keshab Chandra Sen.

x He joined the Brahmo Samaj in 1858 and infused new life into the Brahmo Samaj.
Role of Keshab
x The energy, vigour and persuasive speaking of Keshab popularised the movement and the
Chandra Sen
branches of Samaj were opened outside Bengal in UP, Punjab, Bombay, Madras and other
towns.

x However, Keshab’s liberal and cosmopolitan outlook brought a split in the Samaj.

x Under Keshab’s influence the Samaj began to cut itself from Hindu beliefs; henceforth religious
scriptures of every sect and every people including the Christians, Muslims, Parsis began to be
read in the Brahmo Samaj meetings.
Dismissal of
x On the social front Keshab spoke against the caste system and even advocated inter-caste
Keshab
marriages.
Chandra Sen
x To Debendranath, these developments looked too radical and by virtue of his position as the
sole trustee of the Samaj dismissed Keshab from the office of the Acharya in 1865.

x Keshab and his followers left the parent body in 1866 and formed the Brahmo Samaj of India.

x Debendranath’s Samaj henceforth came to known as the Adi Brahmo Samaj.

x A further split in Keshab’s Brahmo Samaj of India came in 1878. Some close disciples of Keshab
began to regard Keshab as an incarnation. This was not liked by his progressive followers.
Further, Keshab began to be accused of authoritarianism.

x Keshab Chandra had advocated a minimum age for marriage of Brahmos, but did not follow
his own precepts.

x In 1878, Keshab Chandra married his thirteen-year-old daughter with minor Hindu Maharaja of
Split and
Cooch Bihar with all the orthodox Hindu ceremonials.
formation
x He justified his action on the plea that such was the will of God and that he had acted on
Sadharan
intuition.
Brahmo Samaj
x Most of Keshab’s followers felt disgusted and set up a new organisation called the Sadharan
Brahmo Samaj in 1878.

x Ananda Mohan Bose, Shibchandra Deb and Umeshchandra Datta were the key leaders of
Sadharan Brahmo Samaj.

x It is notable that the Sadharan Brahmo Samaj was funded by Rabindranath Tagore’s father in
the initial years.

LOKAMANYA TILAK
# PERSONALITY # MODERN INDIA

FOCUS | June 2018 | RAU’S IAS 92

Download from:- www.UPSCPDF.com


https://t.me/UPSC_PDF www.UPSCPDF.com https://t.me/UPSC_PDF

History, Heritage & Culture

A controversy was generated when a


school textbook used by private English-
medium schools affiliated to the
Rajasthan Board of Secondary Education
(RBSE) described freedom fighter Bal
Gangadhar Tilak as the ‘father of
terrorism’.
In light of this controversy and to clear
the air around the freedom fighter, let
us understand about the contributions
of Lokamanya Tilak.

ABOUT TILAK

x Tilak was born in a Maratha Brahmin family at Ratnagiri.


x After taking a degree in Law in 1879, Tilak planned in collaboration with Agarkar the
Introduction establishment of institutions to impart cheap education to the people.
x Tilak was the first person to openly declare the demand for Swaraj. "Swaraj is my birthight", he
said, "and I shall have it."

x When Lord Curzon, viceroy of India, partitioned Bengal in 1905, Tilak strongly supported the
Bengali demand for the annulment of the partition.
Role during the x He also advocated a boycott of British goods, which soon became a movement that swept the
Bengal nation.
partition x The forms of political action initiated by Tilak—the boycotting of goods and passive
resistance—were later adopted by Mahatma Gandhi in his program of nonviolent
noncooperation with the British.

x Tilak was the first Congress leader to suffer several terms of imprisonments for the sake of the
country, an example emulated by Gandhiji and others.
x For criticising in strong language the treatment meted out to the Maharaja of Kolhapur, the
Government tried Tilak and sentenced him in 1882.
x In 1897, he was charged with instigating the murders of Mr. Rand and Lt. Ayerst and sent to
jail.
9 Mr. Rand, the ‘Plague Commissioners’ and Lieutenant Ayerst were shot dead by the Chapekar
Imprisonment
Brothers.
of Tilak
9 It was alleged that Mr. Rand and Lieutenant Ayerst had wrongfully oppressed the people in
connection with the measures to prevent the plague epidemic. It was also alleged that these
officers misbehaved with the plague patients brutally instead of rendering any duty.
9 Probably, it was the first political murder in modern India.
x Again in 1908, Tilak for commenting on the Muzaffarpur Bomb case was tried for sedition
and sent to Mandalay jail for six years. This bomb case was related to an attempt on life of
Chief Presidency Magistrate, Douglas Kingsford by Khudiram Bose and Prafulla Chaki.

Role in Indian x After joining the Indian National Congress in 1891, he moved an Arms Act resolution at its
National annual conference for modifying the resolutions governing gun prohibition and Indians’
Congress involvement in the military.

FOCUS | June 2018 | RAU’S IAS 93

Download from:- www.UPSCPDF.com


https://t.me/UPSC_PDF www.UPSCPDF.com https://t.me/UPSC_PDF

History, Heritage & Culture

x In 1916, he concluded the Lucknow Pact with Mohammed Ali Jinnah, which provided for
Hindu-Muslim unity in the nationalist struggle.
x It was mostly due to his efforts and those of his associates that the Congress resolution at
Calcutta (1906) demanding Self-Government, Boycott and National Education was passed.
x Due to his efforts, the Congress session at Nagpur in 1920 demanded Swaraj and
authorized the use of 'all peaceful and legitimate means for its attainment.

x Tilak believed in service and sacrifice and had the courage to defy the authority of the
Government.
x It was mostly due to his efforts that the Congress from being the admirer of the Government
Chirol’s turned into a great critic of the British Empire.
allegation x The Anglo-Indian bureaucracy thought Tilak a seditionist and Sir Valentine Chirol called him
the 'Father of Unrest in India.'
x Tilak sued Chirol for defamation and went to England in this connection. Although the case was
lost, it opened his eyes to the real character of British rule in India.

x Tilak was not satisfied with the type of responsible government prevalent in the self-governing
Dominions within the Empire. So, Tilak setup the Home Rule League at the Bombay Provincial
conference held at Belgaum in April 1916.
Home Rule
x The Tilak's League was to work in Maharashtra (excluding Bombay city), Karnataka, the
League
central Provinces and Berar.
x Tilak promoted the Home Rule campaign with a tour of Maharashtra and through his lectures
clarified and popularized the demand for Home Rule.

x Tilak has been described as an Extremist in politics but a Moderate in matters of social reform.
x He did oppose the Age of Consent Bill, not because he did not see the necessity of social
Opinion about
reform but because he believed that a foreign government should not legislate about social
social reform
reform.
x He contended that social changes could better be brought about by educating public opinion.

x Tilak was a scholar of eminence. His books ‘The Arctic Home of the Vedas’ and ‘Gita
Rahasya’ are a testimony to his scholarship.
x As Tilak believed in the power of education to serve the masses. He became the founder-editor
of the journals, Mahratta in English and kesari in Marathi.
Books and
x To foster Indians’ sense of pride, he began celebrating the Ganapati Pooja festival and the
other
Shivaji festival.
contributions
x In 1890, the Poona New English School was founded. He was also associated in the formation
of the Deccan Educational Society and the foundation of the Fergusson Collage, Poona.
x In tributes, Gandhi called him “the Maker of Modern India,” and Jawaharlal Nehru described
him as “the Father of the Indian Revolution.”

MAULANA AZAD
# PERSONALITY# MODERN INDIA

FOCUS | June 2018 | RAU’S IAS 94

Download from:- www.UPSCPDF.com


https://t.me/UPSC_PDF www.UPSCPDF.com https://t.me/UPSC_PDF

History, Heritage & Culture

On the 130th birth anniversary of Maulana Abul Kalam


Azad, he was remembered across the country.
He was born in Mecca where his forefathers had settled
during the Revolt of 1857. He came with his parents to
India and settled down in Calcutta.
He received a traditional Islamic education at home from
his father and other Islamic scholars rather than at a
madrasah (Islamic school).
However, he was also influenced by the Sir Sayyid
Ahmad Khan and he learned English without his father’s
knowledge.

CONTRIBUTIONS OF MAULANA AZAD

x Azad was active in journalism and in 1912 he began publishing a weekly Urdu-language
newspaper in Calcutta, Al-Hilal.
Publication of
x The paper quickly became highly influential in the Muslim community for its anti-British stance,
Newspapers
notably for its criticism of Indian Muslims who were loyal to the British.
x He also published Al Balagh in 1915.

x He joined the Congress in 1920, when Gandhi launched the non-cooperation agitation in
conjunction with the Khilafat movement, and became its president in 1923.
x He committed to building bridges between Hindus and Muslims.
x He also opposed separate electorates, which he attributed to the British policy of divide and
rule.
x He was particularly active in the Khilafat movement (1920–24), which defended the Ottoman
Involvement sultan as the caliph (the head of the worldwide Muslim community). And even briefly enlisted
in Indian the support of Mohandas K. Gandhi.
National x Azad and Gandhi became close and Azad was involved in Gandhi’s various civil-disobedience
Congress campaigns, including the Salt March (1930).
x Azad was president of the Congress Party in 1923 and again in 1940–46—though the party was
largely inactive during much of his second term, since nearly all of its leadership was in prison.
x He represented Indian National Congress in Shimla Conference, 1945. It was convened to
discuss Wavell plan.
x It is notable that Azad was the youngest and the longest serving President of the Congress
during the freedom struggle.

x After the Second World War, Azad was one of the Indian leaders who negotiated for Indian
independence with the British.
x He tirelessly advocated for a single India that would embrace both Hindus and Muslims while
strongly opposing the partition of British India into independent India and Pakistan.
Against
x He later blamed both Congress Party leaders and Mohammed Ali Jinnah, the founder of
partition
Pakistan, for the ultimate division of the subcontinent.
x After the two separate countries were established, he served as India’s first Education
Minister from 1947 until his death.
x His autobiography, India Wins Freedom, was published posthumously.

FOCUS | June 2018 | RAU’S IAS 95

Download from:- www.UPSCPDF.com


https://t.me/UPSC_PDF www.UPSCPDF.com https://t.me/UPSC_PDF

History, Heritage & Culture

x His many noteworthy contributions in free India include setting up of -


9 the University and Secondary Education Commission;
9 the University Grants Commission;
Other 9 reorganisation of the All India Council for Technical Education;
contributions 9 developing the Indian Institute of Science; and
9 setting up the Kharagpur Institute of Technology and many scientific research laboratories.
x In 1992, decades after his death, Azad was awarded the Bharat Ratna, India’s highest civilian
award.

EXCERPTS FROM HISTORY

THE TURNING POINT IN 1932


# MODERN INDIA
(Courtesy: The Hindu)

In the current climate of Dalit assertion, it is relevant to revisit the era when the Dalits asserted their clout in Indian
politics for the first time.
This happened under the charismatic B.R. Ambedkar in the 1920s and 1930s and almost succeeded in gaining separate
representation for the “Depressed Classes”, as they were euphemistically termed in British legalese, in the central and
provincial legislatures.
It is equally important to speculate what it would have done to the Hindu-Muslim equation, and therefore the prospect
of Partition, if Ambedkar had succeeded in reaching his goal. This article attempts to answer this question.

Deep insecurity
The politics of the Muslim elite and all politics in the run-up to Indian independence was elite politics — was driven
primarily by a sense of deep insecurity. This tendency was accentuated when it became increasingly clear from the
1920s that the British would have to leave India sooner or later.
The Muslim sense of insecurity was rooted in many factors relating to history, demography, lack of progress in
English education and, probably most importantly, the shift of the centre of gravity of Indian politics from the heartland
of northern and central India, where much of the Muslim elite, the ashraf, were concentrated, to Calcutta, Madras and
Bombay and their hinterlands dominated by the new English-educated, predominantly Hindu elite.
Partition was the outcome in substantial part of this insecurity although other factors, including aggressive forms of
Hindu nationalism advocated by the likes of V.D. Savarkar, K.B. Hedgewar and M.S. Golwalkar contributed to it in
considerable measure as well.
The soft Hindutva of many stalwarts within the Congress, including Bal Gangadhar Tilak, Lala Lajpat Rai and
Vallabhbhai Patel, added to Muslim concerns, as did Jawaharlal Nehru’s disdain for what he termed the Muslim
League’s “communal politics”.
The Muslim elite’s anxieties were centred largely on the demographic and, therefore, political disparity between
Muslims and Hindus and the domination of India’s political and economic landscapes by the upper caste Hindu elite.
Political parity between caste Hindus and Muslims, therefore, remained the primary goal of the Muslim League
through much of its existence as a political party.

FOCUS | June 2018 | RAU’S IAS 96

Download from:- www.UPSCPDF.com


https://t.me/UPSC_PDF www.UPSCPDF.com https://t.me/UPSC_PDF

History, Heritage & Culture

A landmark announcement in 1932 by British Prime Minister Ramsay Macdonald came close to removing these
anxieties. He accepted Ambedkar’s demand that it was necessary for the “Depressed Classes” to have separate
representation in the central and provincial legislatures in order to protect their interests which ran counter to the
interests of the dominant Hindu castes who also hogged most of the seats in the legislatures.
This was the case because elections in British India were held under a very restricted franchise based primarily on
property, income and educational qualifications. Only about 13% of the population had the right to vote. The Dalits
lagged far behind caste Hindus in all the three qualifications that determined the right to vote and, therefore, were not
only under-represented but also represented by members of those castes that were opposed to according equality to
them.
The British Prime Minister accepted Ambedkar’s arguments and awarded separate electorates to the Depressed
Classes on lines similar to those for Muslims.
The Muslim League, recognising the import of this decision in that it had the potential to weaken the caste Hindu
leadership’s hold on the entirety of the Hindu population, readily accepted the award.
However, to everyone’s consternation, Mahatma Gandhi, who was seen as a leading advocate for Dalit rights, went on
a fast unto death to persuade the British to repeal the award.
To him, the award was a ploy to divide Hindu society that he found unacceptable. Initially, Ambedkar refused to bend
to Gandhi’s coercive fast.
However, when it became clear that Gandhi’s life depended on Ambedkar’s decision, Ambedkar was forced to give up
his demand in return for reserved seats for Dalits but on the basis of a single Hindu electorate. Years later, Ambedkar
came to bitterly regret his decision.
Equally, if not more important, Gandhi’s extreme reaction to the award of separate electorates to the Dalits convinced
the Muslim elite that Gandhi and the Congress were bent on not giving Muslims their due share in the future political
arrangement in India.
Their reasoning was simple: if implemented, the Communal Award, as it was known, would have led to parity between
caste Hindu and Muslim representatives in the legislatures, and the Dalits, who the Muslim elite did not find
threatening and who they saw as potential allies against caste Hindus because of the common fear of upper caste
domination, would have held the balance. This would have precluded the need for demanding Partition and in all
probability kept India united.
This argument sounds plausible because for most of the 1940s Pakistan was but a bargaining counter for Jinnah and
the Muslim League. This is why Jinnah accepted the Cabinet Mission Plan of 1946, which would have kept India united
in the form of a loose federation, but which Nehru torpedoed. With the failure of this last-ditch British effort to keep
India united, Jinnah was hoisted with his own petard and forced to accept what he called a “mutilated, moth-eaten
Pakistan.”

More potent than 1937


It appears in hindsight that Mahatma Gandhi’s stance on the Communal Award — even more than Nehru’s refusal to
enter into a coalition with the Muslim League in the United Provinces in 1937, which is widely seen as the turning point
in Muslim politics in favour of separatism — was responsible for increasing the Muslim leaders’ distrust of the
Congress that made Pakistan an attractive option for them.
One could plausibly argue that Gandhi’s rejection of the Communal Award sent the message to the Muslim leadership
that he and the Congress were more interested in promoting a monolithic Hindu bloc than in nurturing Hindu-Muslim
unity or providing justice for the Dalits in the form demanded by Ambedkar.
This increased their sense of insecurity and finally led to the demand for a separate state comprising the Muslim
majority provinces of British India. The rest is history.

FOCUS | June 2018 | RAU’S IAS 97

Download from:- www.UPSCPDF.com


https://t.me/UPSC_PDF www.UPSCPDF.com https://t.me/UPSC_PDF

SECURITY
# GS Paper III (Main)

Recently, the Ministry of Defence conducted a workshop on


AI IN NATIONAL Artificial Intelligence in National Security and Defence and
SECURITY & DEFENCE produced a ‘Listing of Use Cases’ and established an ‘AI Task
Force’. Let us understand the various listing of use cases for AI
# Internal Security # Technology in the field of national security and defence.

x Artificial Intelligence (AI) simply means the simulation of intelligent behaviour in computers wherein
AI is a sub-field of Machine Learning (ML).
x The basis for AI based applications essentially works on building profile that mimics normal
behaviour by learning from the data generated from a given use case.
Artificial
x Then using machine learning/ deep learning techniques, one can draw correlations among various
Intelligence
parameters in an automated way (from the huge data set) and can flag pattern and anomalous
behaviour for further analysis and insights.
x AI is essentially a dual use technology whereby it can provide technology driven economic growth
and also has the potential to provide military superiority.

THINGS TO KNOW

The Ministry of Defence intends to develop a future roadmap on AI with following goals:
9 To establish tactical deterrent in the South Asian region;
9 To support peaceful and commercial use of AI;
9 To mitigate catastrophic risk;
Future
9 To visualize potential transformative weaponry of future;
Roadmap
9 To facilitate in keeping a check on non-state actors;
9 To develop intelligent, autonomous robotic systems;
9 To enhance capabilities for creation, collection and analysis of data;
9 To bolster cyber defence.

The Terms of Reference for the AI Task Force under Chairmanship of C Chandrasekharan are:
9 Study use of AI by other leading countries including US, China, Japan, Germany, Russia etc.
9 Study of level of AI/ML development in India in general and specific in the context of defence
needs.
9 Make recommendations of making India a significant power of AI in defence specifically in the
Objectives area of aviation, naval, land systems, cyber, nuclear, and biological warfare.
9 Recommendations to include both defensive and offensive needs including counter AI needs.
9 Make specific recommendations with respect to arrangements for AI safety required within
defence ecosystem.
9 Make recommendations for policy and institutional interventions that are required to regulate
and encourage AI based technologies for defence sector.

FOCUS | June 2018 | RAU’S IAS 98

Download from:- www.UPSCPDF.com


https://t.me/UPSC_PDF www.UPSCPDF.com https://t.me/UPSC_PDF

Security

9 Specific suggestions for increased focus on AI within DRDO, BEL, service units, selected academic
institutions.
9 Make recommendations to work with start-ups/commercial industry in the field of use of AI for
defence purposes.
9 Make recommendations regarding appropriate strategies of working with US based companies.
9 Recommendations for funding required to achieve aforementioned goals.

MILITARY CASE USES

Developing lethal autonomous weapon systems for:

Weapon 9 Air, ground and underwater defence requirements;


Systems 9 Autonomous systems for ships, drones, machine guns, etc.
9 Both human-in-the-loop and human-on-the-loop scenarios.

Simulated War games and Training based upon:


9 Training the forces in a simulated environment;
War Games
9 Conducting pilots projects for simulated air combat mission;
and Training
9 Conducting mass personalized training & performance evaluation of personnel;
9 Simulated military equipment’s for practise.

AI can be leveraged for unmanned surveillance for:


9 Collecting video, audio and sensory data in real time;

Unmanned 9 Scouting battlefield and conflict zones;


Surveillance 9 Strengthening perimeter defence and border and maritime patrol;
9 Canvassing harsh terrains and under harsh conditions;
9 Key installations and harbour protection.

DUAL (CIVILIAN & MILITARY) USES

AI can be leveraged for cyber security to:


9 Monitor internet traffic in real time;
Cyber security 9 Act real-time on cumulative intelligence;
9 Detect malware and prevent darknets;
9 Automate cyber offence against targets & adversary networks.

AI can be crucial for intelligence gathering and reconnaissance initiatives such as:

Intelligence 9 Gathering satellite imagery;


and 9 Movement tracking;
Reconnaissance 9 Object and pattern recognition;
9 Analysing unstructured data from sensor data, radar data, video, audio & satellite imagery.

The technology can be leveraged for:

Aerospace 9 Intelligent monitoring based on RF spectrum;


Security 9 Radar perimeter optimisation;
9 Optimal ground weapons allocation in air defence network.

FOCUS | June 2018 | RAU’S IAS 99

Download from:- www.UPSCPDF.com


https://t.me/UPSC_PDF www.UPSCPDF.com https://t.me/UPSC_PDF

Security

Before the start of the holy period of Ramzan, the Government of India
based on recommendation of the Jammu and Kashmir state government
CEASE-OP’S IN
had declared a period of ‘Cease-Op’s’.
J&K It has been withdrawn now and with reference to latest happenings, there

# Internal Security is a formal end to the coalition govt. in Jammu and Kashmir.
But here, let us understand about Cease-op’s in J&K.

THINGS TO KNOW

x Cease-Ops signify that the Army avoids conducting offensive operations, which include cordon and
search (CASO) and search and destroy (SADO) operations.
What is Cease- x The counter terror and counter infiltration grids remain completely intact such as gathering
Ops? intelligence, securing military installations, patrol road security operations, etc.
x Cease-ops does not mean ceasefire at the LoC or that the army will take a back step into its
barracks and halt all operations.

x To create an environment free of terror and violence to enable the citizens of J&K to observe
Ramzan in a peaceful manner and in conducive environment so that terrorists do not exploit their
religious sentiments.
Purpose of
x For this, the government issued instructions to the security forces and the Indian army not to
Cease-Ops
undertake offensive operations during the period of Ramzan in Jammu and Kashmir.
x However, if it is essential for protecting the life of the common people, the security forces will be
compelled to take appropriate action.

BACKGROUND OF NOT SO SUCCESSFUL EFFORTS

x Non-initiation of Combat Operations (NICO) was taken in South Kashmir through 2000-01, an
initiative then taken by Prime Minister AB Vajpayee.
x The 2000-01 NICO resulted in an increase in casualties of the security forces (SF) including an
attack on Kashmir airport.
Past Exercise
x It was called off following no positive response from the terrorists or their sponsors.
x NICO mainly failed as terrorist numbers especially foreign terrorists were still too large and the
counter infiltration grid in the vicinity of the LoC was highly susceptible to infiltration in the
absence of an effective anti-infiltration obstacle.

x Operation All Out was conducted in J&K in 2017 by the Indian army and security forces to
Operation All eliminate all forms of terrorists including of Kashmiri origin & foreign origin.
Out x The operation was launched after the unrest in Kashmir that happened after the encounter of
Burhan Wani in 2016 and attack on Amarnath yatra in 2017.

x Operation All Out allowed only temporary stabilisation but the narrative of encounter of Kashmiri
origin terrorists began to tilt away from the favour of government authorities in Kashmir.
x Encounter of local militants by security forces incited more local youth to pick up arms wherein in
Critique of
2017, 126 local youths joined militancy, a sharp increase from 88 in 2016. The number has crossed
Op. All Out
80 already mid-way through 2018,
x The new militants are friends or relations of those eliminated in encounters or either belong to
villages located in proximity to spots where massively attended funerals for killed militants are

FOCUS | June 2018 | RAU’S IAS 100

Download from:- www.UPSCPDF.com


https://t.me/UPSC_PDF www.UPSCPDF.com https://t.me/UPSC_PDF

Security

held.

NAVAL Indian navy currently undertook operational deployment for joint surveillance
of Exclusive Economic Zone (EEZ) of Maldives in May, 2018. Moreover, Indian
OUTREACH navy has also begun a program of operational deployment of the Indian Navy’s
# External Eastern Fleet ships to South East Asian countries.
Let us understand about Indian navy’s outreach to Maldives and Vietnam.
Security
INDIA AND MALDIVES

x The Joint EEZ surveillance of Maldives is an endeavour of the Government of India and the Indian
Navy to ensure the safety and security of the vast EEZ of Maldives.
Background
x Indian Naval (IN) ship Sumedha has been deployed to undertake joint EEZ surveillance of Maldives as
part of Indian Navy’s ‘Mission Based Deployments’.

x Indian Navy’s Marine Commando (MARCO) conducted the 2nd Asymmetric Warfare Training Exercise
Training
‘Ekatha 2018’ in Maldives.
Exercise
x The exercise focussed on training of diving related issues, medical emergencies, etc.

Î RELATED INFORMATION: EXCLUSIVE ECONOMIC ZONE (EEZ)

An Exclusive Economic Zone (EEZ) is a concept adopted at the Third United Nations Conference on the Law of the
Sea (1982), whereby a coastal State assumes jurisdiction over the exploration and exploitation of marine resources in
its adjacent section of the continental shelf, taken to be a band extending 200 nautical miles from the shore.

The scientific importance of EEZs includes the carrying out of scientific research on varied marine creatures, and the
sampling of the sea-beds for oil and natural gas purposes which can also be carried out therein. Every country has
been given the right to safeguard their territorial waters, but if some countries wish to explore the resources, then they
have to enter into a bilateral agreement with the with the respective neighbouring country.

INDIA AND VIETNAM

x India-Vietnam strategic relations have been strengthened in recent years due to the growing
convergence on security issues.

x The relations between the two countries were elevated to the level of Comprehensive Strategic
Background Partnership during Indian PM’s visit to Vietnam in 2016.

x The relations were further cemented during the visit of Vietnamese president during the ASEAN-
India Commemorative Summit in January, 2018 and further during the bilateral visit to India in
March, 2018.

x Three Indian Naval Ships Sahyadri, Shakti and Kamorta visited Danang, Vietnam and conducted a
Current harbour and military exercise.
interaction x The Indian Navy has had extensive interactions with Vietnam’s Navy, particularly in the field of
training, repairs, maintenance and logistics support aimed at capacity building.

FOCUS | June 2018 | RAU’S IAS 101

Download from:- www.UPSCPDF.com


https://t.me/UPSC_PDF www.UPSCPDF.com https://t.me/UPSC_PDF

Security

SURYA KIRAN - XIII The 13th edition of bilateral annual military Exercise SURYA KIRAN
was conducted recently. It can be termed as the facade of long
# Military Exercise standing strategic ties between India and Nepal. Let us
# Bilateral Exercise understand about this military exercise.

The military exercise is conducted between India and Nepal Army, and was held at Pithoragarh,
Location
Uttarakhand.

The aim of exercise is joint training of troops in counter insurgency/ counter terrorism operations in
Purpose
both jungle and mountainous terrain and disaster management.

SURYA KIRAN military exercise with Nepal is the largest in terms of troop participation, comprising of
Significance
nearly 300 soldiers from both Nepal & Indian army.

Viewpoint

Peace in Nagaland
# Security
(Courtesy: The Hindu)

Read and ponder!


Reams have been written about the Framework Agreement signed on August 3, 2015 between Union government and
the National Socialist Council of Nagalim (Isak-Muivah), or NSCN-IM. The agreement has raised expectations among the
Naga people but also apprehension in neighbouring Assam, Arunachal Pradesh and Manipur, and rightly so. States
that have been created after due consideration of a number of factors and their geographical boundaries clearly
mapped out can hardly be expected to take kindly to any attempt at rearranging those boundaries. But let’s look at
whether this is the only sticking point or if there are issues too — between the protagonists of the Naga peace talks,
the NSCN-IM, and Delhi.

The two sides


A section of articulate Nagas who have questioned the secrecy around the Framework Agreement believe that its terms
need to be discussed and debated. However, this elite conveniently forgets that we are, even today, ruled by a
plutocracy and that large sections of the population have and have had no say in matters affecting their lives. So what
democracy are they talking about? Is it only the Framework Agreement that will change the entire contour of their
present existential crisis? I have my doubts.
Most in Nagaland speak with derision about the NSCN-IM because it is led by Th. Muivah, a Tangkhul Naga from
Manipur. The question often asked on social media is, why should a Tangkhul Naga decide the future of the Nagas of
Nagaland vide the Framework Agreement? But the counter question is why not?
It is true that the NSCN-IM had, for several decades, used the gun to silence its detractors, and indulged in large-scale
extortion and imposed its diktat on the Nagas. It has also tried to sabotage platforms such as the Forum for Naga

FOCUS | June 2018 | RAU’S IAS 102

Download from:- www.UPSCPDF.com


https://t.me/UPSC_PDF www.UPSCPDF.com https://t.me/UPSC_PDF

Security

Reconciliation for alternative discourses on the peace process. But to be fair to the NSCN-IM, its leaders have remained
steadfast to the goal of wresting an “honourable settlement” for the Naga people from the Government of India.
In the Indian establishment too, there are layers of prejudice as far as the Naga demand is concerned. There are some
in the security establishment and the political dispensation who dismiss outright Naga claims of a “unique history,”
instead of trying to understand where this claim arises from.

Lessons from JP
It is to the credit of democratic India that it produced a statesman like Jayaprakash Narayan (JP) who, in his interactions
with the Nagas, showed incredible comprehension and empathy about their history and origin. JP traversed the length
and breadth of Nagaland, later bringing out a book, Nagaland Mein Shanti Ka Prayas (The Attempts to Forge Peace in
Nagaland). He was perhaps one of the first national leaders to argue about a civilisational unity in India which preceded
its political unity. JP was speaking about all the tribes inhabiting India’s easternmost periphery, much before the term
“India” was coined to define this country. Even today the tribes of this region are not influenced by the “Indian” culture.
They are proud inheritors of a unique culture.
Today it is those in Naga civil society and political leaders who make visits to Delhi to prevail upon the ‘Indian’
establishment to take a more pragmatic view of the peace deal and not regurgitate the idea that India must settle all
contentious issues on the basis of the Constitution. The Nagas contend that they never signed the Instrument of
Accession to India (which the Khasi chieftains of present-day Meghalaya and the Maharajahs of Manipur and Tripura
did) and hence cannot be made prisoners to a Constitution they never had a hand in crafting. To add to the
predicament, India has not produced another statesman of the stature of JP who would break bread with the Nagas in
the spirit of a fellow traveller who does not smirk at the Naga claim of a unique history.
But at the same time, the Nagas too must admit that they have journeyed a long way with the rest of India. Both have
developed a comfort level with each other and taken cognisance of the sticking points that prevent the resolution of a
long-drawn struggle. Therefore, it is time to shed political egos and move on keeping in mind the best interests of the
next generation of aspirational Nagas. Today the Nagas have transcended the victimhood syndrome and shed their
hatred for an India once perceived to be the enemy. The current interlocutor, R.N. Ravi, too has been most open and
accepting of a broad spectrum of views from a cross-section of Naga civil society.
It is in this climate of mutual respect that the Framework Agreement ought to proceed, which is what drives it,
according to Mr. Ravi. Insularity and intransigence from either side or from both sides is unlikely to result in any good
for the Naga people.
Speaking of a peace deal within the Constitution, let us admit that pluralism is its cornerstone. And pluralism
legitimises the compromises that are necessary to negotiate contesting claims in a country like India.

FOCUS | June 2018 | RAU’S IAS 103

Download from:- www.UPSCPDF.com


https://t.me/UPSC_PDF www.UPSCPDF.com https://t.me/UPSC_PDF

Miscellaneous
In one of the rarest sustained feats, Rafael Nadal has conquered
RAFAEL NADAL: French Open men’s singles title for the 11
th
time. No other men’s
DID IT FOR THE player has dominated a particular court so consistently as Nadal, who
has doggedly dominated clay court of the French open by winning it
th
11 TIME th
11 time since 2005.

# Tennis #Global He is a symbol of sheer perseverance, and will to win. He now stands
at 17 Grand Slam title just 3 behind his great rival friend, Roger
Tournament Federer. This again opens the debate for Greatest of All Times (GOAT).

MAJOR RESULTS
Runner Up: Dominic
Men’s Final Winner: Rafael Nadal (Spain)
Thiem (Austria)

Women’s Winner: Simona Halep Runner Up: Sloane


Final (Romania) Stephens (U.S.)

FOCUS | June 2018 | RAU’S IAS 104

Download from:- www.UPSCPDF.com


https://t.me/UPSC_PDF www.UPSCPDF.com https://t.me/UPSC_PDF

Miscellaneous

The National Awards for films were started in 1954 as an annual


NATIONAL incentive by the Government of India for the making of artistic,
competent and meaningful films. It cover the entire national spectrum
FILM AWARDS of Indian Cinema, and are the most coveted and prestigious awards in
2018 the country.
The awards are given in three sections – Feature Films, Non-Feature
# AWARDS
Films and Best Writing on Cinema.

MAJOR WINNERS

Dada Saheb Phalke


Vinod Khanna
Award

Best Feature Film Village Rockstars (Assamese)

Best Actor Ridhhi Sen for Nagar Kirtan (Bengali)

Best Actress Sridevi for Mom (Hindi)

Best Director Jayaraj for Bhayanakam (Malayalam)

Best Popular Film Baahubali 2 (Telugu)

The Linnean Medal is awarded to a biologist every year by the Linnean Society of

LINNEAN London. It is the world’s oldest active biological society founded in 1788 and
named after famous Swedish biologist Carl Linnaeus who gave us one of the
MEDAL systems of naming plants and animals.

# AWARDS It is awarded as an expression of the society’s esteem and appreciation for


service to science.

Recently, it was bestowed to an Indian botanist Kamaljit S. Bawa.


He is the president of Bengaluru-based non-profit Ashoka Trust for
Research in Ecology and the Environment (ATREE).
He is being recognised for his pioneering research on the
evolution of tropical plants, tropical deforestation, non-timber
forest products and for decades of work on the biodiversity of
forests in Central America, the Western Ghats and the Eastern
Himalaya.

FOCUS | June 2018 | RAU’S IAS 105

Download from:- www.UPSCPDF.com


https://t.me/UPSC_PDF www.UPSCPDF.com https://t.me/UPSC_PDF

Miscellaneous

CHINA’S TRANSFORMATION
# BOOKS AND AUTHORS
Genre: Non-Fiction, International Affairs
Author: Manoranjan Mohanty
In the midst of growing dominance of China worldwide, this book deals with
the role and thinking of the Communist Party of China and the manner in
which it exercises control over all aspects of the Chinese polity, economy and
society.
Even the rest of the world has been watching with curiosity and appreciation
the great socio-economic transformation that China has been experiencing
since the formation of the People’s Republic of China some seven decades
ago.
Flow of the book: The book focuses on the following:
x Foreign investment and its impact on transforming China.
x Democratic centralism: Philosophy of Communist Party. It is more of
centralism and less democracy, unique for China.
x Household responsibility system: It was launched, allowing households to
lease out land to produce grain for their needs. The surplus would be
procured to feed the urban population. This emerged as the new
experiment to boost rural incomes and food security.
x It is an apt take on China’s socio-economic and global transformation. A
lesson for many countries in holistic transformation.

FOCUS | June 2018 | RAU’S IAS 106

Download from:- www.UPSCPDF.com


https://t.me/UPSC_PDF www.UPSCPDF.com https://t.me/UPSC_PDF

ETHICS, INTEGRITY & APTITUDE


# GS Paper IV (Main)

Most of the questions asked by UPSC in GS Paper-IV


test familiarity of a student with his/her
surroundings, experiential learning, observations,
power of deduction, and rational decision making. In
10 AMAZING SOURCES FOR this regard, through this section of FOCUS, we try to
represent different ways of understanding and
ANSWERING QUESTIONS answering various UPSC questions by developing
basic comprehension about various sources.
#Ethics, Integrity & Aptitude
With respect to above, we are presenting in this
#Questions section a summary of such hints/sources presented
to the students. Let’s have a relook in the form of
revision for comprehensive overview and for better
understanding.

Read, enjoy, ponder and practice !


----------------------------------------------------------------------------------

Possible Source I: Newspaper

An excerpt from The Hindustan Times : On governance flaws and paralysis of implementation

……a man is carrying his dead wife on his shoulder as though she is a log. He is somewhere in the Kalahandi district of
Odisha, on his way to his village on top of a hill, about 50 kilometres away. He is walking because he does not have the
money to hire a vehicle. Hours before the picture was taken, his wife, who was in her thirties, had died of tuberculosis
in a government hospital. The hospital asked him to quickly take her away because a bed is precious in these parts.
According to the man, the hospital refused to help him with an ambulance or a hearse even though, by law, he has a
right to the free facility. The girl who is walking beside him is his youngest daughter, who is 12.

The corpse is swaddled in at least two pieces of cloth. It is a neat, tight wrap. The man must have done it himself. It is
hard to get ambulance drivers in villages to transport the dead. They are either too drunk or they ask for bribes.

How is it that so much of Odisha’s treasures have been mined away without its masses receiving their share? The
poverty of the man then is an inheritance. They would say it is also a result of corrupt politics that is preoccupied with
enriching large corporations. If there was industrialisation in or in the proximity of this district the chances are he
would have been tricked into selling his land for almost nothing, which he would have gambled away anyway. And he
would have joined the millions of landless poor who live in inhuman conditions in the mega cities……

Hint: On avoiding governance flaws and paralysis of implementation

FOCUS | June 2018 | RAU’S IAS 107

Download from:- www.UPSCPDF.com


https://t.me/UPSC_PDF www.UPSCPDF.com https://t.me/UPSC_PDF

Ethics, Integrity & Aptitude

Possible Source II: (Open Page, The Hindu)

Story: I was on a walk down the lane towards the market, when I witnessed a sight.
An elderly man with a torn rug thrown around him as a shield from biting cold was seated on the steps of a shop
building. The shop had not yet opened and he was relaxing outside, as if it was his own palace. The little mat, a long
bag and a smile were all he seemed to have.
He was stretching his legs, humming to himself, lost in his own world. He had transitioned to a different zone. His day
had begun to take off. Slowly, he took out two packets of biscuits from his bag and got started on one of them. That
was when his canine pal, Punnu, came over. It asked neither for the biscuits nor for any pampering. All it did was just
sit beside the man, looking down with those puppy eyes.
I just stood watching this scene.
The man then held Punnu close to himself, wrapped the rug partially around it and in the local language exclaimed, “I
have grown strong over the years, I can bear this hard cold, but you cannot! But maybe you will take inspiration from
me and become like this old man, someday, oh my successor.”
Later he shared some biscuits with Punnu and was happily singing as the dog happily wagged its tail.
Suddenly one of the man’s slippers fell on to the road. I guess he had only one pair. There was panic, to go retrieve
them quickly. I thought I would go ahead and get them for him, but to my amazement Punnu ran across and got them,
without being told to do so. He knew it was his turn to show love.
The man then exclaimed, “Take good care of all my properties this way, my lad!”

FOCUS | June 2018 | RAU’S IAS 108

Download from:- www.UPSCPDF.com


https://t.me/UPSC_PDF www.UPSCPDF.com https://t.me/UPSC_PDF

Ethics, Integrity & Aptitude

A moment of truth, it struck me. True love and affection still exist on earth. The man himself did not have enough food
to check the growl of his stomach, but he went on to share the little he had with the animal, which in turn understood
the man’s need without even being told. He took care of the dog like his own child! He didn’t have much, but he still
had so much contentment within himself and an even bigger smile on his face.

Now see the contrast....


Most people, on the other hand, complain of something or the other that they do not possess. There is always a want
for more and more and much more. Plotted on a graph, needs will always show an upward climb.
Most of us have forgotten how it is to have a big, wide smile, a natural one which comes from deep within. Often it is
either a fake smile or a forced one. At the back of our mind a thought process is always on about fulfilling our never-
ending wants.
So, are we content with what we have as of today? I guess most of us will give the answer, no! Given the chance of
choosing between contentment with peace in life and grabbing loads of newly released offers, most of us would
choose the latter.
Do our needs have an end? I guess the infinity symbol suits best here. Can we see some limits for the wants, at least
farther away in time? It is more like a joke for most of us. We collect things more than memories, as if we all are
immortal.
This should change, at least gradually, maybe one small step at a time.
Realisation of true contentment lies in the fact that it all resides within the small pleasures and joys of life, like a deep
slumber, being able to read your favourite novel as you sip a hot cup of coffee, a random dance as you listen to your
favourite lines, laughing with your family and loved ones, a sudden travel plan ...
All we’ve got to do is to try keeping a foot forward toward this. The results will be impeccable! None of us can behold
the future, so let us not be a part of the mad race to nowhere! Make moments, make life worth living.

Are we content with what we have as of now? Are our needs finite? Can we see some limits
Things to for wants?
ponder
Do our needs ever cease? The path to true contentment starts from the point that happens.

Possible Source III: History

DHAMMA: ON ETHICS FOR ALL


In brief: It is a non-aggressive, moral and philosophical system expounded by the Buddha, which demands no blind
faith from its adherents, expounds no dogmatic creeds, encourages no superstitious rites and ceremonies, but
advocates a golden mean that guides a disciple through pure living and pure thinking to the gain of supreme wisdom
and deliverance from all evil. The teachings of Dhamma are summarised in Noble Eightfold path.

Noble Eightfold path


Ethical conduct is built on the vast conception of universal love and compassion for all living beings, on which the
Buddha’s teaching is based.

According to Buddhism, for a man to be perfect there are two qualities that he should develop equally: compassion
(karuna) on one side, and wisdom (panna) on the other.

FOCUS | June 2018 | RAU’S IAS 109

Download from:- www.UPSCPDF.com


https://t.me/UPSC_PDF www.UPSCPDF.com https://t.me/UPSC_PDF

Ethics, Integrity & Aptitude

9 Compassion represents love, charity, kindness, tolerance,


and such noble qualities on the emotional side, or qualities
of the heart.
9 Wisdom stand for the intellectual side or the qualities of the
mind.
9 Emotional intelligence: If one develops only the emotional,
neglecting the intellectual, one may become a good-hearted
fool; while to develop only the intellectual side neglecting
the emotional may turn one into a hard-hearted intellect
without feeling for others.
Therefore to be perfect, one has to develop both equally. That
is the aim of the Buddhist way of life: in it wisdom and
compassion are inseparably linked together and these are very important traits for an effective governance.

Possible Source IV: Polity

Idea for values based education:


Drawn from the Preamble and Fundamental Duties
The idea of what values should be fostered through education has been best expressed in the Constitution of
India. In its Preamble itself, the Constitution lays down four universal values:
JUSTICE, social, economic and political;
LIBERTY of thoughts, expression, belief, faith and worship;
EQUALITY of status and of opportunity; and to promote among them all;
FRATERNITY, assuring the dignity of the individual and the unity and integrity of the nation.

Further, the Indian Constitution has explicitly laid down Fundamental Duties of its citizens in Article 51A,
emphasizing that every Indian citizen would:
9 ‘promote harmony and spirit of common brotherhood, transcending religious, linguistic and regional or sectoral
diversities;
9 renounce practices derogatory to the dignity of women;
9 value and preserve the rich heritage of our composite culture;
9 protect and improve the natural environment;
9 develop the scientific temper;
9 abjure violence and
9 strive towards excellence in all spheres of individual and collective activity so that the nation constantly rises to
higher levels of endeavour and achievement’.

FOCUS | June 2018 | RAU’S IAS 110

Download from:- www.UPSCPDF.com


https://t.me/UPSC_PDF www.UPSCPDF.com https://t.me/UPSC_PDF

Ethics, Integrity & Aptitude

Possible Source V: Eminent and Ethical Personalities

Hint: Lessons from the life of Nelson Mandela :


“The weak can never forgive; forgiveness is the attribute of the strong.”

x Percy Yutar was the state prosecutor at the 1963 treason trial at which Mandela was convicted
Had lunch of sabotage and sentenced to hard labour for life.
with Percy
x Yutar demanded the death penalty for Mandela.
Yutar
x In 1995, Mandela invited Yutar to dinner. Mandela said that Yutar had only been doing his job..

x Mandela spend 20 years of his incarceration on the Robben Island prison.


x Christo Brand was a jailer responsible for guarding Nelson Mandela at Robben Island.
Invite to
x Mandela invited him to a dinner for marking the 20th anniversary of his release from prison.
Christo Brand
x Speaking about the relationship which developed between the two men, Mandela said it
"reinforced my belief in the essential humanity of even those who had kept me behind bars.".

x On becoming the president of South Africa in 1994, he invited Prisoner warder Paul Gregory to
Invite to Paul the inauguration ceremony.
Gregory x The Mandela acknowledged that he shared a strong bond with his warder. He inspired
Mandela to remain focused to fight against apartheid.

x During the apartheid era, one of the few symbols that summed up oppression for Mandela and
his African National Congress (ANC) colleagues was the hatred towards green Springbok jersey.
x The all white Springbok team was highly racist, arrogant, and was hated by the native blacks.
x Such was the disconnect that in home matches, the black South Africans cheered for the
Supported
opposition team.
Springbok
x The 1995 World Cup Rugby tournament in South Africa was boycotted by the ANC.
rugby team
x But Mandela made a huge statement by wearing a green jersey at the World Cup final in 1995.
x He presented the trophy to South African captian Francois Pienaar, sending out a strong
message to his ANC colleagues that it was time to put aside enmity and become a united
country.

x If a man tortured and imprisoned for nearly three decades could find in himself forgiveness,
Food for what lessons might we take from him on how to respond to acts of aggression and cruelty in
thought our ordinary lives?
x If we cannot forgive in our own lives, perhaps we can learn to forget.

Hint
Lal Bahadur Shastri: Politician with ethics
Some instances from his eventful and ethical life:

x In 1965, Lal Bahadur Shastri (when he was the PM of our country) bought a Fiat car after
taking a loan of Rs 5,000 from Punjab National Bank. When his loan was approved early,
Repaying of car
Shastri told the bank, "The common man should have the same privilege". This is how he
loan
used to think about the nation!
x He died at Tashkent in 1966 before he could repay the loan. The Bank wrote to his widow

FOCUS | June 2018 | RAU’S IAS 111

Download from:- www.UPSCPDF.com


https://t.me/UPSC_PDF www.UPSCPDF.com https://t.me/UPSC_PDF

Ethics, Integrity & Aptitude

Lalita Shastri for repayment of the loan of Rs 5,000 which her husband owed.
x The lady promised to pay back in instalments from her family pension. She repaid every
Rupee of the Rs 5,000.
x We can’t even think of it happening in the 21st Century!

Moral responsibility In 1956, Lal Bahadur Shastri resigned from his post, following a train accident that killed
and resignation around 150 passengers near Ariyalur in Tamil Nadu.

x To overcome the acute food shortage, Shastri asked the experts to devise a long-term
strategy. This was the beginning of famous "Green Revolution" to make India food grain
Thinking and
self-sufficient.
working for the
x Apart from the Green Revolution, he was also instrumental in promoting the White
people and nation
Revolution. The National Dairy Development Board was formed in 1965 during Shastri’s
stint as Prime Minister.

x This means that politics and ethics can go together.


Food for thought x This puts a question:
Q. Is politics devoid of ethics or the type of people entering politics lack ethics?

Possible Source VI: Moral Science stories

Moral story: Controlling anger


Anger is a harmful negative emotion. It is injurious to both personal life and work life. (a) Discuss how it leads
to negative emotions and undesirable behaviours. (b) How can it be managed and controlled?

There was once a young boy who had problem controlling his temper. When he became angry, he would just say
anything that came to his mind and hurt people. So his father gave him a bag of nails and a hammer and said, “Every
time you get angry, hammer one nail into the fence in our backyard”.
The first few days the boy hammered so many nails that he emptied half the bag. Over the weeks, the number of nails
he hammered to the fence reduced and gradually, his temper was much in control. Then came a day when he didn’t
lose his temper at all. His father asked him to remove one nail each day that he manages not to lose his temper.
Finally, on the day the child was removing the last nail, his father says, “You have done well, boy. But do you see the
holes in the wall? The fence is never going to be the same, even after repainting. Likewise, when you say mean things in
anger, you will leave a scar in the person’s mind, as the nails did to the fence”.
Moral: Anger is a dangerous weapon like a knife. When you put a knife in a man and draw it out, the wound heals but
the scar remains.

Moral story: A Wise Old Owl

What is ’emotional intelligence’ and how can it be developed in people?


How does it help an individual in taking ethical decisions?

FOCUS | June 2018 | RAU’S IAS 112

Download from:- www.UPSCPDF.com


https://t.me/UPSC_PDF www.UPSCPDF.com https://t.me/UPSC_PDF

Ethics, Integrity & Aptitude

There was an old owl that lived in an oak. Every day he saw incidents happening around him. Yesterday, he saw a boy
helping an old man to carry a heavy basket. Today, he saw a girl shouting at her mother. The more he saw the less he
spoke.
As he spoke less, he heard more. He heard people talking and telling stories. He heard a woman saying that an
elephant jumped over a fence. He also heard a man saying that he had never made a mistake.
The old owl had seen and heard about what happened to people. Some became better and some became worse. But
the old owl had become wiser each and every day.

Moral: You should be observant, talk less but listen more. This will make you a wise person.

Possible Source VII: Books on life lessons

Small excerpt from: Who will cry when you die?

x The greatest purpose of life is to live a life of purpose.


x For this, you need to discover your calling (Lakshya) because life should not be lived on a default
mode but in a properly planned manner.
Discover x Life has a habit of acting on you as it overpowers a person, and as a result, days become weeks,
yourself weeks become months, and months slip into years. This scenario leaves behind a regret over a life
half-lived.
x So after a life half-lived, a question naturally emerges….”What would you do if you could live your
life over again?” Naturally, you will not like to repeat similar mistakes.

x Is it difficult to maintain focus? Are you overpowered by your thoughts, inertia and problems?
Maintain x Try to question yourself now….”Are the problems you have experienced or the challenges you might
your currently be facing really as serious as you have made them out to be?”
focus x Our lives are just a blip in the realm of time so have your focus and develop wisdom to enjoy the
journey and savour the process.

x The golden thread of a highly successful and meaningful life is self-discipline.


x A good sportsperson is highly self-disciplined so as to improve on daily process.
Develop x For a student, self-discipline is to learn new things on daily basis, and the cumulative effect of daily
self- sustained learning will naturally be seen in the examination.
discipline x Self-discipline basically requires toughness with oneself so as to overcome deviations and desires.
x This tough love with yourself is actually the greatest gift to yourself because it is due to self-
discipline only that you can make your life meaningful and rewarding.

x A positive start to a day is half the work done.


x Silent contemplation is required for atleast 15 minutes on how the day is going to be unfolded for
Have a
reaping positive dividends.
positive
x For next 15 minutes, read and focus on a positive story, or a piece of literature so that early morning
start
infuses and enlightens every remaining hour of the day.
x So start your day well and this is the beginning of right self-discipline.

FOCUS | June 2018 | RAU’S IAS 113

Download from:- www.UPSCPDF.com


https://t.me/UPSC_PDF www.UPSCPDF.com https://t.me/UPSC_PDF

Ethics, Integrity & Aptitude

Possible Source VIII: Dohas

HINT: KABIR KE DOHE

Dohas Meaning

Lesson on Self-realisation "I searched for the crooked man, but failed to find one.
"Bura Jo Dekhan Main Chala, Bura Naa Milya Koye, Jo But when I searched within myself, I realised there was
Mann Khoja Aapna, To Mujhse Bura Naa Koye." none more crooked than me!"

Lesson on Self-discipline "Instead of doing tomorrow's work on the morrow, do it


"Kaal Kare So Aaj Kar, Aaj Kare So Ab, Pal Mein Pralaya today, and do today's work, right now! If the moment is
Hoyegi, Bahuri Karoge Kab.” lost, can you get it back any way?"

Lesson on Politeness "Speak such words, without the ego's manipulative ways,
"Aisee Vani Boliye, Mann Ka Aapa Khoye, Apna Tan that not only your own body remains composed, you
Sheetal Kare, Auran Ko Sukh Hoye." also give your listeners joy through your words."

Lesson on Contentment "Give me only so much, O God, that it suffices to fulfil the
"Saayi Itna Deejiye, Ja Mein Kutumb Samaye, Main Bhi needs of my clan, I should not suffer cravings, nor
Bhookha Na Rahoon, Sadhu Na Bhookha Jaye." should my visitor go unfed."

Lesson on Politeness "Of what use is eminence, to be like a tall date tree? It
"Bada Hua To Kya Hua, Jaise Ped Khajoor Panthi Ko neither offers shade to travellers, not is its fruit easy to
Chaya Nahin, Phal Laage Atidoor." reach!"

Possible Source IX: Issue based movies, documentaries and songs

Each one of us loves to watch movies and the movie libraries are filled with gems created by geniuses who have
beautifully observed the world, and displayed it with brilliance and exuberance.
These can be very important source of case studies reflecting problems and prospects for our society.

SMALL HINT

Movies on reforming education system 3 Idiots, Taree Zameen Per, I am Kalam etc..

Dealing with plastic menace BBC Documentary: Plastic World

FOCUS | June 2018 | RAU’S IAS 114

Download from:- www.UPSCPDF.com


https://t.me/UPSC_PDF www.UPSCPDF.com https://t.me/UPSC_PDF

Ethics, Integrity & Aptitude

Possible Source X: Cartoon Strips from comics and newspapers

What do you think now- values change or remain eternal?


Kindly note: The list of 10 sources mentioned above is not exhaustive, there can be many more sources. This is just an
effort to guide the students for adopting multiple sources of knowledge which are easy to comprehend and also very
interesting.
Finally knowledge is spread all across, try to connect the dots and answer questions both in the examination and in
your daily life.

GET STARTED !
----------------------------------------------------------------------------------

FOCUS | June 2018 | RAU’S IAS 115

Download from:- www.UPSCPDF.com


https://t.me/UPSC_PDF www.UPSCPDF.com https://t.me/UPSC_PDF

Glossary
# Terms

Enrich Your Concepts

x It is also known as the genetic bottleneck.


x This refers to any kind of cataclysmic event that reduces the population of a species to
a relatively tiny number. These could be natural catastrophes like earthquakes and
floods or disasters like major wars and genocides caused purely by human beings.

Population (Extinction of DINOSAURS).


x Bottleneck events can adversely affect the genetic diversity of a species since only a
bottleneck smaller population of the species will now be able to successfully pass on genes to the
# Biology next generation.
x Certain genetic variations may, in fact, become either completely lost or infrequent in
their occurrence, thus reducing their chances of survival.
x Even excessive poaching creates such effect, as tigers were on the verge of extinction
but Project Tiger reversed it substantially.

x This refers to a cognitive bias that causes people to overestimate their worth, while
underestimating their unfavourable drawbacks.
x They attribute positive events in their lives to their own high capability and character
Self-serving and link negative events to external factors like bad luck or the unfavourable actions of
bias other people.
x Self-serving bias helps people to protect their ego from negative feedback that could
#Human Psychology damage their self-worth.
x The world is full of these types, so it is difficult to find a sincere self-analyser looking
inwards for gaps and solutions.

x In continuation to the above ‘Self-serving bias’, this is also a good reflector of human
psychology.
x In short, Introspection illusion refers to a cognitive bias which causes people to believe

Introspection that they understand the underlining reasons and motives behind the choices that
they make.
illusion x Cognitive psychologists argue that when people are asked about the reason for their
#Psychology choices, they usually make up answers that don’t actually explain the rationale behind
their choices.
x Hiding their true self: This is because even when people do not have sufficient insight
into their feelings, they still wish to offer a rational explanation for their choices.

FOCUS | June 2018 | RAU’S IAS 116

Download from:- www.UPSCPDF.com


https://t.me/UPSC_PDF www.UPSCPDF.com https://t.me/UPSC_PDF

Glossary

x This refers to people’s tendency to donate more money and gifts when they are told
about the sufferings of an individual rather than when they are told about the
problems of a large group.
x So, a person may be more likely to donate for a development cause when he is
Identifiable persuaded with information about a poor man's abject living conditions instead of the
miseries of the world’s poor.
victim effect
x The identifiable victim effect is seen as preventing donors from making objective
#Psychology decisions. Instead, it makes them take emotional decisions that may not necessarily
help out the most unfortunate victims in need of help.
x For e.g. The picture of the Orissa man carrying his dead wife, which went viral, poured
more support and donation rather than supporting the cause of overall upliftment of
the poor.

x This is in contrast to Placebo Effect, which cure the people psychologically.


x So it refers to the phenomenon wherein a patient undergoing medical treatment may
suffer from negative side effects purely due to her own negative expectations.
x Negative psychological expectations are believed to cause physiological changes in a
Nocebo Effect patient’s body, leading to negative health effects.

# Psychology; Medicine x Scientists found that patients who were warned about negative side effects of a certain
medicine actually suffered these side effects even though they were only served
harmless fake pills.
x This brings the role of patient’s psychology in treatment, and it should be positive for
faster recovery.

x Contrast to Camouflaging
x It is also known as warning colouration.
x This refers to how animals change colour or adapt physically to counter threats from
various predators. Such features warn predators to refrain from attacking an animal as
Aposematism it is either poisonous or possesses other self-defence mechanisms to harm the
predator.
# Biology
x In contrast to other protective mechanisms, which usually try to hide the creature from
potential predators, aposematic warning mechanisms try to grab the attention of
predators by advertising their dangerous characteristics.
x Eg..Rattle of a Rattle Snake.

x Making other planets habitable for Human Beings.


x This refers to the process by which the living conditions on a planet are changed
deliberately to match the conditions on earth.
x The term was coined by American science fiction writer Jack Williamson.
Terraforming x The terraforming of small areas of planets like Mars has been considered as a way to
# Physics change their harsh living conditions and enable successful human inhabitation.
x Among other things, the process involves the modification of the temperature and
atmosphere of the planet as well as providing an adequate supply of water through
the use of various advanced technologies.
x Even the work is in process to create a full habitable city on Mars by 2117, planned and

FOCUS | June 2018 | RAU’S IAS 117

Download from:- www.UPSCPDF.com


https://t.me/UPSC_PDF www.UPSCPDF.com https://t.me/UPSC_PDF

Glossary

funded by Dubai.

x This refers to the economic benefit received by the citizens of a country when the
government redirects spending from military to civilian purposes.
x So, the resources that were earlier allocated towards war efforts now became available
for civilian uses, thus improving living standards.
Peace dividend
x The reduction of taxes, which were earlier used to fund the military, also increased the
# Economy incentive to work.
x This was witnessed in post Second World War Europe. For instance, countries that took
part in World War II witnessed an economic boom after the war ended and soldiers
returned to work in the more peaceful sectors of the economy.

x Designing of consumer friendly products.


x Consumers are not always aware about their pain points so this refers to any unsolved
problem facing a consumer in the market.

Pain point x For this investors and entrepreneurs try to identify consumer pain points in order to
decide on making investments and developing new products for an unexplored
# Economics market.
x The study of pain points is common to start-ups which try to find the actual potential
of their ideas before committing to making huge investments. It is also the centre of
attention of groups that are involved in conducting market research.

x This refers to the financial profits that a capitalist earns by underpaying his workers.
x The idea of surplus value was proposed by German philosopher Karl Marx in his
various works, including his famous book, Das Kapital.
Surplus value
x Marx believed that labour is fundamental to all value created in any economy and that
#Economics underpaid labour is the source of all profits that accumulate to capitalists.
x Example: This is the most common and pan-business phenomenon. That is why labour
friendly labour laws and HR policies are absolutely essential.

x The windfall tax is one of the options being considered by the government as a
permanent solution to dealing with the problem of spike in oil prices.
x It is a tax levied by governments against certain industries when economic conditions
Windfall Tax
allow those industries to experience above-average profits.
#Economics x Windfall taxes are primarily levied on companies in the targeted industry that have
benefited the most from the economic windfall, most often commodity-based
businesses.

x Compounded annual growth rate or CAGR measures the return on an investment


Compounded which grows over a period.
Annual Growth x Most people consider the absolute return on an investment, which shows the

Rate (CAGR) percentage growth in the initial amount invested.


x CAGR breaks down the actual return from an investment into an annual figure
#Economics assuming a constant annual return for the period under consideration.

FOCUS | June 2018 | RAU’S IAS 118

Download from:- www.UPSCPDF.com


https://t.me/UPSC_PDF www.UPSCPDF.com https://t.me/UPSC_PDF

Glossary

x CAGR is useful to compare historical returns across investment. It also helps in making
an investment choice and helps compare performance across different assets.

x When an investor takes a pool of its money and invests it across different assets
Asset like gold, fixed deposits, equity and even real estate, the process is called asset
allocation with a view to diversify his/her own risk.
Allocation
x The objective of having a thought-out asset allocation across assets is to build a
#Economics portfolio that closely matches investor’s future return expectation while keeping in
mind the risk or volatility that he/she can bear.

x The Unified Payments Interface (UPI) system allows a bank customer to transfer
money and make payments to another bank account on a real-time basis.
Unified x While UPI uses the IMPS (Immediate Payment Service) platform, unlike IMPS, UPI
does not ask for details about the receiver’s bank account. All it requires is a UPI virtual
Payment
ID which is like an email ID. Similarly, to receive payments, you just have to share your
Interface (UPI) virtual ID.

#Economics x Both the sender and receiver need to be on the UPI platform. UPI apps like BHIM app
also allow transfer using the receiver’s bank account details like name, bank account
number and IFSC code.

FOCUS | June 2018 | RAU’S IAS 119

Download from:- www.UPSCPDF.com


https://t.me/UPSC_PDF www.UPSCPDF.com https://t.me/UPSC_PDF

Part TWO

CONTRIBUTORS
ZONE
articles .opinions .essays .notes
by rau’s professors & students

Download from:- www.UPSCPDF.com


https://t.me/UPSC_PDF www.UPSCPDF.com https://t.me/UPSC_PDF

lEAD ARTICLES

OPERATION GREEN – ALTHOUGH LATE BUT


RIGHT INITIATIVE
#Economy
DR. J. C. SHARMA
FACULTY (GS: ECONOMY AND OPTIONAL: ECONOMICS)

The year 2017 was a nightmare for potato farmers in Box : Operation Flood or White Revolution
West Bengal. The State reported a 22 per cent jump in
production at 110 lakh tonnes in 2016-17 leading to a Launched in 1970, Operation Flood, also known as the white
price crash. Prices at the time of harvesting (in revolution, had the twin objectives of increasing milk
December-January), were ruling as low as ₹110-120 a production and empowering farmers. At the time, India was
packet of 50-kg or around ₹2.4 a kg. During 2018, tomato milk deficient, importing to meet its growing needs. Thirty-six
prices at the Vashi wholesale market plummeted to ₹6-7 years later, in 2016, India ranked first in milk production,
a kg from ₹-20 only about a fortnight ago. Onion prices at accounting for 18.5 per cent of world production.
Lasalgaon in Maharashtra, Asia’s largest wholesale
market, declined to ₹25/kg from ₹30-35 last week on Interlinkage and cooperatives: Operation Flood was

improved supplies. Price volatility brought about by aimed at forming a national milk grid that connected milk

variation in production is unique to most agri- producers in rural areas to consumers in urban areas. Delhi,

commodities, particularly so for vegetables such as Mumbai, Kolkata and Chennai were the cities initially

tomato, onion and potato, which are consumed targeted to create a market. The grid was to be developed by

throughout the year. The seasonal and regional removing middlemen and consolidating milk producers

productions of these perishable commodities pose a through cooperatives. The grid helped in multiple ways.

challenge in connecting farmers and consumers in a Effectively reaching consumers without middlemen enabled

manner that satisfy both. higher monetary incentives for producers. The demand-
supply gaps closed as well, as milk pricing was less
Finance Minister in his budget speech on 1st February, susceptible to regional and seasonal variation. Operation
2018 addressed this challenge and proposed to launch Flood also brought with it the benefits of cooperatives. It
an ‘Operation Greens’ on the lines of ‘Operation Flood’. brought milk production in the ambit of the organized
Operation Flood changed the face of milk production in sector, and enabled development of infrastructure. Now
India; making the country the largest producer of milk in cooperatives collectively performed not only the task of
the world — in 2016-17, the country produced 164 procuring milk, but also other activities in the supply chain.
million metric tonnes (MMT). But it’s important to note In fact, an essential part of Operation Flood was direct
that Operation Flood was driven largely by smallholders marketing by the cooperatives. According to the National
and the AMUL model has ensured that 75-80 per cent of Dairy Development Board, “direct marketing of milk by
the price paid by milk consumers goes to the farmers. producers’ cooperatives increased by several million litres a
The following Box summarizes the success story of day”.
‘Operation Flood’ in the country since 1970.

FOCUS | June 2018 | RAU’S IAS 121

Download from:- www.UPSCPDF.com


https://t.me/UPSC_PDF www.UPSCPDF.com https://t.me/UPSC_PDF

Modernization: As Operation Flood progressed, it began to have been connected to e-NAM, and the remaining 115
rely on technology and modernization to increase will be connected soon. It is also announced that a total
production. Better feed and healthcare was provided to of 22,000 rural haats will be upgraded for the benefit of
cattle. Cattle were vaccinated, given urea and protein rich small farmers. These will be turned into Gramin
diets. Operation Flood relied on artificial insemination as Agricultural Markets (GrAMs) where farmers can sell
well. their produce directly to consumer and bulk purchasers.
The government also plans to strengthen physical
Participation of women: Operation Flood focused on infrastructure at these local agri markets through
educating the producers, and funding research and MGNREGA and other government schemes. An Agri-
development to enable greater yield. Another important Market Infrastructure Fund of Rs 2000 crore will be set
aspect were women’s cooperatives, which contributed to the up for developing these 22,000 GrAMs and 585 APMCs.
movement significantly. As per Census 2011, 65 per cent of This interlinking intends to remove the need for multiple
the total female workforce in India is engaged in agriculture. mandi prices at different levels within the same state,
Over 30 per cent of the total cultivators and 42.6 per cent of and aids in free flow of agricultural commodities. While
agricultural labourers are women. The government has been Operation Flood focused on increasing milk production,
attempting to improve the state of women in the field of Operation Green wouldn’t necessarily need to do that,
agriculture by providing more opportunities. Mahila Kisan owing to the Green Revolution that has already taken
Sashaktikaran Pariyojana was one such scheme place in the country. The idea behind Operation Greens
implemented in 2010-11. A few centrally sponsored schemes is to double the income of farmers by the end of 2022.
are also required to earmark at least 30 per cent
expenditure on women farmers. Operation is essentially a price fixation scheme that aims
to ensure farmers are given the right price for their
Transportation and storage: Given India’s weather produce. The main objective of operation green will
conditions, Operation Flood overcame the obstacle of therefore be to reduce price volatility in these
transporting a perishable like milk through 140 insulated rail commodities, thereby helping farmers augment incomes
milk tankers of a capacity of 40,000 litres. Further another on a sustainable basis. It also aims to provide these
25 rail tankers of 21,000-litre capacity were used to ferry vegetables to consumers at affordable prices. The litmus
milk. There were many such insulated road milk tankers as test of this scheme would be in containing the booms
well. For future use, storage facilities for powdered milk were and busts in prices. It will need to avert situations where
created. Schemes like the Pradhan Mantri Krishi Sinchayee farmers have been forced to dump potatoes and
Yojana, Bharat Nirman and PMGSY, focused on water tomatoes on roads. Also, checking the prices from going
harvesting, irrigation and rural roads which facilitated the through the roof has compelled the government to ban
farmers to improve the productivity of their cattle and exports. It has resorted to de-stocking and conducted
transportation o milk from remote rural areas. As a result, income tax raids on traders — the raids on onion traders
the share of livestock production in agriculture GDP has in September last year are a case in point.
increased from 4% in 2002-03 to 13% in 2012-13.
India is the second largest producer of vegetables in the
Operation Greens wants to replicate that success story in world with about 180 MMT. But China produces four
fruit and vegetables, starting with tomatoes, onions and times more vegetables than India. The following graph
potatoes (TOP). Operation Green will help regularize shows a structural break in India’s vegetable production
supply of TOP so that the market does not face a glut. around 2003-2004. Although not as revolutionary as the
Applying the idea of inter-linkage in operation flood, Green Revolution in wheat, or the White Revolution in
operation green would focus the interlinking of markets, milk, yields of potatoes, onions and tomatoes — which
which the government is already focusing on through e- constitute almost half of the country’s vegetable
NAM aiming at interconnecting e-mandis for a unified production — have shown a healthy growth. For
national market to ensure better price for the farmers. example, between 2003-2004 and 2017-2018, potato
Budget speech in 2017 had mentioned the expansion of production increased from 28 MMT to 49 MMT while
e-NAM linkage while in current year speech, it is onion yields went up from 6.3 MMT to 21 MMT and
announced that out of 585 major mandis under the tomato production increased from 8.1 MMT to 22 MMT.
Agricultural Produce Marketing Committee (APMC), 470

FOCUS | June 2018 | RAU’S IAS 122

Download from:- www.UPSCPDF.com


https://t.me/UPSC_PDF www.UPSCPDF.com https://t.me/UPSC_PDF

on farmers’ fields. Such storage facilities have to be


cost effective. A potato cold storage in UP, for
example, that buys power at almost Rs 10/kwh from
SEBs, can become efficient by generating solar power
at less than Rs 4/kwh. Large-scale investments in
storage will require tweaking of the Essential
Commodities Act.

x Third is linking the processing industry with organized


retailing. On an average, about one-fourth of the
However, the problem with these commodities is that produce must be processed. India with 2% of fruits
their prices collapse when their production rises sharply. and vegetable being processed is way behind on this
This is because the country lacks modern storage curve compared to most Southeast Asian countries.
facilities and the links between processing and organized Dehydrated onions, tomato puree and potato chips
retailing are very weak. As a result, farmers often end up should become cheap, so that an average household
receiving less than a fourth of what consumers pay in can use them. Processing industry adds value and
major cities. This must change. Operation Greens needs absorbs surpluses. Increasing the allocation for the
to ensure that farmers receive at least 60 per cent of food processing industry by 100 per cent in Union
what consumers pay. In the case of milk, the producers Budget for FY 2018-19 is a welcome step in this
get more than 75 per cent of what consumers pay. The context. The food processing ministry is coordinating
basic principles of Operation Flood, listed below, would the initiative of Operation Greens with other
be useful to operationalize Operation Greens as well. concerned Ministries/Departments as well as with
State Governments. By developing such forward and
x First, link major consumption centres to major backward linkages, the government can ease large
production centres with a minimal number of price fluctuations, raise farmers’ share in the price
intermediaries. The real challenge is to find the right paid by the consumer and at the same time, ensure
markets that can give them remunerative prices on a lower prices for the consumers — a win-win situation
sustainable basis. So, someone needs to map mega for all.
consuming centres and link their retail networks with
the producing centres of each commodity. Farmers The government has come up with similar programmes
can be organized in farmer producer organizations such as the Pradhan Mantri Kisan SAMPADA Yojana,
(FPOs). NABARD and SFAC (Small Farmers’ which was launched in 2016, however, Operation Green
Agribusiness Consortium) together have about 3,000 is overarching and is specifically targeted at TOP, unlike
FPOs, which could be the starting points for the SAMPADA. The targeted programmes earlier were not
aggregation of commodities, assaying, sorting, executed in a holistic manner. Operation Green ties the
grading, and even packing with bar codes, reflecting two ends and also the consumer. It is an end-to-end
their traceability. The Agricultural Produce Market solution. The NITI Aayog also expressed the view that
Committee Act will have to be changed to allow direct Operation Green will fill or at least narrow down the gaps
buying from FPOs, and giving incentives to these in the value chain logistics from farm gate to delivery
organizations, private companies and NGOs to build point with respect to pack houses, cold chains and
back end infrastructure as was done in the case of primary processing facilities. The Operation Greens
milk. The announcement of tax concessions to FPOs would promote farmer producers
for five years is a welcome step in that direction, if it organizations/companies, agri-logistics and processing
encourages building such critical infrastructure. facilities. But then there’s the risk of spreading resources
too thinly across components and states, rather than
x Second is the investment in logistics, starting with having a focused mission scalable over space and time, it
modern warehouses, that can minimize wastage. An is replicating success of Operation Flood.
example is of cold storage for onions, where wastage
is reduced to less than 10 per cent, compared to the The Union Budget provides ₹ 500 crore for Operation
25-30 per cent wastage in traditional storage facilities Greens for FY 2018-19. Apart from adequate funding, the

FOCUS | June 2018 | RAU’S IAS 123

Download from:- www.UPSCPDF.com


https://t.me/UPSC_PDF www.UPSCPDF.com https://t.me/UPSC_PDF

success of operation green will depend upon how – in identified cities subject to availability of total
effectively it will address the issues relating to the size; production. These markets have to be developed
range; technology; markets; and institution(s) involved in through a viable business model.
TOP. The Ministry implementing the Operation Green
may consider the following suggestions while allocating x There does not seem to be any institution today that
the earmarked budget under the operation: can implement Operation Greens similar to what the
National Dairy Development Board for milk did in
x Given the fact that India’s production of potatoes in Operation Flood. Government need to identify and
2016-17 was estimated at 48.61 MMT, with 22.43 MMT develop a similar institution (say NAFED) with
for onions and 20.71 MMT for tomatoes, the funds adequate autonomy to operate in commercial
may be allocated to these crops in the same order settings.

x As there are so many varieties are available of TOP, x The TOP project will further expand the processing of
grown in different climatic conditions and in different Onion, Potato, and Tomato and to meet the
seasons, making marketing intervention in all might increasing demand from processing units,
result in thinly distribution of resources amongst Government, with suitable diversification
them. In the beginning, the Operation Green may programmes embedded with adequate incentives,
target connecting the bulk producing centres of TOP need to encourage to farmers to increase production
with bulk consuming Centres in identified large cities under the TOP project.
for effective utilization of resources.
The Ministry of the Food Processing Industries needs to
x In horticultural produce, viable technology options for work out the outline of the Operation Greens instantly. It
long-term storage, transport, and processing need to seems that the Ministry has already started consultations
be developed or fine-tuned; keeping in view the with stakeholders to determine the contours of
energy intensity of available technologies, and rising Operation Greens, with a focus on preventing distress
fuel prices. sale by tomato, potato and onion farmers. It is expected
that a vibrant strategy keeping in view the contours listed
x The markets seamlessly connecting producers to above will be rolled out soon by the Government of India.
consumers for TOP – whether processed or otherwise

INDIA’S COASTAL BORDER MANAGEMENT


#Security
ANKUR SHARMA, EDITORIAL TEAM
& EDUCATOR: DAILY NEWS SIMPLIFIED

entire coast of India falls within the tropics. The nine


Overview of India’s coastal states are Gujarat, Maharashtra, Goa, Karnataka,
Kerala, Tamil Nadu, Andhra Pradesh, Odisha and West
Coastal Borders Bengal. India’s exclusive economic zone (EEZ) is of 200

India has a coastline of 7,517 km, of which the mainland nm from its coastline with an area of over 2 million sq

accounts for 5,422 km. The Lakshadweep coast extends km.

for 132 km and the Andaman and Nicobar Islands have a India’s long coastline presents a variety of security
coastline of 1,962 km. The Indian coastline is distributed challenges such as illegal landing of arms and explosives
among nine coastal states and four UTs, and almost the

FOCUS | June 2018 | RAU’S IAS 124

Download from:- www.UPSCPDF.com


https://t.me/UPSC_PDF www.UPSCPDF.com https://t.me/UPSC_PDF

at isolated spots on the coast, infiltration/ex-filtration of also responsible for maintaining maritime surveillance
anti-national elements, use of the sea and off shore across India’s two million sq. km Exclusive Economic
islands for criminal activities, smuggling of consumer Zone (EEZ). The ICG is also responsible for overall
and intermediate goods through sea routes, presence of coordination between the Central and state agencies in
vital industrial and defence installations near the coast, matters relating to coastal security. The coastal areas are
among other forms of security threats. The inability of safeguarded by the police forces of the respective
forming physical barriers on the coast, as compared to coastal states and UTs, which have jurisdiction of up to
land borders, also enhance the vulnerability of the 12 nautical miles (nm) from the coast. The ICG and the
coasts to illegal cross border activities. Moreover, India’s Indian Navy have jurisdiction over the entire maritime
coastal border security includes the security of India’s zone up to 200 nm, including the 12 nm of territorial
Island territories of Andaman & Nicobar islands and waters. The Centre government has a ‘Definitive five-
Lakshawadeep. year Action Programme’ for the Coast Guard. The aim is
to make the Coast Guard a 175-ship and 110-aircraft
Institutional Framework force by 2022 to plug the operational gaps and
strengthen its capabilities to safeguard coastal security,
island territories, offshore assets and the marine
for Coastal Border environment, as well as undertake anti-piracy, anti-
smuggling, oil-spill and pollution-control operations.
Security
The Department of Border Management under the Initiatives to Maintain
Ministry of Home Affairs focusses on the issues relating
to the management of international land and coastal Security at Coastal
borders, strengthening of border policing and guarding,
creation of infrastructure such as roads, fencing and
flood lighting on the borders, and implementation of the
Borders
Border Area Development Programme. The MHA is A surveillance system called the Coastal Surveillance
implementing supplementary Coastal Security Scheme Network (CSN), comprising a chain of static sensors
(CSS) to strengthen the marine police of nine coastal having radars, an automatic identification system (AIS),
states and four UTs to enhance surveillance and close day/night cameras and metrological sensors has been
water patrolling in the coastal areas. The established at several locations along the coastline and
functions/responsibilities of the department for coastal island territories by the ICG. In order to achieve near
regions include the following: gapfree surveillance of the entire coastline, additional
radar stations and mobile surveillance systems, apart
x All matters relating to coastal border, including the
from vessel traffic management system (VTMS)
island territories of Andaman and Nicobar and
connectivity at the Gulf of Kutch and Gulf of Khambhat,
Lakshadweep.
are being installed. In 2011, the government approved
x Strengthening of border policing, surveillance and the deployment of radiological detection equipment
patrolling of all coastal borders (RDE) at the 16 major ports and has been extended to
x Creation of infrastructure along the coastal belt more ports afterwards.

x Implementation of Coastal Security Schemes (CSS) The Central government in association with State & UT
government has been issuing biometric card to Indian
According to the Maritime Zone Act, 1976, the maritime
fishermen and has also established a National Marine
zones of India are divided into five Coast Guard regions,
Fishers Database (NMFD) which can be accessed by all
with the Indian Coast Guard (ICG) responsible for the
authorised agencies, both at the Central and state/UT
enforcement of each maritime zones.
levels. The ICG has also been taking several steps to
The ICG is a multi-mission organisation that is educate fishermen communities in the coastal areas of
responsible for coastal security in territorial waters, the country to help them ward off threats from the sea
including areas to be patrolled by the coastal police. It is as they serve as the first line formation in India’s coastal

FOCUS | June 2018 | RAU’S IAS 125

Download from:- www.UPSCPDF.com


https://t.me/UPSC_PDF www.UPSCPDF.com https://t.me/UPSC_PDF

security architecture and thereby can serve as the ‘eyes maritime treaty. The purpose of the set of regulations is
and ears’ of the Indian coastal security mechanism. to establish a standardised framework across
international ports and ships, which then allows
Coastal Security Scheme (CSS) was launched in 2005
governments to efficiently evaluate risks and offset
across all nine coastal states and four coastal UTs. The
threats and vulnerabilities to shipping and port facilities
main objective of the scheme was to strengthen
through alterations to its three security levels and
infrastructure of the marine police force in order to
undertaking the security measures prescribed by the
improve patrolling and surveillance of the coastal areas,
Code. The ISPS Code was made mandatory in India in
especially the shallow areas close to the coast.
2004, with its implementation handled by the Company
In response to the 1993 Mumbai blasts, Operation Swan Security Officer (CSO), under the Shipping Corporation of
was launched in 1993 as a joint operation of the Indian India (SCI).
Navy and the ICG in conjunction with the respective
National Automatic Identification System is established
state administration. The primary aim of this operation
by setting up shore stations on existing lighthouses
was to prevent the unauthorised and illegal entry of men
along the Indian coast for facilitating aid to marine
and landing of arms, explosives and contraband along
navigation and tracking of SOLAS vessels. The AIS
the coast of Gujarat and Maharashtra by sea. It also
network plans to track all SOLAS compatible vessels and
focused on obtaining intelligence about unusual
this will provide an overall image of AIS-compliant
movements or activities of personnel near the coastline
vessels along the Indian coastline thereby creating
having a bearing on security and to facilitate immediate
maritime domain awareness along India’s coastline.
actions to stall attempts at violating the sea frontiers for
nefarious purposes. Under this plan, a three-tier system The Coast Guard is currently operating more than 130
was established. Along the outermost layer beyond 200 ships consisting of offshore patrol vessels, fast-patrol
nm, the Indian Navy was assigned the responsibility of vessels and pollution control vessels, hovercrafts,
patrolling the high seas. The ICG was assigned the smaller interceptor boats/crafts. The air assets consist of
responsibility of patrolling the intermediate layer Dornier maritime surveillance aircraft, Chetak choppers
between 12 and 200 nm. For the innermost layer (within and Dhruv advanced light helicopters (ALHs). The ICG
12 nm), a joint coastal patrolling team comprising and marine police are also working according to the
personnel from the Indian Navy , state police and ‘hub-and-spoke’ concept with the hub being the ICG
customs was set up to increase surveillance in shallow stations and the spoke being the Coastal Police Stations
waters, creeks and inlets which were previously (CPS). To revalidate the coastal security mechanism and
unmonitored. create awareness among the fishermen at sea, regular
boarding operations are also being conducted to
India has a Vessel Tracking Management System
validate and check the credentials of the occupants of
deployed by the Indian Navy with coastal radars being
the vessels, including their identity cards and
set up within the frameworks of the National Command
registration documents.
Control Communication Intelligence (NC3I) programme
to help counter potential infiltration from terrorists and In order to thwart the landing of terrorists through sea
pirates. The integrated radar system create a complex routes, the BSF formed its first commando unit called
network of marine tracking that is controlled by the Creek Crocodiles. The Creek Crocodile Commando unit is
Information Management and Analysis Centre (IMAC) in an elite commando force with 42 commandos
Gurugram, Haryana. responsible for manning 85 km of hostile creek area
where India shares a border with Pakistan along the
The International Ship and Port Facility Security
Rann of Kutch. The coastline is abundant with creeks
(ISPS) Code is a comprehensive set of guidelines and
that could be used by infiltrators to evade security forces
regulations established for the security of ships and port
and radars. They are equipped with all terrain vehicles
facilities. It was developed by the International Maritime
(ATVs) and fast patrol boats.
Organisation (IMO) in response to the 9/11 attacks, the
code is constituted in the International Convention for
the Safety of Life at Sea (SOLAS), an international

FOCUS | June 2018 | RAU’S IAS 126

Download from:- www.UPSCPDF.com


https://t.me/UPSC_PDF www.UPSCPDF.com https://t.me/UPSC_PDF

PUBLIC PRIVATE PARTNERSHIP


#Economy
RITESH KUMAR SINGH
FACULTY: G.S. CONTEMPORARY ISSUES, ANSWER WRITING & ESSAY,
AND EXECUTIVE EDITOR

India of 21st century has to develop at a rapid pace to the project in totality, along with the accompanying risks
bring the maximum people out of poverty by and rewards/returns, are shared on the basis of a
transforming the economy and embarking on the path predetermined, agreed formula, which is formalized
of sustained progress in all quarters and sectors of the through a contract. A PPP project is basically based on a
economy. For this to happen, Public Private Partnership significant opportunity for the private sector to innovate
(PPP) has become a new buzzword for addressing the in design, construction, service delivery, or use of an
ills, related to stagnation of growth and development in asset.
our country.
PPPs are different from privatization. While PPPs involve
Further, tracing India’s economic growth trajectory, the private management of public service through a long-
country started on a positive note on the basis of term contract between an operator and a public
planned development, by investing heavily in capital authority, privatization involves outright sale of a public
intensive industries, industrial towns, dams, and service or facility to the private sector.
premium educational institutions. But after initial gains,
the rate of growth stagnated due to vicious cycle of red
tapeism, licence raj, and lack of breathing space to
Fundamental qualities of
private entrepreneurs and enterprises. So, to unshackle
the stronghold of negativities and to create space for
a PPP project
faster growth and development, PPP as a model of High priority, government-planned projects: The
development is being promoted by the centre and the projects must have emerged from a government-led
states. planning and prioritization process. The project must be
To comprehend PPP better, let us make an effort to such that, regardless of the source of public or private
understand the entire concept of PPP in detail. capital, the government would still want the project to
be implemented quickly.
Meaning of PPP: Public-Private-Partnership or PPP is a
mode of implementing government programmes/ Genuine risk allocation: Shared risk allocation is a
schemes in partnership with the private sector. The term principal feature of a PPP project. The private sector
private in PPP encompasses all non-government must genuinely assume some risk....
agencies such as the corporate sector, voluntary Mutually valuable: Value should be for both sides,
organizations, self-help groups, partnership firms, which means government should also genuinely accept
individuals and community based organizations. PPP, some risks and not transfer the entire risk to the private
moreover, subsumes all the objectives of the service sector, and vice versa.
being provided earlier by the government, and is not
Potential Benefits of Public Private Partnerships: The
intended to compromise on them. Essentially, the shift
financial crisis of 2008-11 has brought about renewed
in emphasis is from delivering services directly, to
interest in PPP in both developed and developing
service management and coordination.
countries. Facing constraints on public resources and
Salient features of PPP: Not all projects with private fiscal space, while recognizing the importance of
sector participation are PPP projects. Essentially, PPPs investment in infrastructure to help their economies
are those ventures in which the resources required by

FOCUS | June 2018 | RAU’S IAS 127

Download from:- www.UPSCPDF.com


https://t.me/UPSC_PDF www.UPSCPDF.com https://t.me/UPSC_PDF

grow, governments are increasingly turning to the government should therefore determine whether the
private sector as an alternative additional source of greater costs involved are justified.
funding to meet the funding gap. While recent attention
x There is a cost attached to debt – While private sector
has been focused on fiscal leveraging of projects,
can make it easier to get finance, finance will only be
governments look to the private sector to help them
available where the operating cash flows of the
deliver infrastructure for a number of other reasons:
project company are expected to provide a return on
x Exploring PPPs as a way of introducing private sector investment (i.e., the cost has to be borne either by
technology and innovation in providing better public the customers or the government through subsidies,
services through improved operational efficiency. etc.)

x Incentivizing the private sector to deliver projects on x Some projects may be more politically or socially
time and within budgets. challenging to introduce and implement than others -
particularly if there is an existing public sector
x Imposing budgetary certainty by setting present and
workforce that fears being transferred to the private
the future costs of infrastructure projects over time.
sector, if significant tariff increases are required to
x Utilizing, PPPs as a way of developing local private make the project viable, if there are significant land
sector capabilities through joint ownership with large or resettlement issues, etc.
international firms, as well as sub-contracting
x There is no unlimited risk bearing – private firms (and
opportunities for local firms in areas such as civil
their lenders) will be cautious about accepting major
works, electrical works, facilities management,
risks beyond their control, such as exchange rate
security services, cleaning services, maintenance
risks/risk of existing assets. If they bear these risks
services, etc.
then their price for the service will reflect this. Private
x Using PPPs as a way of gradually exposing state sector will also expect a significant level of control
owned enterprises and government to increasing over operations if it is to accept significant risks.
level of private sector participation and structuring
x Private sector will do what it is paid to do and no
PPPs in a way so as to ensure transfer of skills leading
more than that – therefore incentives and
to capacitated entities that can eventually export
performance requirements need to be clearly set out
their competencies by bidding for projects/ joint
in the contract. Focus should be on performance
ventures.
requirements that are out-put based and relatively
x Creating diversification in the economy by making the easy to monitor.
country more competitive in terms of its facilitating
x Government responsibility continues – citizens will
infrastructure base as well as giving a boost to its
continue to hold government accountable for quality
business and industry associated with infrastructure
of utility services. Government will also need to retain
development.
sufficient expertise, whether the implementing
x Supplementing limited public sector capacities to agency and/ or via a regulatory body, to be able to
meet the growing demand for infrastructure understand the PPP arrangements, to carry out its
development. own obligations under the PPP agreement and to
x Extracting long-term value-for-money through monitor performance of the private sector and
appropriate risk transfer to the private sector over enforce its obligations.
the life of the project – from design/ construction to
operations/ maintenance. PPP bridging the gap to
Potential Risks of Public Private Partnerships: There
are a number of potential risks associated with PPPs fund India’s growth story
such as given hereunder:
Government of India’s Definition: “Public Private
x Development, bidding and ongoing costs in PPP Partnership (PPP) Project means a project based on a
projects are likely to be greater than for traditional contract or concession agreement, between a
government procurement processes - the Government or statutory entity on the one side and a

FOCUS | June 2018 | RAU’S IAS 128

Download from:- www.UPSCPDF.com


https://t.me/UPSC_PDF www.UPSCPDF.com https://t.me/UPSC_PDF

private sector company on the other side, for delivering sector efficiencies through contracts based on
an infrastructure service on payment of user charges”. availability/performance payments.

India had a few notable PPPs as early as the 19th x Performance Based Management/ Maintenance
century. The Great Indian Peninsular Railway Company contracts- In an environment of constrained
operating between Bombay (now Mumbai) and Thana economic resources, PPP that improves efficiency is
(now Thane) (1853), the Bombay Tramway Company more relevant. PPP models such as performance
running tramway services in Bombay (1874), and the based management/maintenance contracts are
power generation and distribution companies in encouraged. Sectors amenable for such models
Bombay and Calcutta (now Kolkata) in the early 20th include water supply, sanitation, solid waste
century are some of the earliest examples of PPP in management, road maintenance etc.
India.
x Modified Design-Build (Turnkey) Contracts: In
Public-private partnership (PPP) is emerging as the new traditional Design-Build (DB) contract, private
success route in India’s attempts to build world-class contractor is engaged for a fixed-fee payment on
infrastructure and boost social sector development. completion. The primary benefits of DB contracts
Over the last decade, policymakers at both central and include time and cost savings, efficient risk-sharing
state levels have been increasingly focusing on and improved quality. A new Turnkey DB approach
infrastructure investments so as to enable fast paced with the payments linked to achievement of tangible
economic growth. PPP, in fact has been identified as the intermediate construction milestones (instead of
key to policymakers’ attempts to create the requisite lump-sum payment on completion) and short period
infrastructure for enabling double-digit GDP growth and maintenance / repair responsibilities is being
enhancing people’s welfare. In fact, the government considered.
expects private investments to contribute 50 per cent to
One of the key elements of successful PPP projects in
total infrastructure investments (worth USD 1 trillion) in
India is clear understanding of the proposed asset and
India the next five years. It will be no surprise if a large
prudent sharing of risks and rewards associated with the
chunk of these investments are directed through the
asset, and to ward off poor preparations, inappropriate
PPP route.
business models and fiscal uncertainties to vested
interests leading to development of skewed qualification
PPP Models supported by criteria.

If implemented in right earnest, PPPs have the potential


the Government in India to create synergies between public authorities and
private sector companies; provide value for money for
x User-Fee Based BOT models - Medium to large scale
the taxpayer through optimal risk transfer and risk
PPPs have been awarded mainly in the energy and
management; enable speedy, efficient and cost effective
transport sub-sectors (roads, ports and airports).
delivery of projects; and accountability for the provision
Although there are variations in approaches, over the
and delivery of quality public services. PPP can also help
years the PPP model has been veering towards
in innovation and diversity in the provision of public
competitively bid concessions where costs are
services along-with effective utilisation of state assets for
recovered mainly through user charges.
the benefit of all users of public services. In short, PPP
x Annuity Based BOT models- In sectors/projects not has the right ingredients to chart a uniform and holistic
amenable for sizeable cost recovery through user course for the growth and development of our country.
charges, owing to socio-political-affordability
considerations, such as in rural, urban, health and
education sectors, the government harnesses private

FOCUS | June 2018 | RAU’S IAS 129

Download from:- www.UPSCPDF.com


https://t.me/UPSC_PDF www.UPSCPDF.com https://t.me/UPSC_PDF

lEAD ESSAYS

ARE WORKING WOMEN IN INDIA GETTING FAIR


DEAL?
ANISHA TIWARI
EX-RAU’S IAS STUDENT

While women are respected for sustaining the family and


‘I measure the progress of a community by the degree of
having ‘Shakti’ or mother power, they still seem to be
progress its women have achieved’ proclaimed Babasaheb
submissive to their husband’s interests. The Indian
Ambedkar, the architect of the Indian Constitution.
working woman thus does not seem to be getting a fair
The 21st century has seen greater strides in achieving deal in managing her relationships at home and her work
equality status in the work-sphere between the sexes. commitments. Often, a woman’s wants are subjugated to
But with it have also increased the frequency of hassles the interests of her better half in a patriarchal society.
which the modern Indian woman juggling work and But this situation gains even more significance in the
home faces. It would be an understatement to say that modern age, when substantial women empowerment
the Indian working woman faces a multitude of problems has resulted in both the man and woman in the family
in the social, economic, political, domestic, psychological undertaking profitable employment. Even in such cases,
sphere and that she is not facing a fair deal. This essay the scenario still seems to be that a woman must bend
attempts to tackle this very important issue affecting herself and accompany her husband to a different place
nearly half of our 1.25 billion population. The Indian in case of a transfer of the place of occupation.
Constitution itself tried to set up an example, by
The burden of taking care of the household seems to
including as a provision in the Directive Principles of
have been entrusted solely to the women who are
State Policy, Article 51A (e), which exhortedsociety to
responsible for all the manual work from cooking to
renounce practices derogatory to the status of women.
cleaning the home. It would not be an exaggeration to
Any analysis of the status of Indian working women has say that they are looked down upon by the male section
to first start by looking at the attitudes prevalent in one of the population for failing to contribute economically to
of our prime social institution, the Indian family. the family. Also, when the tide has shifted in recent
Women are the prime creators and progenitors of the times, with more and more women taking up gainful
institution of the family, having bestowed by god the employment outside the confines of the home, they are
power to beget offspring. For this reason, the position of again facing hurdles much to their distress. The menfolk
the mother figure in the Indian cultural sphere is a very are unwilling to contribute their part to finishing the
strong one and obtains a lot of respect. Traditionally, household work and as a result women are entrusted
women have been seen as the group responsible for with the double burden of doing household work and
taking care of the children while the men are delegated their official employment outside of home. The strong
the task of ensuring the financial stability. So the psychological bias of the Indian male mind, which
dominant idea through the centuries seems to have been considers work inside the household as unworthy and
the women staying at home to take care of the child beneath his male ego is the reason for this prejudice. A
while the father ventures out to work. fundamental shift in this attitude needs to occur if the
Indian woman is to get an honourable treatment as the
21st century working woman.

FOCUS | June 2018 | RAU’S IAS 130

Download from:- www.UPSCPDF.com


https://t.me/UPSC_PDF www.UPSCPDF.com https://t.me/UPSC_PDF

India is undergoing a transition of sorts in the workplace Bhattacharya, Biocon CEO Kiran-Mazumdar Shaw,
atmosphere. From being a society in which economic PepsiCO CEO Indra Nooyi are examples of successful
work outside the home was conducted solely by men, Indian women. But the supposed ‘glass-ceiling’ exists in
steady strides have been taken forward to bring various ways and manifests itself in different forms to
evermore greater women participation in various prevent women from rising up to the higher echelons of
occupations through the spread of literacy, educational an organization. Owing to stereotypes, women are
opportunities and improved health indicators. At the evaluated and doubted as to whether they would be
time of Independence, female literacy rate used to be a suitable to occupy a powerful and decision-making
paltry 7%. It has now achieved the respectable figure of position. This ends up as a vicious cycle and women do
64% according to the 2011 Census. Greater spread of not rise above a particular level in an organization as her
education has brought with it increased awareness about care-taking abilities so naturally suited to building a
the plight of Indian women. home are not seen as a proper fit in the corporate world.
As women do not rise up to decision-making positions
Household work is officially not considered an ‘economic
and are not hence given a say in appointments, the
good’ and is therefore not included while calculating and
attitude persists for succeeding generations.
estimating the Gross Domestic Product (GDP) of goods
and services produced in a country. This is an extreme Psychological studies have documented that women
injustice as it implies that the back-breaking work within bring in their own particular skill set and ability to the
a house often entails to somehow insignificant in the global business processes and they are an asset to
development of a country. It needs scarcely be reminded increasing the resource allocation to the weaker sections,
that it is only the nourishment and hard-work which a the women and children. They are also credible partners
woman puts into bringing up her children and building in trying to build a more equitable society which does not
her home, that go on to become future architects of the have the huge income inequality which exists presently.
nation. The 33% reservation for women in local government
bodies such as panchayats and municipalities has been
While women at workplaces outside the home are seen
documented to have mobilized more benefits to women.
to be positively working towards increasing the economic
Similar effect could be observed if the Women’s
output of a nation, women inside the home who perform
Reservation Bill is passed by the Parliament for providing
a more important task of bringing up the future of the
reservation in the central and state legislatures.
country and nurturing obedient and law-abiding citizens
are put down as simply being ‘housewives’. And women India is benefited every year by the number of women
who absorb themselves in the office are frowned upon who enter its administrative and police services and
for neglecting their family, children and home. It seems other allied government services. Their entry will pave
that women are caught between a rock and a hard place the way for a government which identifies and takes care
in India. of inclusive development. But, the basic struggle of
juggling their work at home and the office may still
Whether it be the taking of ‘maternity leave’ to ensure the
persist, if a shift in the societal behaviour does not occur.
successful delivery of her child or taking off from work
It is an ordeal for the working woman to obtain safe and
early to care for a sick child at home, these practices are
unmolested conduct from the workplace to home. The
despised as detrimental to the career progress and
public environment is so vitiated against single women
prospects of a woman in the male-dominated culture of
caught alone without male support that female freedom
the workplace. Certain studies have documented the fact
is intensely circumscribed in India. The 2012 Delhi Rape
that women are paid much lesser than an equally
case and the Mumbai Shakti Mills Compound Rape cases
talented man for comparable work, despite the Equal
is a powerful reminder of this fact. It is difficult for a
Remuneration Act enacted by the Indian Government in
woman to venture out of her home alone. Any discussion
1976. This workplace inequality is narrowing but still
on the complications faced by the Indian working woman
persists.
is incomplete without analysing the socio-political and
A positive development occurred in this area recently socio-economic problems concerned with the
with the tabling of a bill to quantify the contribution of a employment of rural Indian women. The informal and
woman to household work in the Parliament. There are unorganized sector accounts for most of the
women who head Fortune 500 companies. There have employment numbers in our country. Added to this is the
been women head of corporations in India as well. The increasing casualization of the workforce in the wake of
present SBI (State Bank of India) chairperson Arundhati the reforms of 1991 which ushered in Liberalization,

FOCUS | June 2018 | RAU’S IAS 131

Download from:- www.UPSCPDF.com


https://t.me/UPSC_PDF www.UPSCPDF.com https://t.me/UPSC_PDF

Privatization and Globalization. The number of workers has had female heads of State and women Chief
employed on a contractual basis has now increased to Ministers. Thus in a sense, Eastern civilization can be
75% of the regular workforce. This means more job conceived to be much more liberal in accepting women
insecurity and a smaller social security net. Women in leaders in the political sphere. But in contradiction to
rural areas thus are affected much more by globalization this, the status of crime against women in the developed
since the informal sector is much larger in rural India western world is much lesser than Indian conditions.
than in urban India. For example, the influx of cheap
Indian culture provides glimpses of a time when women
agricultural products into the Indian market has affected
were liberated and there existed a much more egalitarian
Indian rural women landless labourers much more. This
society than what is now. Examining the position in
is because men can be much more mobile in their search
ancient India it is clear from the evidence in the Rig-Veda,
for work and migrate to other areas in search of
the earliest literature of the Indo-Aryans, that women
employment. But women are bound by societal
held an honourable place in early Indian society. There
constraints and family responsibilities, like taking care of
were a few Rig Vedic hymns composed by women.
children and old parents. Hence they would find it
Women had access to the highest knowledge and could
difficult to migrate.
participate in all religious ceremonies. In domestic life
The traditional village society may not intervene in what too she was respected and there is no suggestion of
seems to be a husband wife issue unlike in urban areas seclusion of women and child marriage. Buddhism also
where modernization and westernization have brought accorded to women a position approximating to equality.
with it increasing assertiveness and confidence to stand The highest spiritual states were within the reach of both
up for one’s rights in the part of the urban Indian men and women and the latter needed no masculine
woman. Thus, in this aspect the condition of the rural assistance or priestly intermediary to achieve them.
Indian woman seems to be much worse than the urban
Increased women participation in the political sphere,
Indian woman in this case.
paid maternity leave, day care centres for children, baby
True emancipation would be reached when women are care centres or crèches at the workplace, increased
given complete freedom to unleash their creative gender sensitization are some of the steps can be taken
energies. Micro-finance institutions, such as Bharatiya to improve the status of women. Importantly, the next
Mahila Bank that promote women entrepreneurship are generation of children must be made sensitive to the
a good start. The Self-Employed Women’s Association is hardships faced by their mothers at the home and
undertaking a stellar role in this field. Rural women workplace. This would improve the current bottom
should be empowered to be economically independent ranking of India in the Gender Inequality Index.
of their husbands. Until then, Indian rural and urban
If some women in our country are blessed with such
women would be facing a multitude of problems on the
supportive partners, it is in the capacity of every person
home and work front and would not be getting a fair
in this country to understand that working women are
deal.
getting facing a fair share and to be concurrently
The political sphere can also be considered to be a supportive. The day is not long off when this shift will
workplace of sorts and a public service office. If our occur and change the status of Indian women for the
country has such flourishing women political leaders it is better.
equally possible to attain equality in all workplaces in the
As Rabindranath Tagore had sung not long ago, ‘Where the
country. The Prevention of Sexual Harassment at the
mind is without fear and the head held high; …….into that
Workplace Act, 2013 is a good step in this direction. In
heaven of freedom my father, let my country awake.’
accordance with our liberal democratic constitution, India

FOCUS | June 2018 | RAU’S IAS 132

Download from:- www.UPSCPDF.com


https://t.me/UPSC_PDF www.UPSCPDF.com https://t.me/UPSC_PDF

Part Three

practiCe
ZONE
mcqs . descriptive questions .
questions based on editorials

Download from:- www.UPSCPDF.com


https://t.me/UPSC_PDF www.UPSCPDF.com https://t.me/UPSC_PDF

Here are given 50 Multiple Choice Questions (MCQs)

Multiple Choice Questions for self-practice. These questions have been framed
from this issue itself. So, a reader is expected to
attempt the questions, and also refer to the Answer
(MCQs) for prelims gs paper i Key given after the MCQs. These are helpful for General
Studies- Paper I (Preliminary Examination).

Q1. Consider the following statements: Select the correct answer using the code given
1. Maulana Abul Kalam Azad was the India’s first below:
Education Minister. (a) 1 only
2. He was awarded Bharat Ratna, India’s highest (b) 1 and 2
civilian award. (c) 2 only
Which of the statements given above is/are (d) 1, 2 and 3
correct?
(a) 1 only Q5. Which of the following books is/are written by
(b) 2 only Lokmanya Tilak?
(c) Both 1 and 2 1. The Arctic Home of the Vedas
(d) Neither 1 nor 2 2. Gita Rahasya
Select the correct answer using the code given
Q2. India Wins Freedom was written by which of below:
the following persons? (a) 1 only
(a) Dr. Rajendra Prasad (b) 2 only
(b) Pandit Jawahar Lal Nehru (c) Both 1 and 2
(c) Bhagat Singh (d) Neither 1 nor 2
(d) None of the above
Q6. Consider the following statements:
Q3. Consider the following statements: 1. Vachana literature is a form of writing in Kannada
1. Lord Irwin called Tilak the 'Father of Unrest in that evolved in the 17th century as a part of the
India. Veerashaiva movement.
2. To foster Indians’ sense of pride, Tilak began 2. They carry profound philosophy and thought
celebrating the Ganapati Pooja festival and the provoking ideas.
Shivaji festival. Which of the statements given above is/are
Which of the statements given above is/are correct?
correct? (a) 1 only
(a) 1 only (b) 2 only
(b) 2 only (c) Both 1 and 2
(c) Both 1 and 2 (d) Neither 1 nor 2
(d) Neither 1 nor 2
Q7. The prehistoric rock paintings of
Q4. Which of the following persons played Pandavulagutta is located in which of the
important role in concluding Lucknow Pact? following states?
1. Mohammed Ali Jinnah (a) Telangana
2. Lokmanya Tilak (b) Tamil Nadu
3. Rajkumar Shukla (c) Kerala

FOCUS | June 2018 | RAU’S IAS 134

Download from:- www.UPSCPDF.com


https://t.me/UPSC_PDF www.UPSCPDF.com https://t.me/UPSC_PDF

(d) Karnataka 4. Rotten Potatoes


Select the correct answer using the code given
Q8. The prehistoric rock paintings of below:
Pandavulagutta is home to: (a) 1 and 2 only
th
1. An 8 century inscription of the Rashtrakuta (b) 1,2 and 3
period. (c) 2, 3 and 4
2. Painted frescoes from the 12th century Kakatiya (d) 1, 2, 3 and 4
empire.
Select the correct answer using the code given Q12. Which of the following is/are the objective(s)
below: of National Wind-solar hybrid policy?
(a) 1 only 1. Promotion of large grid-connected wind-solar PV
(b) 2 only hybrid system.
(c) Both 1 and 2 2. To encourage new technologies and methods
(d) Neither 1 nor 2 involving the combined operation of wind and
solar PV plants.
Q9. Consider the following statements about Select the correct answer using the code given
Dantidurga: below:
1. The Rashtrakutas kingdom was founded by (a) 1 only
Dantidurga. (b) 2 only
2. He established his capital at Manyakhet. (c) Both 1 and 2
Which of the statements given above is/are (d) Neither 1 nor 2
correct?
(a) 1 only Q13. Consider the following statements:
(b) 2 only 1. PPA (Power Purchase Agreement) is a contract
(c) Both 1 and 2 between the purchaser of electricity and
(d) Neither 1 nor 2 electricity generator setting out the terms and
price for supplying electricity.

Q10. Which of the following persons were the key 2. The recent auctions for solar power procurement
leaders of Sadharan Brahmo Samaj? have led to the discovery of very high tariffs.

1. Ananda Mohan Bose Which of the statements given above is/are


correct?
2. Shibchandra Deb
(a) 1 only
3. Umeshchandra Datta
(b) 2 only
Select the correct answer using the code given
below: (c) Both 1 and 2

(a) 1 only (d) Neither 1 nor 2

(b) 2 only
(c) 1 and 2 Q14. Consider the following statements regarding
‘Pradhan Mantri Jan VikasKaryakaram’:
(d) 1, 2 and 3
1. The Programme aims to address the development
deficits in the identified minority concentration
Q11. As per the National Policy on Biofuels, 2018
areas.
which of the following can be used as raw
2. The programme has been restructured after
material for ethanol production?
renaming the programme Multi-Sectoral
1. Sugarcane Juice
Development Programme (MsDP).
2. Cassava
Which of the following statements is/are correct?
3. Broken rice

FOCUS | June 2018 | RAU’S IAS 135

Download from:- www.UPSCPDF.com


https://t.me/UPSC_PDF www.UPSCPDF.com https://t.me/UPSC_PDF

(a) 1 only Q18. Which of the following is/are the objectives of


(b) 2 only ‘Integrated Scheme on Agriculture
(c) Both 1 and 2 Cooperation’?

(d) Neither 1 nor 2 1. To provide financial assistance for improving the


economic conditions of cooperatives
2. To provide infrastructure facilities for grading,
Q15. Consider the following statements regarding
standardization and quality certification of
‘Universal Service Obligation Fund’:
agricultural produce
1. The Indian Telegraph (Amendment) Act, 2003
3. To establish a nationwide marketing information
giving statutory status to the Universal Service
network
Obligation Fund (USOF)
Select the correct answer using the code given
2. The Universal Service Obligation (USO is raised
below:
through a 'Universal Access Levy’ (UAL).
(a) 1 only
Which of the following statements is/are correct?
(b) 1 and 2
(a) 1 only
(c) 2 and 3
(b) 2 only
(d) 1, 2 and 3
(c) Both 1and 2
(d) Neither 1 nor 2
Q19. Consider the following statements regarding
Van DhanVikas Kendra (VDVK):
Q16. Which of the following is/are the component
1. Van DhanVikasKendras are being established in
of Green Revolution- Krishonnati Yojana?
tribal districts of only those states mentioned in
1. Mission for Integrated Development of
Scheduled VIth of the Indian Constitution.
Horticulture Crops
2. Tribal Cooperative Marketing Development
2. National Mission for Sustainable Agriculture
Federation of India Limited (TRIFED) is the Nodal
3. Sub-Mission on Agriculture Mechanisation
Agency to implement VDVKs at the national level.
Select the correct answer using the code given
Which of the statements given above is/are
below:
correct?
(a) 1 only
(a) 1 only
(b) 1 and 2
(b) 2 only
(c) 2 and 3
(c) Both 1 and 2
(d) 1, 2 and 3
(d) Neither 1 nor 2

Q17. Which of the following is/are the objectives of


Q20. Consider the following statements regarding
‘National e-governance Plan (NeGP-A)’?
The Tribal Cooperative Marketing
1. To bring farmer centricity & service orientation to Development Federation of India (TRIFED):
the programmes
1. It functions under the administrative control of
2. To enhance reach & impact of extension services Ministry of Tribal Affairs, Govt. of India.
3. To improve access of farmers to information & 2. TRIFED has its Head Office located in New Delhi
services throughout crop-cycle and has a network of Regional Offices located at
Select the correct answer using the code given various places in the country.
below: Which of the statements given above is/are
(a) 1 only correct?
(b) 1 and 2 (a) 1 only
(c) 2 and 3 (b) 2 only
(d) 1, 2 and 3 (c) Both 1 and 2
(d) Neither 1 nor 2

FOCUS | June 2018 | RAU’S IAS 136

Download from:- www.UPSCPDF.com


https://t.me/UPSC_PDF www.UPSCPDF.com https://t.me/UPSC_PDF

Q21. Arun-III is ahydroelectric project between India (d) Uranium exports from US to India would without
and which of the following country? the 3rd tier of US verification
(a) Nepal
(b) Bhutan Q27. Which of the following are the parameters of
(c) Afghanistan the proposed measurement of Green GDP in
(d) Bangladesh India?
1. Farmland

Q22. A canal connecting Pacific ocean and Atlantic 2. Wildlife


Ocean is located in which of the following 3. Emission patterns
country? Select from codes given below:
(a) Honduras (a) 3 only
(b) Panama (b) 2 and 3
(c) Columbia (c) 1 and 2
(d) Belize (d) 1, 2 and 3

Q23. Which of the following correctly identifies the Q28. Which of the following summit and summit
location of India’s first cyber security cluster? partner of India correctly matched?
(a) Chennai 1. Sochi Summit - Russia
(b) Bangalore 2. Wuhan Summit - Japan
(c) Hyderabad Select the correct answer from the code given
(d) Gurugram below:
(a) 1 only

Q24. Natanz nuclear enrichment plant has recently (b) 2 only


been in the news. It is located in (c) Both 1 and 2
(a) North Korea (d) Neither 1 nor 2
(b) Pakistan
(c) Israel Q29. Operation All out was recently in the news. It
(d) Iran is associated with which of the following
states in India?

Q25. S-400 Weapons Defence System belongs to (a) Tamil Nadu


which of the following countries? (b) Nagaland
(a) Russia (c) Jammu & Kashmir

(b) Israel (d) Tripura


(c) United States
(d) China Q30. Artificial Intelligence Taskforce has
established under which of the following
ministries?
Q26. CAATSA law of US has recently been in the
news. Which of the following correctly (a) Ministry of Home Affairs
explains its implication on India? (b) Ministry of Electronics & Information Technology
(a) H1-B visa applications of Indians would be (c) Ministry of Defence
processed with priority (d) None of the Above
(b) India’s defence engagement with Russia would be
sanctioned Q31. Which of the following can be said to be the
(c) 123 agreement with US would need to be re- broad framework of The Real Estate
negotiated (Regulation and Development) Act, 2016?

FOCUS | June 2018 | RAU’S IAS 137

Download from:- www.UPSCPDF.com


https://t.me/UPSC_PDF www.UPSCPDF.com https://t.me/UPSC_PDF

1. Optional registration of projects with the 1. Article 355 imposes an obligation upon the Centre
Authority. to ensure that each state government is carried
2. Mandatory Public Disclosure of all project details on in accordance with the constitution.
with Real Estate Authority. 2. The Executive Power of a State shall be vested in
3. Optional registration of Real Estate Agents. the Council of Ministers.
Select the correct answer using the code given Which of the statements given above is/are
below: correct?

(a) 1 and 3 (a) 1 only


(b) 2 only (b) 2 only
(c) 3 only 2 (c) Both 1 and 2
(d) None of the above (d) Neither 1 nor 2

Q32. Consider the following statements about Q35. Sarkaria Commission can be associated with
powers of Real Estate Regulatory Authority? which of the following?
1. To impose penalty or interest regarding 1. Centre-State relationship
contravention of obligations by the promoters. 2. Discretionary powers of the governor
2. It can refer matters to Competition Commission 3. Inter-state Council
of India regarding distortion of free competition in Select the correct answer using the code given
the real estate market. below:
3. It can call for information, record reasons but (a) 1 only
cannot conduct investigations on any complaints. (b) 2 only
Which of the statements given above is/are (c) 1 and 2
correct?
(d) 1, 2 and 3
(a) 1 and 3
(b) 2 and 3
Q36. Which of the following terms can be
(c) 1 only associated with Protection of Women from
(d) 1 and 2 Domestic Violence Act, 2005?
1. Domestic Relationship
Q33. Which of the following statements regarding 2. Shared Household
discretionary power of Governor is/are 3. Shelter Homes
correct?
4. Dowry
1. The Government of India Act, 1935 did not grant
5. Carpet Area
any discretionary powers to the Governor.
Select the correct answer using the code given
2. The Constitution of India does not clearly
below:
mentions about the nature of discretionary
(a) 1, 3 and 4
powers of Governor
(b) 2, 3, 4 and 5
Select the correct answer using the code given
(c) 1, 2, 3 and 4
below:
(d) 1, 3, 4 and 5
(a) 1 only
(b) 2 only
Q37. Parliamentary privilege has been accorded to
(c) Both 1 and 2
which of the following?
(d) Neither 1 nor 2
1. Speech made on the floor of the House
2. Vote given on any legislation
Q34. Consider the following statements:
3. Publication of reports of Parliamentary
Committee

FOCUS | June 2018 | RAU’S IAS 138

Download from:- www.UPSCPDF.com


https://t.me/UPSC_PDF www.UPSCPDF.com https://t.me/UPSC_PDF

Select the correct answer using the code given (c) Both 1 and 2
below: (d) Neither 1 nor 2
(a) 1 and 2
(b) 2 and 3 Q41. Which of the following are OZONE depleting
(c) 1 and 3 substance:
(d) 1, 2 and 3 1. Hydrochlorofluorocarbons
2. Hydrobromofluorocarbons
Q38. Which of the following can be termed as Public 3. Hydrofluorocarbons
Document? Select the correct answer using the codes given
1. Documents forming the acts or records of the acts below :
2. Records of Tribunals (a) 1 and 2
3. Official Reports of unlisted companies (b) 1 and 3
Select the correct answer using the code given (c) 2 only
below: (d) 1,2 and 3
(a) 1 and 2
(b) 2 and 3 Q42. Talanoa Dialogue was recently in news.
(c) 1 and 3 Talanoa refers to :
(d) 1, 2 and 3 (a) Talanoa is a place in pacific island
(b) Talanoa is religious belief of tribes of pacific
Q39. Consider the following statements about region.
Assam Accord: (c) Talanoa is an awareness generation campaign
1. It was signed between Government of India, state related to environmental conservation
government of Assam and the leaders of Assam (d) Talanoa is a traditional word used in Fiji and
movement. across the Pacific to reflect a process of inclusive,
2. As per the Accord, 1st January, 1966 shall be the participatory and transparent dialogue.
base date and year for purposes of detection and
deletion of foreigners. Q43. Which of the following countries does not
Which of the statements given above is/are surround Lake Victoria:
correct? (a) Uganda
(a) 1 only (b) Ethiopia
(b) 2 only (c) Tanzania
(c) Both 1 and 2 (d) Kenya
(d) Neither 1 nor 2
Q44. The Sea sparkling observed off the Mumbai’s
Q40. With reference to ‘ Satkosia Tiger reserve ‘ , coast is because of:
consider the following statements : (a) Presence of algae
1. Satkosia is the meeting point of two bio- (b) Presence of fluorescent sand
geographic regions of India; the Deccan Peninsula (c) Presence of light emitting chemicals
and the Eastern Ghats, contributing to immense
(d) Presence of fluorescent marine animals
biodiversity.
2. River Godavari passes through it.
Q45. ‘Colony collapse disorder’ refers to:
Select the correct answer using the codes given
(a) The phenomenon in which human colonies are
below :
damaged because of earthquakes.
(a) 1 only
(b) The phenomenon under which human colonies
(b) 2 only
are being affected by GLobal warming.

FOCUS | June 2018 | RAU’S IAS 139

Download from:- www.UPSCPDF.com


https://t.me/UPSC_PDF www.UPSCPDF.com https://t.me/UPSC_PDF

(c) The phenomenon under which coral reefs are Q48. Consider the following statements:
being damaged due to coral bleaching. (1) Article 51A (Fundamental Duties) of the
(d) The phenomenon that occurs when the majority Constitution requires to promote and propagate
of worker bees in a colony disappear. the National Anthem and the National Flag.
(2) It does not contain the concept of National Song.
Q46. Black Panther is found in which of the Which of the statements given above is/are
following reserves? correct?
(a) Bhadra Tiger Reserve (a) 1 only (b) 2 only
(b) Dandeli-Anshi Tiger Reserve (c) Both 1 and 2 (d) Neither 1 nor 2
(c) Panna Tiger reserve
(d) Achanakmar Tiger Reserve Q49. The Sariska Tiger Reserve is located in the
state of

Q47. With reference to ‘Kigali Agreement’, consider (a) Madhya Pradesh


the following statements: (b) Chhattisgarh
1. Kigali Agreement amended the 1987 Montreal (c) Jharkhand
Protocol and aims to phase out all Ozone (d) Rajasthan
depleting substances.
2. Kigali agreement will be a legally binding on Q50. The Bhitarkanika National Park, a unique
member countries. ecosystem, and which is highly dynamic and at
Select the correct answer using the codes given the same time fragile, is located in
below: (a) Andhra Pradesh
(a) 1 only (b) Madhya Pradesh
(b) 2 only (c) West Bengal
(c) Both 1 and 2 (d) Odisha
(d) Neither 1 nor 2

1. Answer (c) 13. Answer (a) 25. Answer (a) 37. Answer (d)

2. Answer (d) 14. Answer (c) 26. Answer (b) 38. Answer (a)
3. Answer (b) 15. Answer (c) 27. Answer (d) 39. Answer (c)
4. Answer (b) 16. Answer (d) 28. Answer (a) 40. Answer (a)
5. Answer (c) 17. Answer (d) 29. Answer (c) 41. Answer (a)
6. Answer (b) 18. Answer (a) 30. Answer (c) 42. Answer (d)
7. Answer (a) 19. Answer (b) 31. Answer (b) 43. Answer (b)

8. Answer (c) 20. Answer (c) 32. Answer (d) 44. Answer (a)

9. Answer (c) 21. Answer (a) 33. Answer (c) 45. Answer (d)

10. Answer (d) 22. Answer (b) 34. Answer (a) 46. Answer (c)

11. Answer (d) 23. Answer (c) 35. Answer (d) 47. Answer (b)

12. Answer (c) 24. Answer (d) 36. Answer (c) 48. Answer (a)

49. Answer (d)

50. Answer (d)

FOCUS | June 2018 | RAU’S IAS 140

Download from:- www.UPSCPDF.com


https://t.me/UPSC_PDF www.UPSCPDF.com https://t.me/UPSC_PDF

Here, we are sharing Essay Type Questions for self-


practice. These questions have been framed on
important issues covered in editorials drawn from
Essay Type Questions leading newspapers. A reader is expected to read the
editorals, understand & analyse them, and practice
for mains gs papers I, II & III the question given above each editorial in 200 words
to command on issues relevant for General Studies
Papers- I, II, III & Essay.

Q1. Job creation is the only way to ward off the demand for more reservations. Comment.

Quick Edit Times of India | Polity and Society

During the course of a public address, Prime Minister Narendra Modi reiterated his government’s decision to slice and
dice the existing reservation of 27% for Other Backward Classes (OBCs) in central government jobs.

The context to this announcement is that last year NDA appointed a commission to suggest a “scientific approach” to
subcategorize the OBC pool. The government pointed out that nine states had subcategorized earlier. Therefore, the
aim was to complement what they had done with another exercise in slicing and dicing the OBC list to help
communities who had not received adequate benefits of reservation.

Modi’s announcement will at best serve narrow political ends. Slicing and dicing of the OBC list by states have not
helped them deal with a jobs crisis for the youth. There are a couple of reasons for it.

Government jobs are a tiny fraction of what India’s burgeoning work force needs. Most job seekers end up in the
private sector where a narrow identity based on caste doesn’t matter. Second, the slicing and dicing serves to cover
abysmal failure of all governments, which is providing quality education to children.

Education provides the most effective opportunity for upward mobility. NDA’s current exercise, therefore, will only
serve to harden narrow caste-based identity without solving India’s jobs crisis..

Key words: 1. Other Backward Classes (OBCs), 2. subcategorize

Q2. Fighting misinformation requires better policing and sustained rational and reasoned
Information, Education and Communication (IEC). Analyse.

Fake news kills Times of India | Governance and Social Justice

The spate of lynchings in recent days and weeks reported from south Indian states like Karnataka, Telangana, Tamil
Nadu and Andhra Pradesh – on account of rumours of child lifting circulating through WhatsApp messages – deserves
urgent attention of governments across India. The speed and extent of dissemination of WhatsApp messages makes
them a clear and present danger when the intent is to subvert law and order. The combination of medieval mind-sets
and post-modern technology that cloaks rumour as news is at the root of these lynchings.

Add churnings in a country that is being increasingly polarised to serve political ends – leading to phenomena such as
antipathy to migrant labourers in the south and gau raksha movement in north India – and it’s a lethal mix. The
rumours and hysteria find their own victims: in most cases outsiders unlucky enough to fall in the way of a bloodthirsty
mob. Among the victims have been Muslim cattle traders, poor jobseekers from other states, and women with mental
ailments. The Union government will recruit social media monitors for all districts but it is doubtful these can defang
fake news without improving policing and awareness.

The worst possible response would be for government to crack down on mainstream media, ostensibly for the
purpose of checking fake news. This would lead, more than anything else, to the proliferation of fake news and

FOCUS | June 2018 | RAU’S IAS 141

Download from:- www.UPSCPDF.com


https://t.me/UPSC_PDF www.UPSCPDF.com https://t.me/UPSC_PDF

rumours. Instead, police personnel at grassroots are best placed to detect the propagation of such messages and the
formation of mobs; they have to be trained for this new dimension of beat/ community policing. Awareness needs to
be created about fake news on social media, and people must wait for authoritative corroboration. But mischief
mongers and the existence of social biases and divides leave no room for complacency.

Key words: 1. Community policing

Q3. Heat waves are a public health emergency, so our country needs to develop a comprehensive
response. Give your opinion.

Burning issue Times of India | Environment

Indian summers have been sizzling in recent years. Worldwide, 2016 and 2017 have been the hottest years in record.
South Asia is especially susceptible to heat waves, and at current emission levels, heat wave frequency could increase
by about 75 times. Cities tend to trap heat because of paved surfaces and lack of tree cover. In 2015, over 2,241 people
died in India because of severe heat. The most vulnerable are the poor, who live and labour outdoors, and the elderly.
Animals are also at high risk. These mortality numbers often understate the problem, because people die of direct
heat stroke, or because it triggers cardiovascular collapses.

Though heat waves have been more intense, it is only recently that the government has begun to devote financial and
infrastructural resources to dealing with them. The Indian Meteorological Department started issuing advisories a
couple years ago, with 15 day forecasts and regular updates. But we need central, state and local governments to
coordinate adaptation strategies. While depleting tree cover and water scarcity are macro-level problems to be tackled,
cities must create local and immediate action plans – Ahmedabad pioneered one in 2013, and Odisha and Maharashtra
have made their own.

Emergency response needs to be improved – cooling centres must be created in public spaces, water distribution must
be arranged for, health workers and hospitals must be prepared, ambulances should carry icepacks. It is also up to
communities to protect themselves, to teach people how to minimise exposure, stay hydrated, and how to deal with
heat exhaustion, cramps and heat stroke. There must be an effort to adjust the hours for strenuous work, where
possible. A comprehensive response will call upon everyone to pitch in, government, businesses and civil society must
pool their efforts to stay cool.

Key Words: 1. Heat stroke, 2. Indian Meteorological Department

Q4. Corporate Social Responsibility goes beyond the provisions of Companies Act, 2013. Critically
analyse.

Pollution not an externality, Vedanta Economic Times | Economy, Polity, Social Justice and Environment

It is extremely unfortunate that a dozen people have been killed and many more injured over a polluting copper
factory in southern Tamil Nadu’s Thoothukudi. It reflects failure of policy, failure of policing, failure of politics and
corporate disregard for the environment and laws that regulate the impact of human action on nature. Payment of
compensation to the victims of police violence will not make up for such failure. Copper smelting creates pollution, as
do many industries.

This means that active measures must be taken to collect, treat and dispose of the pollutants in a fashion that does not
harm the water, air or earth around the factory or the people who live there, not that society can do without those
industries. There was a time when an industrial unit could consider pollution to be an externality, something it created
but was not responsible for. Environmental, specifically, pollution control regulation seeks to internalise the cost of
pollution. No enterprise should try to avoid or reduce this cost by compromising on mitigation and abatement.

FOCUS | June 2018 | RAU’S IAS 142

Download from:- www.UPSCPDF.com


https://t.me/UPSC_PDF www.UPSCPDF.com https://t.me/UPSC_PDF

It is failure on this count that has allowed popular opposition to build up against the Sterlite copper plant in
Thoothukudi. It is evident that people have little faith in either the company’s promise to contain pollution or the
government’s ability to enforce pollution control. The cost of breaching public trust is high.

It is strange that some non-government organisations had to lead the protests. A responsible political party would
have protested against the pollution rather than against the factory, which produces vital jobs and income, besides
pollution. Vedanta, which owns Sterlite, should sack the present management, bring in a new team, invest in pollution
control, engage the public and salvage its reputation and the plant.

Key words: 1. Copper smelting

Q5: Biofuels can serves multiple purpose such as pollution control, reducing oil import dependency,
rationalising oil consumption prices and making villages energy self-sufficient. Give your opinions.

Biofuels part of low carbon transition Economic Times | Economy and Environment

Against the backdrop of soaring oil prices, it is notable that the Union Cabinet has given its green signal for a new
biofuels policy designed to use crop and municipal waste to produce ethanol and biodiesel for blending with petrol and
diesel. Blending was 3.3% for ethanol with petrol last fiscal, and the upside can be promising waste-to-wealth potential
of over Rs. 1 lakh crore that can reduce CO2 emissions by three million tonnes annually. Short of a miraculous advance
in electricity storage, India will need to work out such low-carbon paths to fully sustainable growth strategies.

In parallel, there’s a huge potential to significantly boost blending, say, up to 15% by 2022, with a more viable fuel like
methanol, by using domestic coal. The plan now is to set up a dozen new advanced biofuel refineries, to use waste
biomass as feed-stock. It is also noteworthy that the Centre reportedly plans to foot the entire Rs. 685-crore bill to
prevent burning of crop stubble in Punjab later this year, so as to stem air pollution in Delhi and much beyond. The
target is to step-up ethanol blending of petrol to 10%, although blending biodiesel with diesel remains minimal, using
inedible oilseeds like jatropha and used cooking oils. Besides, upwards of 62 MMT of municipal solid waste is
generated annually nationwide, and the scope to use crop residue for biofuels is considerable.

A NITI Aayog study avers that methanol, a single-carbon compound, can serve as our best alternative for fuel blending.
It is a highly efficient fuel that can be routinely blended with petrol, and that can be further converted to dimethyl
ether, a diesel alternative that can be blended with LPG as well. Given that India is the third-largest oil importer, a pilot
project for methanol brooks no delay.

Key words: 1. ethanol, 2. methanol, 3. jatropha,

Q6. Rather than disinvestment in nation’s silver, the right option is to improve the management and
work methodology in the PSUs to turn the tide in their favour. Critically analyse.

A flop foretold Indian Express | Economy

That the strategic disinvestment of Air India would be a failure was a foregone conclusion well before the end of the
deadline for the expression of interest from bidders. India may be one of the fastest growing aviation markets, besides
China, which is reflected in the growth in passenger traffic in almost all the major metros and capacity constraints at
the country’s top airports. Yet few would have expected any investor to buy an airline weighed down by a debt of over
Rs 33,000 crore, a bloated workforce, with the government planning to retain 24 per cent of Air India equity.

The government would be better served if it jettisons attempts to tweak the norms for sale and instead adopts the
course which it often advises private firms to follow — the bankruptcy or insolvency route. Today, investors and firms
have realised that it is better to buy distressed firms which are put on the block by creditors rather than put money in a
state-run airline. The bidding for Binani Cement and Bhushan Steel are a case in point as is that of Ruchi Soya, which
may be far more attractive for an investor than an airline with a market share of a little over 13 per cent and legacy
issues. The government’s timing of the sale, too, is awry — with rising fuel prices and competition from nimble low cost

FOCUS | June 2018 | RAU’S IAS 143

Download from:- www.UPSCPDF.com


https://t.me/UPSC_PDF www.UPSCPDF.com https://t.me/UPSC_PDF

carriers squeezing the margins of the bigger players in the industry both in the domestic and global aviation markets
after a stable period of low oil prices and sustained profits. And the latest earnings of some of the listed aviation firms,
including Jet Airways, reflect the hit on account of higher oil prices. Air India is a textbook case of yet another state-
owned firm in India run to the ground after the opening up of the economy and the failure of the state to retreat from
non-strategic and competitive sectors. It is a measure of the operational success of late entrants, such as private peers
Indigo, that they have managed to gain a market share of close to 40 per cent even as the state-run airline continued to
flounder.

As early as 2003, the Naresh Chandra committee had said that deregulation and privatisation were the way forward
in the aviation industry. The neglect by successive governments has come at a huge cost in terms of infusion of public
funds and value destruction. Any more waffling on this will mean shifting the burden further on to taxpayers.

Keywords: 1. Strategic disinvestment, 2. Naresh Chandra committee

Q7. Many hill stations in India are reaching the limits of their ecological carrying capacity due to
tourism. Critically evaluate.

The parched hills Indian Express | Environment

It’s peak tourist season in Shimla and a photograph that’s making news shows people queuing up at the city’s Mall to
get water from tankers. It’s a telling commentary on all that has gone wrong in the hill station this summer. “Give
Shimla time to breathe and recover. If you love Shimla please don’t visit,” reads a message that has gone viral on social
media. The crisis drew the attention of the Himachal Pradesh High Court. It ordered a halt on construction activity and
directed the state’s chief secretary to find out if water can be diverted from the Annadale golf course and the Indian
Institute of Advanced Study, which have huge water storage tanks. These measures may provide immediate relief to
Shimla’s residents, but a longstanding solution to its water woes will require the city to conduct an audit of its water
resources and refurbish its supply infrastructure.

Shimla was built in the 19th century for 25,000 to 30,000 people. The city today has a permanent population of
1,72,000 and a floating tourist population of almost a lakh, which have to make do with a water network designed in
the 1870s. Pipelines laid in the colonial era were never upgraded. During the peak tourist season, the city requires
about 45 million litres of water a day (MLD). But even at its best, the Shimla municipality has never managed to supply
more than 35 MLD. With construction activity depleting Himachal Pradesh’s forests, the state’s aquifers are rarely
recharged adequately to meet the demands of the tourist season. A 70 per cent deficit in rains and scanty snowfall last
winter have brought matters to a head this summer: The availability of water in the city touched a new low of 18 MLD.

Shimla is not the only Indian hill station with a water problem. Ootacamund regularly records a 50-60 per cent water
supply deficit during the tourist season in May-June. And last month, a committee appointed by the Supreme Court to
monitor the tourist carrying capacity of Mussorie found that the town’s peak summer demand is 14.5 MLD, while the
supply is only 7.60 MLD. Carrying capacity studies such as the one undertaken in Mussoorie are, in any case, rare in
Indian hill stations. For tourism to be sustainable in these cities, it’s imperative that they undertake such studies. That’s
the major lesson from Shimla’s current water crisis.

Keywords: 1. Aquifers

Q8. The magic and charisma of Monsoon goes beyond its requirement for Indian agriculture. Give your
opinions.

The downpour Indian Express| Culture and Environment

It is a sight-and-sound show the Subcontinent waits for with bated breath. After the scorching heat of April and May,
punctuated with the occasional downpour, the clouds arrive over the seas off the Kerala coast close to June 1. Then, a
streak of light traces a line over the skies and a rumble breaks out. On land, people take cover as sheets of water

FOCUS | June 2018 | RAU’S IAS 144

Download from:- www.UPSCPDF.com


https://t.me/UPSC_PDF www.UPSCPDF.com https://t.me/UPSC_PDF

sweep past. The exhilaration of getting drenched in the rain soon makes way for worries about leaky roofs, flooding
and the breakdown of normal life. On the seas and the ghats, the monsoon also evokes a sense of fear, and terror; the
rain is merciless as it launches killer waves and triggers landslides. When peace descends after the downpour and the
world turns into a canvas of shades of green, the monsoon assumes a spiritual self; it is a time when man and nature
are in communion.

Traders from the West, Romans and Arabs, rode the monsoon wind, Hippalus, named after the Greek scientist who is
believed to have discovered it, to dock on India’s western coast. Thus began a transaction of material and spiritual
goods — spices, languages, religions including Judaism, Christianity, Islam. In modern times, the monsoon has its use
value primarily in agriculture. The southwest monsoon replenishes the Subcontinent’s water resources; a weak
monsoon spells disaster for the crops and launches a thousand queues at street corner taps. Kerala is now learning to
encash the monsoon by packaging it for tourists. Lazing by the rain-drenched sea to Ayurvedic massages, tourists are
offered a range of options that hold the promise to spice up the local economy.

The Malayalam word for monsoon is Kalavarsham — the downpour of time. Year after year, for centuries, nature has
put up this show, the magic refusing to recede.

Keywords: 1. Hippalus

Q9. The proposal for simultaneous elections involves too many practical difficulties and is nearly
impossible to implement. Give your opinions.

Mission impossible The Hindu | Polity & Governance

The idea of holding simultaneous elections to the Lok Sabha and the State Assemblies appears to have caught the
imagination of the Bharatiya Janata Party-led government at the Centre. Prime Minister Narendra Modi has been
talking about this for some time now. It is not too much of a leap to surmise that he believes that voters are likely to
back the same party in both elections, and that in the absence of a national alternative to his candidature at the Centre,
such a voting pattern may help the BJP across States too. The Law Commission’s move to seek the opinion of the
public, political parties, academicians and other stakeholders, on the proposal appears to be aimed at giving concrete
shape to this political viewpoint. The Commission has released a three-page summary of its draft working paper,
setting out the amendments that may be required in the Constitution and electoral laws. It proposes to put together a
report to forward to the Centre after getting the views of the public. Among its “possible recommendations” is a
“constructive vote of no-confidence”: while expressing lack of confidence in one government, members of the
legislature will have to repose trust in an alternative. It also suggests that premature dissolution of the House could be
avoided if all members sit together and elect a leader. This would entail a temporary waiver of the anti-defection law
so that members could help form a stable government without the fear of disqualification. However, these are reforms
that can be adopted even if simultaneous elections are not held.

In principle, simultaneous elections to the Lok Sabha and State Assemblies have the benefits of saving poll expenditure
and helping ruling parties focus on governance instead of being constantly in election mode. The flip side is that it is
nearly impossible to implement, as it would mean arbitrarily curtailing or extending the term of existing legislatures to
bring their election dates in line with the due date for the rest of the country. This would be the most difficult change to
execute, as such a measure would undermine federalism as well as representative democracy. The Commission has
suggested an alternative: categorise States based on proximity to the next general election, and have one round of
State Assembly polls with the next Lok Sabha election, and another round for the remaining States 30 months later.
This would mean that India would have a set of elections every two and a half years. But governments have been
brought down or have collapsed on their own, leading to mid-term polls in different States and even at the Centre in
different years. Given the difficulties involved in shifting to simultaneous elections, we may have to live with the reality
that some part of the country will go to polls every few months..

Key words: 1. simultaneous elections 2. anti-defection law

FOCUS | June 2018 | RAU’S IAS 145

Download from:- www.UPSCPDF.com


https://t.me/UPSC_PDF www.UPSCPDF.com https://t.me/UPSC_PDF

Q10. What are the different mechanisms which state authorities should adopt in case of violent protest
by public without adopting harsh measures such as firing. Comment.

Entirely preventable The Hindu | Polity & Governance

The protest against the copper smelter plant of Sterlite Copper in Thoothukudi has witnessed its deadliest turn so far,
with the death of 12 people in police firing. It was clear the movement would put up a show of strength on May 22, the
100th day of this phase of protests — in fact, the Madurai Bench of the Madras High Court had predicted that it was
“likely to trigger a law and order situation” and declared that the “protesters do not have any intention of conducting a
peaceful protest”. Yet, the Tamil Nadu government failed to gauge the intensity of what was coming. It is a tragic irony
that such an angry and violent demonstration could have been staged at a time when the plant is not operational and
after the Tamil Nadu Pollution Control Board refused to renew its consent to operate. It raises questions about the
government’s failure to drive this point home forcefully, and casts a doubt about the real intent of some of the
protesters, possibly a small section comprising hardline groups. The immediate task is to compensate the public for its
losses and end the alienation of the affected communities through talks. But the commission of inquiry headed by
retired judge Aruna Jagadeesan must examine why 12 lives were brutally snuffed out, more specifically, the chilling
accusation that snipers were deployed by the police force to pick out protesters in a premeditated manner. Any police
response must be commensurate with the gravity of the situation; there is no place for heavy-handedness and a
disproportionate use of force. The inquiry must establish who gave the orders to fire and on what basis. Also, why the
police failed to intervene well before the protest developed an angry head of steam.

Sterlite stakes claim to be India’s largest copper producer and is a major presence in Tamil Nadu’s industrial mix. But it
has had mixed fortunes over the two decades of its production, including periods when it was under administrative
orders of closure, a Rs. 100-crore fine imposed for pollution by the Supreme Court in 2013, and consistent opposition
from fishermen. Now, there is a fresh injunction and the Madras High Court has restrained it from a proposed capacity
expansion plan. This, together with the decision to not renew consent for operation, gives a moment for pause for all
sides. An urgent process, such as an all-party meeting, is needed to heal the wounds, and infuse confidence in the
community. A credible environmental audit should be undertaken, without compromising on the ‘polluter pays’
principle. The TNPCB, which usually scores poorly on transparency, should commission credible experts to assess the
quality of air and water in Thoothukudi. Only such verifiable measures will build public confidence, and make orderly
industrialisation viable.

Key words: 1. protests 2. Law and order 3. government’s failure 4. Pollution 5. environmental audit 6. polluter pays’ principle

Q11. As per United Nations, New Delhi would edge past Tokyo as the world’s most populous city by
2028. Suggest suitable steps which should be taken by the government to accommodate various needs of
a rising urban population in Indian cities.

Growing cities: Urban governance The Hindu | Polity & Governance

Cities are economically vibrant spaces around the world and draw a large number of rural migrants looking for better
prospects. This is a sustained trend, particularly in developing countries now, as production, jobs and markets get
concentrated. More evidence of this comes from the UN Department of Economic and Social Affairs, which has
released its 2018 Revision of the World Urbanization Prospects. Forecasting for the year 2050, the UN agency
estimates that the percentage of urban residents in India would be 52.8, compared to 34 today, while Delhi would edge
past Tokyo as the world’s most populous city by 2028. India, China and Nigeria are expected to lead other countries
and account for 35% of the projected growth in urban population by mid-century. This forecast frames the challenge
before developing countries, India in particular. Urbanisation in the country is a complex process, since it is defined not
by a constant migration of rural residents but by the flow of workers, mostly men, and the expansion of big cities
through the addition of neighbouring towns. Among governments there is a strong policy emphasis on improving
facilities in rural areas, indicating a political preference for reduced migration to urban centres, although there is a
natural economic magnetism to cities. The imperative before the Centre and State governments is to come up with

FOCUS | June 2018 | RAU’S IAS 146

Download from:- www.UPSCPDF.com


https://t.me/UPSC_PDF www.UPSCPDF.com https://t.me/UPSC_PDF

policies that provide adequate services in the villages, while investing in cities to ensure that their high levels of
productivity and efficiency are not compromised.

Even with only a third of the population living in cities, civic anarchy is rampant in the country. Housing deficits have
led to the proliferation of slums, lack of enforcement of building norms has left the metros heavily congested, and poor
investment in public transport has fuelled unsustainable levels of private vehicle use. Moreover, as recent data
released by the World Health Organisation show, 14 Indian cities are among the top 20 worldwide with the worst air
quality profiles for fine particulate matter of 2.5 micrometres. Most cities are also unable to collect and dispose of
municipal waste scientifically, and simply dump them in the suburbs. Such a dismal scenario can only get worse with
higher population concentrations, unless city governments come into their own. Even two-and-a-half decades after
municipal laws were reformed, elected Mayors lack the stature and authority to introduce urgently needed reforms.
Now is the time to take a fresh look at urban governance. While the Centre’s goal of homes for all by 2022 is laudable, it
is unlikely to be realised without a push from the States, and the launch of schemes driven by innovation and low-cost
approaches. Augmenting rental housing should be a priority within the plan. Integrating green spaces, open commons
and wetlands will make cities cleaner and aesthetically richer.

Key words: 1. rural migrants 2. developing countries 3.World Urbanization Prospects 4. Urban Population 5. flow of workers
6. civic anarchy 7. Housing deficits 8. air quality 9. municipal waste

Q12. Border management is a complex task due to difficult terrain and hostile relations with some
countries. Elucidate the challenges and strategies for effective border management.

Alien versus alien: illegal immigration in Assam The Hindu | Polity & Governance

In Assam, where illegal migration, in fact as well as in exaggeration, has defined the political landscape since the
1980s, public hearings and meetings held by a Joint Parliamentary Committee over the Citizenship (Amendment) Bill,
2016, have inevitably taken place in a charged atmosphere. Parties and civil society groups have argued that the Bill
provides legitimacy to Hindus who have migrated from Bangladesh post-1971. It precludes individuals from six
religious minorities from three “Muslim-dominant countries” (Afghanistan, Bangladesh and Pakistan) from being
defined as “illegal immigrants” under the Foreigners Act, 1946. The intent behind this Bill, promised by the BJP in the
run-up to the 2014 general election, is to clear a path to citizenship for minorities persecuted in the three countries.
The National Register of Citizens, on the other hand, does not distinguish migrants on the basis of religion and
regards all post-March 24, 1971 migrants, irrespective of their religion, as illegal aliens who need to be deported.
Clearly, the Bill is seen by detractors to be breaking the general consensus on the NRC forged after years of political
differences and legal challenges to the Assam Accord, of which the ongoing exercise to update the register is an
outcome. Chief Minister Sarbananda Sonowal has responded saying his government will continue to protect the rights
of the citizens, and hinted that the JPC should take the detractors’ views on board.

The Bill is indeed discriminatory and problematic in limiting accelerated citizenship to non-Muslims. As the case of the
Rohingya highlights, Muslims in neighbouring countries who are fleeing persecution are being denied refuge in India
currently. Besides, the detractors, although some of their objections stem from a native chauvinism, have a point. The
Bill conflates the definition of migrants, people who shift voluntarily, with that of refugees, who are forced to do so
under duress giving them a claim to humanitarian protection. The NRC puts the onus on migrants to prove their status
of residence prior to 1971 based on a series of documents that would lead to registration as a citizen. The Bill seeks to
bring in considerations of religious identity. There are unresolved issues with the NRC process as well. There is the
question of modalities of deportation, which would involve negotiations with Bangladesh. As of now Assam has six
detention centres for illegal migrants. If the NRC process identifies more illegal aliens for deportation, they would have
to be detained in such centres and there is no knowing how long they would have to stay there. Besides, the
implementation of the Bill will mean non-Muslims will not be subject to these steps, thereby clearly discriminating
against Muslims identified as illegal aliens. The Centre needs to apply much more thought before pushing the Bill, for
its contradictions in Assam and for its larger religious assumptions.

FOCUS | June 2018 | RAU’S IAS 147

Download from:- www.UPSCPDF.com


https://t.me/UPSC_PDF www.UPSCPDF.com https://t.me/UPSC_PDF

Key words: 1. Illegal migration 2. Joint Parliamentary Committee 3. National Register of Citizens 4. Assam Accord 5. religious
identity

Q13. India’s party-centric democracy has increasingly become hostage to vulgar display of politics of
intimidation. Discuss the role of Election Commission to deal such situations of vulgar display of money
and power by political parties in power.

Spoiling the party Indian Express | Polity & Governance

Just as it became evident that no political party was likely to get a simple majority, a tweet from Kerala Tourism
triggered a conversation on social media: “After the rough and tumble of the Karnataka verdict, we invite all the MLAs
to unwind at the safe and beautiful resorts of God’s Own Country”. It was prescient. As the political drama unfolded
over who gets to form the government, the Congress and the Janata Dal (Secular), which stitched together an alliance
after the election and staked claim for office, began preparations to shift their MLAs to safe houses even as HD
Kumaraswamy alleged the BJP had offered Rs 100 crore to each of his MLAs. The Congress and JD(S) leaders were
evidently unsure that their MLAs could resist the BJP’s alleged overtures. 113 of them were on the road to a resort in
Hyderabad, under the close watch of party functionaries.

Karnataka has witnessed this scenario before, and so have many other states. The two national parties, Congress and
BJP, as well as regional groups such as the Telugu Desam, AIADMK and the JD(S) have at various times been forced to
shift entire legislative units to hotels and resorts to prevent poaching of their flock. The immorality of the poachers is
self-evident. But what is to be said of political parties that so easily suspect the loyalties of their own legislators? In fact,
the fear of defections had driven the Congress, then holding office with the largest ever parliamentary majority in
India’s history, to pass the the anti-defection law in 1985. The distrust of one’s own legislators is now endemic to most
parties.

It reveals a crisis in India’s party-centric democracy, which is increasingly becoming hostage to big money. The
skyrocketing of campaign expenses and opacity in election financing has transformed the character and conduct of
parties. Even ideology-bound cadre outfits make compromises on candidate selection and solicit funds from dubious
sources to stay in the game. This, in turn, has caused political convictions and affiliations to become fluid and even
susceptible to bribes and inducements. Every party is forced to renegotiate relations with its supporter, sympathiser,
cadre, leader, candidate during the time of the election and, in times of a fractured verdict, with its legislators.

Political parties are aware of this decline but seem unwilling or incapable of arresting the slide and reinforcing their
authority. Much of this is undoing the commendable work that the Election Commission has done to make elections
free and fair.

Key words: 1. simple majority 2. party-centric democracy 3. campaign expenses 4. ideology-bound cadre 5. Immorality

Q14. Implementation of labour reforms is essential for a developing economy like ours. Discuss the
roadblocks being faced by the government to implement drastic labour reforms in India.

Waiting for labour reforms and Godot Economic Times | Polity & Governance

The government reportedly has a blueprint for labour reforms and wants to swiftly implement worker-friendly laws.
A commitment to workers’ welfare is welcome. It includes codes on minimum wages, social security, occupational
safety, health and working conditions, besides providing for easier rules for employers to hire and fire workers.
Apparently, the government wants to put on hold laws that are perceived to go against the interest of workers. Even
the ruling party’s trade unions will oppose such reforms and their implementation would be fraught in an election
year. Political circumstances will allow the rollout of holistic labour reforms only after the general elections, which are a
year away. Labour must be aligned with the dynamics of growth in a globalised economy. Indeed, it should protect
workers, but should also give more flexibility to employers to adjust to market needs. For example, the Wages Bill that
seeks to empower the Centre to set a minimum wage across sectors that states will need to abide by will raise salaries

FOCUS | June 2018 | RAU’S IAS 148

Download from:- www.UPSCPDF.com


https://t.me/UPSC_PDF www.UPSCPDF.com https://t.me/UPSC_PDF

of workers in the unorganised sector. A rise in their incomes will make them significant consumers and drive
domestic demand. Similarly, the draft code on occupational safety seeks to rid the workplace of hazards and ensure
workers’ well-being. This will help raise productivity. The long dither over reforming industrial relations must end.
Companies need more leeway than what they have at present to hire and dispense with labour. The laws for flexible
hiring and retrenchment should factor in a mandatory social security net. The larger point is for workers to adjust to
the industrial structure and practices to match the dynamics of growth. Aligning their interests with those of the
company should be the goal.

Key words: 1. labour reforms 2. worker-friendly laws 3. trade unions 4. globalised economy 5. wages Bill 6. unorganised
sector

Q15. Election Commission cannot afford malfunctioning of VVPAT during the upcoming State Assembly
and Lok Sabha elections. Suggest measures that must be taken by the Commission to ensure conduct of
free, fair and transparent elections in India.

Malfunctioning of VVPAT Times of India | Polity & Governance

The by-elections to four Lok Sabha and 10 assembly seats across the country have left some unanswered questions
regarding the efficacy of Electronic Voting Machines (EVMs) and Voter Verifiable Paper Trail (VVPAT) machines.
Most of the complaints received were from Uttar Pradesh and Maharashtra. Almost one-fifth of VVPAT machines had
to be replaced in UP’s Kairana and Maharashtra’s Bhandara-Gondiya followed by 13.16% in Palghar Lok Sabha seats.
This resulted in opposition parties comprising Congress, SP, RLD, Shiv Sena and NCP knocking on the Election
Commission’s door demanding restoration of ballot paper.

EC maintains the malfunctions were largely due to extreme heat conditions, first-time use by polling staff, and
placement of machines under direct light. The recently concluded Karnataka elections also reported malfunctioning of
4.18 % of the total 57,786 VVPAT machines deployed. But the buck stops with Election Commission (EC) and it should
have ensured proper training and facilities were provided at polling stations. Reverting to ballot paper is not a viable
alternative to EVMs as it would once again lead to political parties patronising criminal elements to influence elections.
While EC must get its act together, the onus is also on political parties to support the institution which is pushing for
more transparency and smooth conduct of elections.

Key words: 1. by-elections 2. Electronic Voting Machines (EVMs) 3. Voter Verifiable Paper Trail (VVPAT) machines 4.
malfunctions 5. Election Commission (EC) 6. ballot paper

Q16. In what way the environment could affect Foreign Direct Investment in the country? Discuss

The impact of environment on FDI The Hindu | Economy

Is foreign direct investment (FDI) linked to easing environmental regulations? The pollution haven hypothesis, which
theorises that global industries flock to places with low environmental regulation, was tested recently in the India by
VinishKathuria, a Professor at the Indian Institute of Technology, Bombay. The findings were published in a study titled
‘Does environmental governance matter for foreign direct investment? Testing the pollution haven hypothesis for
Indian States’ in the Asian Development Review. The results from 21 States show that environmental regulation has little
impact on FDI, which is dependent on other variables of infrastructure and labour instead. In terms of policy, it would
help India if it focussed more on these factors rather than on a blanket change in environmental regulation.

To tabulate the impact of environmental regulation on industry, Kathuria looked at the pollution abatement
expenditures of the manufacturing sector between 2002 and 2010. In theory, higher costs in dealing with pollution
from an industry show stricter environmental norms being in place.

Of the 28 States reviewed, 16 saw pollution abatement costs reduce (including Maharashtra, Gujarat, Tamil Nadu and
Karnataka), while only in 11 States did the costs increase (undivided Andhra Pradesh, Odisha, Haryana, Madhya
Pradesh and Kerala).

FOCUS | June 2018 | RAU’S IAS 149

Download from:- www.UPSCPDF.com


https://t.me/UPSC_PDF www.UPSCPDF.com https://t.me/UPSC_PDF

Kathuria also looked at industrial composition, which previous studies had not dealt with. A State with a high number
of chemical industries, for instance, will have higher environmental costs despite the level of environmental
regulations. To adjust for this, Kathuria created a industrial-composition-adjusted abatement index using unit level
data from the Annual Survey of Industries. The index shows that Chandigarh, Odisha and Karnataka have the highest
environmental stringency, while Bihar, Delhi and the northeastern States have the lowest.

FDI inflows were also compared to the control variables of per capita income of 21 States, manufacturing share of
the State GDP, installed electricity generation capacity, transmission and distribution losses in the power sector (a
reflection of industrial regulations), literacy, proximity to coast, and the readiness of a State to accept investments.

The study’s statistical models employed showed a positive correlation with industrial and economic regulations, while
the study notes: “Environmental stringency does not influence a foreign firm’s location decision when other
infrastructure and market access-related factors are considered.”

Key words: 1. environmental regulation; 2. FDI; 3. abatement index; 4. per capita income;

Q17. To what factors can be attributed to recent fall in the rupee? What implication does the trend have
on the Indian Economy?

Why is the rupee in a free fall? The Hindu | Economy

India’s macroeconomic threats lie exposed as it grapples with the rupee’s slide. The currency sunk to a closing low of
68.07 against the U.S. dollar, its lowest level in 16 months, before recovering slightly the next day. The rupee, already
one of the worst performing Asian currencies, has now weakened 6.2% in 2018. The rise in crude oil prices through
this year, amidst rising geopolitical tensions in West Asia and dwindling global supply, have obviously hurt the rupee
and the trade balance. Meanwhile, despite a depreciating currency, India’s merchandise exports are stumbling instead
of gaining from the opportunity. April clocked a sharp decline in exports from employment-intensive sectors such as
readymade garments and gems and jewellery, according to official data. The trade deficit has consequently widened to
$13.7 billion in April, compared to $13.25 billion in the same month in 2017. The value of oil and petroleum product
imports increased by 41.5% from last year to hit $10.4 billion. U.S. sanctions following Washington’s withdrawal from
the Iran nuclear deal and a June 22 meeting of OPEC should drive oil price trends hereon. Oil prices apart, the
tightening of U.S. monetary policy has almost always spelled trouble for emerging market economies hooked to
Western capital inflows. This time it is no different; capital outflows are scuppering the currencies of many emerging
market economies.

As the U.S. Federal Reserve has come to adopt a more hawkish stance, investors in search of higher risk-adjusted
yields have started to pull money out of emerging markets. Yields on emerging market bonds have risen as investors
sold them off aggressively. The yield on the 10-year bond issued by the Indian government has risen to more than
7.8%, from 7.1% in early April. Foreign portfolio investors (FPIs) pulled out ₹15,500 crore from India’s capital markets in
April, which is the highest monthly outflow since December 2016. Not surprisingly, about two-thirds of the outflow was
attributed to the bond market. The current headwinds from the reversal of capital flows were only to be expected.
India is better placed than countries such as Argentina or Turkey. But that’s no reason to be complacent as external
account risks can get out of hand very quickly. A hike in the RBI’s benchmark interest rates could stem the capital
exodus, but with core inflation picking up and the government keen on a rate cut as a growth catalyst, the RBI has an
unenviable dilemma on its hands. Policymakers, blessed with relatively benign external economic conditions after the
taper tantrum of 2013, will have to find means to spur exports — whether by facilitating swifter GST refunds or taking
on tariff and non-tariff barriers from the developed world. Efforts to diversify India’s energy basket also need greater
stress.

Key words: 1. macroeconomic; 2. currencies; 3. OPEC; 4. Federal Reserve; 5. bonds; 6. capital exodus

Q18. Critically analyse the role of The Pradhan Mantri Jan-Dhan Yojana in ensuring financial inclusion
in the country.

FOCUS | June 2018 | RAU’S IAS 150

Download from:- www.UPSCPDF.com


https://t.me/UPSC_PDF www.UPSCPDF.com https://t.me/UPSC_PDF

The Jan-DhanYojana, four years later The Hindu | Economy

The Pradhan Mantri Jan-DhanYojana (PMJDY), one of the flagship schemes of the present government, was launched in
August 2014. The ‘J’ in JDY is the ‘J’ in ‘JAM’ (Jan Dhan-Aadhaar-Mobile) through which the Economic Survey of 2015
claimed that “every tear from every eye” could be wiped. As the Narendra Modi government enters its fifth year, a
critical evaluation of the scheme is in order, especially since this is one of the schemes through which the government
is trying to battle its anti-poor image.

The recently released World Bank Global Findex data show that 80% of Indian adults now have a bank account, which
is being celebrated as the success of the JDY. While the increase in the proportion of adults having bank accounts is
indeed impressive (80% in 2017 from 53% in 2014), 48% of those who have an account in a financial institution made
no withdrawal or deposit in the past one year.

Financial inclusion is not just about opening bank accounts, but also about using these accounts and providing access
to formal credit. In fact, the major limitation of the JDY has been that while it has managed to get many people to open
bank accounts, there is no commensurate increase in the use of these accounts, availability of formal credit, or
savings in financial institutions, especially among the country’s marginalised and poorer sections.

Access to formal credit


One of the ways in which access to credit can be assessed is the credit-deposit ratio, which tells us how much credit
can be availed per ₹100 of bank deposits by a particular population group. The Reserve Bank of India (RBI) categorises
the population into rural, semi-urban, urban, and metropolitan. We look at the first two regions specifically where one
would expect the poorer beneficiaries to be present in larger numbers. Figure 1 shows that the credit-deposit ratio for
the rural population increased from 41% in 1999 to 66.9% in 2016. However, much of the rise took place before the JDY
was launched, particularly during the tenure of the United Progressive Alliance-1 government, when the credit-deposit
ratio increased from 43.6% in 2004 to 57.1% in 2009. Since 2014, it has more or less stagnated in rural areas and has
deteriorated slightly from 58.2% in 2014 to 57.7% in 2016 for semi-urban populations. Therefore, there is no sign, at
least on this count, of increased access to formal credit that the PMJDY is supposed to have ensured for its
beneficiaries.

To get a more accurate picture of access to credit for poorer populations, we look at the data by credit size. The RBI
provides figures for credit at a disaggregated level in terms of small versus large borrowers. Small borrowers are
defined as those with outstanding loans under ₹2 lakh. And the picture here is no better. The share of small borrowers
in total credit has also been falling during the Modi government (Figure 2). In fact, it has been falling since 2002. While
the decline in the share during the 2004-14 period can be explained by the dramatic rise in corporate credit of large
borrowers, there is no reversal in this trend even after the rate of growth of credit fell in general in more recent times
as a result of rising non-performing assets and the debt overhang of public sector banks. Even in 2016, the best year
under Mr. Modi on this count, it merely matches the lowest rates of growth witnessed during the crisis period of 2009-
10. Based on these trends, it can be argued that there seems to be no increase in access to credit for the poor whether
as a result of the JDY or otherwise. At best, the status quo has been maintained.

To further probe access to credit for small borrowers, we look at these loans in two categories — agricultural credit
and personal loans — which are more likely to be the ones which JDY beneficiaries will be using as against industrial
or other loans. The data show that while the share of small agricultural credit has stagnated during this regime, that of
the small personal loans, which covers home, vehicle, durable goods and so on, has fallen.

Dealing with money lenders

Poor households in India, in the absence of access to formal credit, have to deal with moneylenders who charge
exorbitant rates of interest. This is one of their biggest worries. A recent source that is available in this regard is the
Household Survey on India’s Citizen Environment and Consumer Economy, 2016, which shows that while for the top
1% of the population, one in six are exposed to informal credit, within the poorest section of the population, the figure
is four times as high, with two in three taking credit from informal sources. Access to bank accounts seems to have had
little effect on their dependence on private money lenders.

FOCUS | June 2018 | RAU’S IAS 151

Download from:- www.UPSCPDF.com


https://t.me/UPSC_PDF www.UPSCPDF.com https://t.me/UPSC_PDF

About the issue of money lenders, a study by the RBI in 2017 states: “We document high levels of unsecured debt, and
perhaps more importantly, debt taken from non-institutional sources such as moneylenders. Such debt generates high
costs for Indian households, and... is likely to lead to households becoming trapped in a long cycle of interest
repayments. We note that this phenomenon has been well-documented over the decades, but nevertheless remains
stubbornly persistent.” Therefore, it is not surprising that the report finds that nearly half of the households that take
loans from moneylenders are not able to repay them in time, which is a typical condition for a debt trap.

To conclude, the available evidence presented so far does not suggest that the precarious conditions of indebtedness
that poor people of this country find themselves in has seen any signs of abating as a result of the JDY.

Key words: 1. JAM; 2. Global Fiindex data; 3. Financial inclusion; 4. credit size

Q19. India’s demand for a permanent solution on public stockholding subsidies at the World Trade
Organization (WTO) is being challenged by the U.S. In this context, discuss India’s food security hurdle in
WTO.

Food for thought Business Line | Economy

In a less noticed move, the US earlier this month submitted a petition to the WTO, once again arguing that India’s
agriculture subsidies exceeded WTO-specified limits. The move smacks of bad faith and flawed reasoning. In November
2014, the Modi government got the WTO General Council to agree that public stock-holding for food security would
not be challenged until a ‘permanent solution’ was arrived at. The pact reversed the decision taken at the December
2013 Bali Ministerial that only promised a temporary reprieve to price support programmes. However, the US has not
kept its side of the deal. It has been stonewalling efforts to arrive at a permanent solution to public stock-holding. At
the December 2017 Buenos Aires Ministerial, the developed world distorted the agenda by introducing a range of new
issues. An open discussion on public stock-holding would have worked to the advantage of large farm producers such
as India, China and Thailand and, in fact, highlighted the market-distorting farm subsidies of the developed world. It is
clear that the Trump administration is going all-out to prise open world markets and protect its own producers. Its
disregard for multilateral rules has increased.

India’s procurement programme can be defended on a number of grounds. First, India’s small and marginal farmers,
who account for the bulk of all agriculturists, are exempt from subsidy reduction commitments under Article 6.2 of the
WTO’s Agreement on Agriculture. Second, as India and China have argued in a joint paper released last year, the
developed world benefits from an overall farm subsidy ceiling of 5 per cent on farm output, as against a product-
specific cap of 10 per cent on the produce of developing world. Under the guise of an overall cap, the former is able to
extensively support some crops more than others. A product-specific cap may lead to an overestimation of input
subsidies that are not crop-specific. Third, product-specific price support is based on the difference between the
prevailing price and an ‘external reference price’ (ERF) based on 1986-88 prices. Apart from the ERP being outdated,
the subsidy is calculated in the domestic currency rather than the dollar. Depreciation of the former exaggerates the
subsidy. The developed world subsidises its farmers generously, and indeed distorts world markets, by masking its
subsidies under the ‘blue box’ and ‘green box’ categories. The former is considered ‘minimally distorting’ as it deals
with curtailing production, whereas the latter is deemed ‘non-distorting’ as it covers various income support. A debate
on a ‘permanent solution’ to public stock-holding should lead to a more transparent system.

That said, India should consider prioritising income support over price support. The demand for a ‘pay commission’ of
sorts for agriculturists, raised by farmers’ organisations, is not without merit. We could redesign the food subsidy bill,
but on our terms alone.

Key words: 1. WTO; 2. public stock-holding; 3. price support programmes; 4. Agreement on Agriculture; 5. external reference
price; 6. blue box; 7. green box

Q20. Explain how Flipkart-Walmart deal will affect retail sector in the country? What arrangements
need to put in place to protect interest of various stakeholders in the retail sector?

FOCUS | June 2018 | RAU’S IAS 152

Download from:- www.UPSCPDF.com


https://t.me/UPSC_PDF www.UPSCPDF.com https://t.me/UPSC_PDF

Nothing to fear Business Line | Economy

The scare-mongers have been proved wrong: the growth of big offline retailers and e-commerce giants over the last
decade or so has not driven mom-and-pop shops all across India to the wall. There is, hence, no reason to believe that
the mega deal between Flipkart and Walmart — the first, an e-commerce player that controls about 45 per cent of
India’s e-commerce market, and the second a dominant offline retail giant that, however, has been at the fringes of the
Indian scene with about 20 cash and carry stores — will alter this situation. India’s e-commerce market, at about $18
billion, amounts to about 2.5 per cent of the $650-billion retail economy. Organised offline retail as a whole accounts
for 6-7 per cent of this large pie, despite their having been around for quite a while. Far from devouring the small
players, they are struggling with large overheads as well as the average consumer’s abiding preference for kirana
shops, particularly with respect to buying groceries. India’s retail sector accounts for 8 per cent of all employment, and
contrary to fears, the livelihoods here certainly have not been disrupted over the last decade. In fact, e-commerce has
emerged as an employment provider, an aspect conceded recently by the West Bengal government. As a response to
corporate brick-and-mortar retail, neighbourhood retailers have scaled up and now provide stiff competition, besides
employing a number of shop assistants. So far, India’s retail market, thanks to its sheer size and diversity, has allowed
all variety of players to operate.

The trouble is that policy has not kept pace with this reality. It was only in January this year that India permitted 100 per
cent foreign investment in single-brand retail, while relaxing the 30 per cent local sourcing rule for their Indian
operations. Now, these entities can offset their 30 per cent requirement, if they do the same for their global
operations. The absence of curbs on foreign single-brand retail may well have spurred an offline giants to create
greenfield infrastructure rather than buy up stake in another company. This goes to show that ill-thought out
regulations do not achieve the best outcomes. The Centre should consider relaxing curbs on multi-brand retail, and
allow freer play of market forces seeking to enter India’s buzzing retail space.

Instead of being unduly fixated on ownership issues, regulations should focus on protecting consumers and
producers. Laws and institutions should be wary of anti-competitive practices, such as predatory pricing. Local sourcing
should be implemented in a reasonable way. The key is to strike the right balance between regulation and ensuring a
congenial business climate. The Centre should adopt an integrated approach to promoting FDI in retail, warehousing
and food-processing. This can transform the fortunes of millions of farmers and shield consumers from price
volatility. For too long have reforms in the food marketing chain been held hostage to exaggerated fears, holding up
greenfield investments.

Key Words: 1. food-processing; 2. employment

Q21. What are the implied lessons of the detente in Korean Peninsula for the India-Pakistan conflict?

Lessons from Korea Indian Express | Global Politics

The peace and reconciliation surge in the Korean Peninsula after Panmunjom summit between the leaders of North
and South Korea draws similar resembelance to India-Pakistan conflict. There are some common features between the
conflicts in the Peninsula and the Subcontinent. For one, both the regions were partitioned after the Second World
War. For another, nuclear weapons loom large over them. That is probably where the similarity ends.The differences
between the two regions, however, are far more striking. The religious and voluntary basis of the South Asian partition
stands in contrast to the primacy of the geopolitical in the division of the Korean Peninsula.Both the North and South
Korea are formally committed to the idea of unification. In the Subcontinent, though, the idea of unification is taboo.
Pakistan continues to fear that India is not reconciled to the Partition and wants to undo it. However, unification of
Korean peninsula is not being a reality currently. What is being considered is a peace treaty, an open border, greater
economic and commercial cooperation based on common identity. But, this idea of peace in Indian sub-continent is
based on the idea of a shared identity faces much resistance from the deep state in Pakistan.

The difference in both scenarios lies inthe nuclear question in the Subcontinent and the Peninsula. In Korea, the entire
focus is on the denuclearisation of the Peninsula. While the North that has its own nuclear weapons, the South

FOCUS | June 2018 | RAU’S IAS 153

Download from:- www.UPSCPDF.com


https://t.me/UPSC_PDF www.UPSCPDF.com https://t.me/UPSC_PDF

depends on the extended deterrence offered by the US nuclear arsenal.After the Indian and Pakistani nuclear tests in
the summer of 1998, the world first demanded tfrom both countries that they roll back their nuclear and missile
programmes but it became clear that the international community had to live with a nuclear South Asia. The
relationship between nuclear weapons and peace is also framed differently in the two regions. In Korea,
denuclearisation is seen as a precondition for peace. In South Asia, political reconciliation between India and Pakistan
may help reduce the salience of nuclear weapons.

An equally important difference relates to the way North Korea and Pakistan have defined the strategic value of
nuclear weapons. Kim Jong-Un opened the door for peace negotiations with South Korea and the US after its cabaility
of nuclear detterance. However, the Pakistan army saw them as providing the impunity to conduct a low intensity
conflict against its neighbours — India and Afghanistan — through cross-border terrorism.A major difference between
the two regions is the role of great powers. Although no region in Asia was immune from the great power rivalry
during the Cold War, the degree of that intervention varied quite a bit. The Peninsula was a frontline theatre in the
conflict between the East and West. The US and the People’s Republic of China fought each other in the Korean
Peninsuala during 1950-53.Since the end of the Cold War, the US, China, Russia and Japan have played an active role in
promoting peace in the Peninsula through the framework of six-party dialogue involving the two Koreas. India took the
Kashmir question to the United Nations Security Council. It accepted American and Soviet mediation with Pakistan
during the 1960s. After 1971, India insisted on the bilateral approach with Pakistan and fended off frequent efforts by
the major powers to butt in.

The Korean Peninsula and the Indian subcontinent have to find their own paths to peace and what can be learned
from Korea’s is the need for creative diplomacy. Pakistan can learn from North Korea thatnuclear weapons are not an
end in themselves and that the leverage offered by nuclear weapons could be traded for economic benefit and
normalisation of relations with adversaries.
Keywords: 1. Panmunjom Summit; 2. Denuclearisation; 3. Great Powers

Q22. Assess the influence of the United States on the Wuhan summit between India & China.

Modi-Xi meet in Wuhan Indian Express | Global Politics

At Wuhan, China’s talked on collaborating with India on saving globalisation, defending the WTO, promoting a
multipolar world, and emphasising ‘strategic autonomy’ was very much part of China’s international mobilisation
against Trump. The informal summit in Wuhan between Prime Minister Narendra Modi and President Xi Jinping was
widely billed as India’s ‘reset’ of its China policy. A close look suggests it was Beijing that was really recasting its policy
towards its Asian neighbours. Delhi was merely responding to an unexpected opportunity. China’s reset of its regional
policy was itself a response to the American upheaval under President Donald Trump.

Donald Trump has challenged widespread perception that the balance of power between America and China was
tilting in favour of the latter. Trump’s threat to launch a trade war against Beijing, his denial of market access to the
Chinese technology companies and the demonstration of the ability to upend the politics of the Korean Peninsula have
in a short period altered the international context for China. The dangers to China from Trump’s policies on trade and
security began to dawn on China, wherein it has turned on the charm offensive towards its Asian neighbours. China’s
outreach is not extends to its difficult neighbours India Japan and Vietnam.

At Wuhan, China’s talk on collaborating with India on saving globalisation, defending the WTO, promoting a multipolar
world, and emphasising ‘strategic autonomy’ was very much part of China’s international mobilisation against Trump.
At the same time, China is also trying to find ways to accommodate some of Trump’s concerns on trade. After all,
China’s economic stakes in America are massive.As Trump makes America a big variable in world politics today, it is not
clear if India has fully appreciated the implications. India has appreciated US policy on China and Pakistan but not on
economic, international trade & immigration aspects.

Keywords: 1. Reset of ties

FOCUS | June 2018 | RAU’S IAS 154

Download from:- www.UPSCPDF.com


https://t.me/UPSC_PDF www.UPSCPDF.com https://t.me/UPSC_PDF

Q23. The growing bonhomie among India, Australia & France within the Indo-Pacific region can entail
an economic laden cooperation rather than a security bound cooperation.

New Indo-Pacific Axis Indian Express | Global Politics

Chinese assertiveness and the volatility of American policies has allowed for a new strategic arrangement for stability
and balance with India, France and Australia to join forces. Franceconsiders “Paris-Delhi-Canberra axis” should
become an established regional structure, reflecting an Indo-Pacific “geo-strategic reality in the making. Australia has
also agreed to “together define a joint strategy for the Indo-Pacific, with a concrete roadmap of objectives and actions”.
France intendsfor regular trilateral discussions involving external affairs and defence ministers of India. India, France
and Australia have a striking convergence of security interests, defence capabilities and maritime geography. These
three democracies are also drawn together by values based on respect a rules-based order informed by the sovereign
equality of nations and the need to guard against coercion and interference, whether from states or from terrorism.

France has substantial equities in the Indian Ocean, with territory and force presence. The bilateral military logistic
cooperation agreement signed during President Macron’s recent visit to India is a sign of how rapidly a partnership can
evolve when there is will. France is not only an Indian Ocean player: It has an appreciation of the wider Indo-Pacific as
the global centre of gravity. France has territory in the Pacific and a naval role in both oceans. It has 85 per cent of its
huge maritime economic exclusive zone in the Indo-Pacific, along with 8,000 defence personnel and 1.6 million
citizens.France is thus the European country most engaged and most capable of contributing to a balanced and layered
diplomatic architecture to reduce tensions in this most global of regions. It can do so as a leader in the European
Union, a compelling voice in the liberal democratic order globally, and an enduring strategic power in its own right.

The Indo-Pacific is crucial to the future of all powers. It is made for multipolarity: Too vast for hegemony to be
sustained or successful. Yet that does not mean that the region’s shifting game of power and influence will avoid what
Macron has cautioned as “hegemonic temptations”, involving frictions, risks and unipolar encroachments on others’
independence.Notably, China is extending its interests and influence across the Indian Ocean and also now in the
South Pacific. The geoeconomic overlay of its China’sBelt and Road infrastructure will bring naval access and military
presence, whether or not this is China’s grand strategy. That raises great anxieties, just when others are uncertain
about the trajectory of American power under President Donald Trump and beyond.

This, therefore, is the age of a networked Indo-Pacific security architecture. These links are not exclusively among
America’s allies and established partners. Countries like India, Japan and Australia have strengthened ties with each
other, while cultivating South East Asian friends such as Vietnam, Indonesia and Singapore. This layered approach
involves mutually-reinforcing roles for bilaterals, the quadrilateral and inclusive multilateralism centred on ASEAN.

The three countries are ideally placed to share data to form a common operating picture of the Indian Ocean, watching
for environmental stresses, illegal fishing and other and maritime crimes. This could build on existing cooperation
between France and Australia in the Pacific. In time, they could formalise three-way information sharing on seaborne
traffic of all kinds. Between their island territories — France’s Reunion and Mayotte, Australia’s Cocos and Christmas
islands and India’s Andaman and Nicobar islands — they possess a triangle of some of the most strategic maritime
surveillance real estate in the Indian Ocean. It is in everyone’s interests for other countries to form “a partnership of
equals” with China, informed by principles such as freedom of navigation and overflight and the independence of all
nations. This is not about rejecting China’s rise but about engaging it in a system of rules and mutual respect. New trust
and cooperation among the other powers of the Indo-Pacific will help provide them the means to do so.

Keywords: 1. Paris-Delhi-Canberra axis; 2. India Ocean; 3. Indo-Pacific; 4. Security Architecture

Q24. Assess the implications of unpredictability of US foreign policy under Donald Trump on India’s
foreign affairs.

India’s Diplomacy, Trump Effect Indian Express | Bilateral Relations

FOCUS | June 2018 | RAU’S IAS 155

Download from:- www.UPSCPDF.com


https://t.me/UPSC_PDF www.UPSCPDF.com https://t.me/UPSC_PDF

India’s diplomacy has entered a very intensive phase with the current uncertain international environment that is
throwing up new opportunities as well as fresh challenges for India’s ability to respond effectively to the new
international and regional uncertainty. The current turbulence in the international and regional environment is largely
due to US President Donald Trump’s vigorous unilateralism and a major departure from the traditional American
economic and foreign policies. In response, China, Russia, Japan and Europe are at once trying to find ways to
propitiate Trump as well as hedge against the current volatility in US policies.As Trump demands reciprocity in
commercial relationships to redress America’s massive trade imbalance with the rest of the world, most of America’s
partners are eager to make bilateral deals with Washington. As Trump questions the costs and benefits of alliances,
America’s traditional partners in Europe and Asia have been compelled to consider the logic of strategic autonomy
from the U S. Neither Trump’s allies nor his adversaries can now afford to take Washington for granted.

Trump has also muddied the regional environment in East Asia. His attempt at normalising relations with North Korea
has left much of the region, especially China and Japan stunned. China — long standing ally of North Korea — is
concerned that it might be cut out of the unexpected direct diplomacy between Trump and Chairman Kim Jong-un.
Japan, in contrast, is an ally of the United States. It is deeply is concerned that Trump’s nuclear deal with Kim might
leave Tokyo exposed to the residual North Korean nuclear capabilities. Beijing and Tokyo are also wary about the
consequences of a potential reconciliation between North and South Korea and the resurgence of Korean nationalism.
If the old alliances are under stress in North East Asia, the regional security framework for South East Asia, anchored in
the ASEAN, has come under considerable strain, thanks to the return of the great power rivalries. While the ASEAN
remains central to the region’s future, most countries in the region are insuring against the current strategic
uncertainty by boosting their national defence capabilities and diversifying their strategic partnerships.

The PM’s decision to dispatch General V K Singh, the minister of state for external affairs to North Korea, in the first
high-level diplomatic engagement in two decades, is a reminder of India’s unique relationship with the divided
peninsula. Modi’s visit to Indonesia later this month promises to reverse the prolonged Indian strategic neglect of
Indonesia. Modi’s high-voltage diplomacy towards major powers and Asia, however, is not matched by the
government’s approaches to trade and security. India’s seemingly neuralgic opposition to trade liberalisation threatens
international isolation amidst rearrangement of the global economic order. Delhi’s problems with implementing large
infrastructure projects beyond its borders has also limited India’s ability to deepen economic and military connectivity
with Asia. Delhi’s weak defence industrial base and tentative military diplomacy have prevented it from measuring up
to its own claim on being a “regional security provider”.

Keywords: 1. ASEAN; 2. Regional Security Provider; 3. North Korea

Q25. Assess the relations between India & Indonesia with larger framework of ASEAN.

No longer sea-blind Indian Express |Bilateral Relationship

Despite their proximity, physical and cultural, the strategic distance between Delhi and Jakarta has been incredibly vast.
Past references to Indonesia mark of anti-imperialism and third worldism marked by the Bandung conference in April
1955.. The Bandung political metaphor had, of course, collapsed by the end of the 1950s. Intense friendship between
Delhi and Jakarta turned into mild hostility in the early 1960s. After that the two sides settled down to an extended
period of mutual neglect. A variety of internal, regional and global political developments widened the political gulf
between India and Indonesia. After Bandung, India turned its back on Asia and focused on the non-aligned movement.
Jakarta moved towards the minor variant of Asianism in South East Asia.

Underlying the enthusiasm for the Indo-Pacific in Delhi and Jakarta is a deeper change in both nations. Both of them
have begun to rediscover their long-neglected maritime destiny. Prolonged inward economic orientation and border
disputes in the north west and the north meant India had no time for its seas in the 20th century. After two-and-a-half
decades of reform that has globalised the Indian economy, Delhi is paying more attention to maritime issues.With
thousands of islands, Indonesia is a massive archipelagic nation. But Jakarta until recently saw no reason to think

FOCUS | June 2018 | RAU’S IAS 156

Download from:- www.UPSCPDF.com


https://t.me/UPSC_PDF www.UPSCPDF.com https://t.me/UPSC_PDF

maritime. The benign external environment secured by the United States meant Jakarta could devote most of its
energies on internal territorial consolidation. Now, as the Asian waters open up to great power contestation, Jakarta
can no longer remain sea blind. A little over three years ago, President Jokowi unveiled the ambition to turn Indonesia
into a “global maritime fulcrum” by taking advantage of its special geographic position as the land bridge and sea link
between the Indian and Pacific Oceans.

As they respond to the long-ignored maritime imperative, Delhi and Jakarta may have finally found an anchor for their
strategic partnership. Although India and Indonesia have a shared sea frontier in the Indian Ocean, there was little
maritime business between the two. Today, India & Indonesia have the opportunity to build a peaceful and prosperous
“maritime mandala” in the heart of the Indo-Pacific through a number of steps. These include developing shipping
links, building new ports, promoting a blue economy in the Andaman Sea, and advancing cooperative security
framework for the Malacca Straits and the Bay of Bengal.

Keywords: 1. Bandung Conference; 2. Maritime Mandala; 3. Global Maritime Fulcrum; 4. Indo-Pacific

FOCUS | June 2018 | RAU’S IAS 157

Download from:- www.UPSCPDF.com


https://t.me/UPSC_PDF www.UPSCPDF.com https://t.me/UPSC_PDF

Here, we are sharing case studies for self- practice. These


Case Studies case studies are drawn from various aspects of professional
and personal experiences, and are helpful for General
For mains Gs paper IV Studies- Paper IV.
Do practice them!

Case 1: You are a young, aspiring and sincere employee in a Government office working as an assistant, to the
director of your deportment. Since you’ve joined recently, you need to learn and progress. Luckily your superior
is very kind and ready to train you for your job. He is a very intelligent and well-informed person having
knowledge of various departments. In short, you respect your book and are looking forward to learn a lot from
him.
Since you’ve good tuning with the boss, he started depending on you. One day due to ill health he invited you at
his place for finishing some urgent work. You reached his house and you heard shouting noises before you could
ring the bell. You waited for a while. After entering, boss greeted you and explained the work. But you were
constantly disturbed by the crying of a woman. At last, you inquired with the boss but his answer did not satisfy
you. Next day, you were compelled to inquire further in the office and found out that his behavior is very had at
home with his wife. He also heath up his wife.
His wife is not well educated and is a simple woman in comparison to her husband. You see that though your
boss is a nice person in the office, he is engaged in domestic violence at home. In such a situation, you are left
with the following options. Analyse each option with its consequences.
a. Just ignore thinking about it because it is their personal matter.
b. Report the case to the appropriate authority.
c. Your own innovative approach towards the situation.

Case 2: You are the Executive Director of an upcoming InfoTech Company which is making a name for itself in
the market.
Mr. A, who is a star performer, is heading the marketing team. In a short period of one year, he has helped in
doubling the revenues as well as creating a high brand equity for the Company so much so that you are thinking
of promoting him. However, you have been receiving information from many corners about his attitude towards
the female colleagues; particularly his habit of making loose comments on women. In addition, he regularly
sends indecent SMS’s to all the team members including his female colleagues.
One day, late in the evening, Mrs. X, who is one of Mr. A’s team members, comes to you visibly disturbed. She
complains against the continued misconduct of Mr. A, who has been making undesirable advances towards her
and has even tried to touch her inappropriately in his cabin. She tenders her resignation and leaves your office.
What are the options available to you?
1. What are the options available to you?
2. Evaluate each of these options and choose the option you would adopt, giving reasons.

Case 3: There is a disaster-prone State having frequent landslides, forest fires, cloudbursts, flash floods and
earthquakes, etc. Some of these are seasonal and often unpredictable. The magnitude of the disaster is always
unanticipated. During one of the seasons, a cloudburst caused devastating floods and landslides leading to high
casualties. There was major damage to infrastructure like roads, bridges and power generating units. This led to

FOCUS | June 2018 | RAU’S IAS 158

Download from:- www.UPSCPDF.com


https://t.me/UPSC_PDF www.UPSCPDF.com https://t.me/UPSC_PDF

more than 100000 pilgrims, tourists and other locals trapped across different routes and locations. The people
trapped in your area of responsibility included senior citizens, patients in hospitals, women and children, hikers,
tourists, ruling party’s regional president along with his family, additional jail. As secretary State and prisoners in
jail.

As a civil services officer of the State, what would be the order in which you would rescue these people and why?
Give justifications.

FOCUS | June 2018 | RAU’S IAS 159

Download from:- www.UPSCPDF.com


https://t.me/UPSC_PDF www.UPSCPDF.com https://t.me/UPSC_PDF
https://t.me/UPSC_PDF www.UPSCPDF.com https://t.me/UPSC_PD

You might also like